Download - Ευκλειδης Β 83

Transcript
Page 1: Ευκλειδης Β 83
Page 2: Ευκλειδης Β 83

ΕΛΛΗΝΙΚΗ ΜΑΘΗΜΑΤΙΚΗ ETAIPEIA Τ ε ύ χ ο ς 83 - Ιανουάριος - Φ ε β ρ ο υ ά ρ ι ο ς - Μάρτιος 2012- Ευρώ: 3,50

e-mail: [email protected] www.hms.gr

ΜΑΘΗΜΑΤΙΚΟ ΠΕΡΙΟΔΙΚΟ ΓΙΑ ΤΟ ΛΥΚΕΙΟ

ΠΕΡΙΕΧΟΜΕΝΑ Γενικά θέματα .; Οι Πυθαγόρειοι

.; Η Ιερή Γεωμετρία και η κοινωνία .; Μαθηματικές Ολυμπιάδες -1 Homo Mathematicus .; Άλγεβρα: .; Γεωμετρία:

.; Άλγεβρα:

.; Γεωμετρία: -1 Κατεύθυνση: .;

Α' Τάξη Συναρτήσεις Εγγεγραμμένα Σχήματα Αναλογίες - Ομοιότητα

Β' Τάξη Εκθτική Λογαριθμική Συνάρτηση Λογαριθμικές - Εκθετικές Ανισώσεις Κανονικά Πολύγωνα- Κύκλος Κωνικές Τομές Η Διοφαντική εξίσωση αχ+βψ=γ

Γ' Τάξη .; Γενικ� Παιδεία: Θέματα Πιθανοτήτων -1 Κατευθυνση: Η ανισότητα στα προβλήματα

της ανάλυσης Δραστηριότητες .; Ερευνητικές Εργασίες -1 Στήλη του Μαθητή -1 Ο Ευκλείδης προτείνει ... -1 Το Βήμα του Ευκλείδη .; Τα Μαθηματικά μας Διασκεδάζουν

ΕΚΔΟΣΗ ΤΗΣ ΜΑΘΗΜΑτtΚΗΣ ΕΥΑΙΡΕΙΑΣ ΠΑΝΕΠΙΣΤΗΜΙΟΥ 34 106 79 ΑΘΗΝΑ

Τιt-.: 210 3617784-3616532 Fox: 2103641025

Εκτελεστική Γραμματεία Πρόεδρος:

Τασσόπουλος Γιώργος

Εκδότης: Αντιπρόεδροι: Καλογερόποuλος Γρηγόριος Ευσταθίου Βαγγέλης Διευθυντής: Κερασαρίδης Γιάννης Κρητικός Εμμανουήλ Γ -_ _ ραμματεας: Ε�ιμελεια _Εκδοαης:

Χριστόπουλος Παναγιώτης Ζωτος Βαγγελης

Α ά Δ Μέλη: ργuρ κης • Κωδικός ΕΛ.ΤΑ.: 2055 ISSN: 11 05 - 7998 Στεφανής Π.

Ταπεινός Ν.

1 5 9

27

31 30 34

37 41 43 47 52

57

61

65 67 68 71 76

Γ ράμμα της Σύνταξης Αγαπητοί μαθητές και συνάδελφοι,

Σας ευχαριστούμε για την ανταπόκριση που δείχνετε

στο περιοδικό. Οι σημαντικές εργασίες σας, βοηθούν στην

βελτίωση του. Υπάρχουν αρκετά άρθρα με επίκαιρο ενδια­

φέρον προς δημοσίευση, για τα επόμενα τεύχη.

Θυμίζουμε ότι τα άρθρα που στέλονται σε ηλεκτρονική

μορφή βοηθούν πολύ στην επιμέλεια έκδοσης. Σύντομα θα

έχετε στα χέρια σας και το 4ο τεύχος για μια ολοκληρω­

μένη επανάληψη της ύλης των εξετάσεων

Με ευχές για υγεία και πρόοδο

Ο πρόεδρος της Συντακτικής Επιτροπής: Γιώργος τασσόπουλος

Ο αντιπρόεδροι: Βαγγέλης Ευσταθίου, Γιάννης Κερασαρίδης

Συντακτική επιτροπή Αθανασόπουλος Γεώργιος Κόντζιας Νίκος Σdi'rη Εύα Αναστασίου Γιάννης Κστσιφάκης Γιώργος Σταϊκος Κώστας Ανδρουλακάκης Νίκος Κουτρουμπέλας Κώστας Στάϊκος Παναγιώτης Αντωνόπουλος Νίκος Κυριαζής Ιωάννης Στρατής Γιάννης

Αργυράκης Δημήτριος Κυριακόπουλος Αντώνης Ταπεινός Νικόλαος

Βακαλόπουλος Κώστας Κυριακοπούλου Κων/να Τασσόπουλος Γιώργος

Ευσταθίου Βαγγέλης Κυβερνήτου Χρυστ. Τζιώτζιος Θανάσης

zαχαρόπουλος Κων/νος Λαζαρίδης Χρήστος Τριαντάφυλλος Γιώργος

Ζώτος ΒαγγεΛης Λάππας Λευτέρης Τριάντος Γεώργιος

Κακκαβάς Απόστολος Λουρίδας Σωτήρης Τσαγκάρης Ανδρέας Μαλαφέκας Θανάσης Τσατούρας Ευάγγελος

Καλίκας Σταμάτης Μανωλάκου Σταματική Τσικαλουδάκης Γιώργος

Καμπούκος Κυριάκος Μαυρογιαννάκης Λεωνίδας Τσιούμας Θανάσης ΚανεΛλος Χρήστος Μενδρινός Γιάννης Τυρλής Ιωάννης Καραγκούνης Δημήτρης Μεταξάς Νικόλαος ΦανεΛη Άννυ Καρκάνης Βασίλης Μπρίνος Παναγιώτης Χαραλαμπάκης Ευστάθιος Κατσούλης Γιώργος Μυλωνάς Δημήτρης Χαραλαμποπούλου Λίνα Κερασαρίδης Γιάννης Μώκος Χρήστος Χαραλάμπους Θάνος

Θ Καρδαμίτσης Σπύρος Πανδής Χρήστος Χριστιάς Σπύρος

' Κηπουρός Χρήστος Πανουσάκης Νίκος Χριστόπουλος Θανάσης

11" //,Ιδρ,υμα Κλάδη Κατερίνα Πουλιάσης Παναγιώτης Χριστόπουλος Παναγιώτης

/ Ωναση Κονόμης Άρτι Ρέγκλης Δημήτρης Ψύχας ΒαyyεΛης

χορηγός έκδοσης ' Η έγκαιρη πληρωμή

της συνδρομής βοηΘάει cπην έκδοση του περιοδικού

••••••••••••••••••••••••••••••••• • •••••••••••••••••••••••••••••••••••••••••••• • Τα διαφημιζόμενα βιβλία δε σημαίνει όrι προτείνονται από την Ε.Μ.Ε. • Οι συνεργασίες, [τα άρθρα, οι προτεινόμενες ασκήσεις, οι λύσεις ασκήσεων κλπ.] πρέπει να στέλνοντα�αι;, στmφεία της

Ε.Μ.Ε. με την ένδειξη "Για τον Ευκλείδη Β'". Τα χειρόγραφα δεν επιστρέφονται. liJ!LilE:f6[Λf....:. Ετήσια συνδρομή (12,00 + 2,00 Ταχυδρομικά = ευρώ 14,00). Ετήσια συνδρομή για Σχολεία ευρώ 12,00

Το αντίτιμο για τα τεύχη που παραγyέλνοvται στέλνεται: (1 ). Με απλή ταχυδρομική ετπταyή σε διαταγή Ε.Μ.Ε. Τ αχ. Γραφείο Αθήνα 54 Τ.Θ. 30044 (2). Στην ιστοσελίδα της Ε.Μ.Ε., όπου υπάρχει δυνατότητα τραπεζικής συναλλαγής με την τράπεζα EUROBANK (3). Πληρώνεται στα γραφεία της Ε.Μ.Ε. (4). Με αvπκαταβολή, σε εταιρεία ταχυμεταφορών στο χώρο σας, κατά την παραλαβή.

Εκτύπωση: ROTOPRINT IA. ΜΠΡΟΥΣΑΛΗ & Σ/Α EEI. τηλ.: 210 6623778-358 Υπεύθυνος τυnογpαφείου: Δ Παπαδόπουλος

Page 3: Ευκλειδης Β 83

Οι Πυθαyόρειοι: 1 . τί είναι οι φυσικοί αριθμοί, κατά τους Πυθα­

γορείους, ποιές είναι οι ιδιότητές τους και ποιό ρόλο διαδραματίζουν στην πυθαγόρεια οντολογία ;

2 .Σε τι συνίσταται η έννοια της aσυμμετρίας και ποιές ήταν οι συνέπειες της ανακάλυψης αυτής για το κοσμοείδωλο των Πυθαγορείων ;

3. Πώς συνέβαλε η γεωμετρική θεωρία περί κα­νονικών πολυέδρων στην αντίληψη των Πυ­θαγορείων για τη Φύση ;

Πυθαγόρας - Πυθαγόρειοι Στην πρώιμη ελληνική διανόηση, δύο κύρια

ρεύματα της παράδοσης χαρακτηρίζονται κατά τη μεταγενέστερη αρχαιότητα ως ιωνικό και ιταλι­κό1. Το δεύτερο αρχίζει με τον Πυθαγόρα που, αν και Έλληνας της Ανατολής από γεννησιμιού του2, άφησε το πατρικό του νησί, τη Σάμο, νωρίς και μετανάστευσε στην Νότιο Ιταλία περί το 350 π.χ. , όπου εγκαταστάθηκε και ίδρυσε την αδελφότητά του, στην πόλη του Κρότωνα. Αυτήν την αδελφό­τητα κυνήγησαν και διασκόρπισαν για πολιτικούς λόγους και ως εκ τούτου τον 5° αιώνα βρίσκουμε πυθαγόρειες κοινότητες σε διάφορα σημεία του ελληνικού χώρου.

Για τους Πυθαγορείους, το κίνητρο προς τη φι­λοσοφία δεν ήταν ό,τι για τους Ίωνες, δηλαδή η απλή επιστημονική περιέργεια. Αποτελούσαν θρη­σκευτική αδελφότητα και αυτό είχε ορισμένες συ­νέπειες. Ορισμένες τουλάχιστον από τις θεωρίες τους θεωρούνταν απόρρητες και δεν έπρεπε να τις μάθουν οι βέβηλοι3 • Ο ίδιος ο ιδρυτής είχε αγιο­ποιηθεί ή τον έβλεπαν σαν ημίθεο.

Αυτό σήμαινε ότι εξ αρχής γύρω του δημιουρ­γήθηκε μία αχλή από θαυμαστές διηγήσεις, από τις οποίες μας είναι δύσκολο να ξεδιαλύνουμε ποια ήταν πραγματικά η ζωή και η διδασκαλία του Πυ­θαγόρα. Το πρώτιστο και ιδιαίτερο ενδιαφέρον του

1 Guthήe, W.K.C., Οι Έλληνες φιλόσοφοι, από τον Θαλή ως τον Αριστοτέλη, Παπαδήμας 2001, σελ. 41 2 Guthήe, W.Κ.C., ό.π., σελ. 41 3 Kahn, C.H., Ο Πυθαγόρας και οι Πυθαγόρειοι, Ενάλιος 2001, σελ.47

Αγγελική Αποστόλου

ήταν τα Μαθηματικά. Και μπορούμε να θεωρή­σουμε βέβαιον πως αυτό δεν σημαίνει απλώς ότι έτρεφε δεισιδαιμονίες περί τους αριθμούς, αλλά ότι έκανε ανακαλύψεις ολοκληρωτικά και εκπλη­κτικά καινούργιες και προόδους πραγματικές και σημαντικές στον τομέα αυτόν. Ο ίδιος δημιούρ­γησε μία σχολή στοχαστών με έντονο επιστημονι­κό προσανατολισμό, η οποία συγχρόνως χαρακτη­ριζόταν από μία προτίμηση, που άγγιζε το όριο της θρησκευτικής πίστης, ή και προκατάληψης4, με το να αναγάγει το οικοδόμημα της φύσης στην καθο­λική λογική . Οι μαθητές τη Σχολής σχημάτισαν μία κοινότητα που έμοιαζε με θρησκευτικό τάγμα.

Οι διδασκαλίες των Πυθαγορείων ήταν στο με­γαλύτερο μέρος τους υπερβολικά δυσνόητες, ώστε να είναι κατανοητές μόνο από εκείνους που ανή­καν στην Σχολή . Το γεγονός ότι δεν υπάρχει καμ­μία καταγεγραμμένη μαρτυρία σχετικά με τις δι­δασκαλίες των Πυθαγορείων, έως την εποχή του Φιλολάου, δεν πρέπει να αποδοθεί στην εχεμύθεια την οποία ορκίζονταν οι μαθητές της Σχολής αλλά στην απουσία στοιχείων. Αυτή τη δυσκολία, όπως φαίνεται, αντιμετώπιζε και ο Αριστοτέλης, δεδο­μένου ότι ήταν δύσκολο να αποσαφηνιστεί ποιο μέρος της πυθαγόρειας φιλοσοφίας μπορεί να απο­δοθεί με βεβαιότητα στον ιδρυτή της Σχολής. Είναι σαφές ότι δεν γνώριζε τίποτε σχετικά με συγκε­κριμένες διδασκαλίες ηθικού ή φυσικού περιεχο­μένου, οι οποίες αποδίδονται στον ίδιο τον Πυθα­γόρα και όταν ομιλεί σχετικά με το πυθαγόρειο σύστημα αναφέρεται πάντοτε σε αυτό χρησιμο­ποιώντας τον όρο «οι Πυθαγόρειοι» και άλλες φο­ρές, «οι λεγόμενοι Πυθαγόρειοι»5•

Πυθαγόρειο Κοσμοείδωλο Φυσικοί Αριθμοί Ο Αριστοτέλης, στο έργο του «Μετά τα φυσι­

κά» αναφέρει : «Στην εποχή των φιλοσόφων (του Λεύκιππου και του Δημόκριτου) και πριν από αυ­τούς οι λεγόμενοι Πυθαγόρειοι, ασχολήθηκαν με την μελέτη των Μαθηματικών και υπήρξαν οι πρώτοι που ανέπτυξαν αυτή την επιστήμη στο βαθμό που, έχοντας ανατραφεί με αυτά, πίστευαν ότι οι αρχές των Μαθηματικών οφείλουν να απο­τελούν αρχές όλων των υπαρκτών πραγμάτων». Είναι φανερό ότι η θεωρία των Αριθμών οφείλεται

4 Schrodinger, Ε., Η φύση και οι Έλληνες, Π. Τραυλός, Αθήνα 2000, σελ. 39 5 Heath, Th.L., Ιστορία των ελληνικών Μαθηματικών, τομ. 1 :Από τον Θαλή στον Ευκλείδη, Κέντρο Έρευνας Επιστήμης και Εκπαίδευσης, Αθήνα 2001, σελ. 90-91

ΕΥΚΛΕΙΔΗΣ Β ' 83 τ.3/1

Page 4: Ευκλειδης Β 83

----------------- Οι Πυθαγόρειοι -----------------

στη Σχολή του Πυθαγόρα και γνωρίζουμε από τα «Μετά τα φυσικά» του Αριστοτέλη ότι «φαίνεται πως απέδιδε ύψιστη σημασία στη μελέτη της αριθ­μητικής» 6.

Η κύρια γραμμή της σκέψης των Πυθαγορείων είναι η ρήση ότι «τα όντα είναι οι αριθμοί>/. Κι αυτό διότι στην κοσμολογία τους, και διαφέροντας ουσιαστικά από τους Ίωνες φιλοσόφους, πρόσθε­σαν τις έννοιες της τάξης, της αναλογίας και του μέτρου, τονίζοντας τις «ποσοτικές» διαφορές. Αυ­τό μας δίνει το δικαίωμα να τους ονομάσουμε φι­λοσόφους της μορφής σε αντίθεση με τους Ίωνες που ήταν φιλόσοφοι της ύλη ξ

Πίστευαν, λοιπόν, ότι κάθε ον είναι ό,τι είναι, όχι εξαιτίας των υλικών στοιχείων, που είναι κοινά σε όλα τα όντα, αλλά με βάση την αναλογία κατά την οποία τα στοιχεία αυτά έχουν αναμειχθεί. Και εφό­σον το στοιχείο της αναλογίας κάνει να διαφέρει η μία τάξη των όντων από την άλλη, γι αυτό, υποστή­ριξαν ότι ο νόμος της δομής κάθε όντος ήταν το ου­σιαστικό που έπρεπε να ανακαλύψουμε αν επιθυ­μούσαμε να καταλάβουμε τι ήταν το κάθε ον.

Στηρίζεται, λοιπόν, η φιλοσοφία τους, σε μία οντολογία, ο άξονας της οποίας είναι φτιαγμένος με υλικά που ένας σύγχρονος μαθηματικός θα α­ναγνώριζε σαν βασικά αντικείμενα της καθημερι­νής του δραστηριότητας. Γι αυτό οι Πυθαγόρειοι κατέχουν, ανάμεσα στους Προσωκρατικούς, μία ιδιαίτερα σημαίνουσα θέση, σε σχέση με αυτό που σήμερα θα λέγαμε Φιλοσοφία των Μαθηματικών9•

Σύμφωνα με την Πυθαγόρεια κοσμολογία, το σύμπαν δημιουργήθηκε από τον αριθμό (Ι), μετά από διαίρεσή του που πραγματοποιήθηκε από ει­σπνοή απείρου. Το άπειρο εισβάλλει στο αδιαφο­ροποίητα «είναι» και το διασπά δημιουργώντας από τη μονάδα τη δυάδα. Ας δούμε όμως, πιο συ­γκεκριμένα, τι ακριβώς σήμαινε για τους Πυθαγο­ρείους το δίπολο πέρας - άπειρο . Ο κόσμος, κυρι­ολεκτικά δημιουργείται από το δίπολο αυτό, με τον εξής τρόπο : Αρχικά υπήρχε το δίπολο. Το «εί-

6 Kahn , C.H., Ο Πυθαγόρας και οι Πυθαγόρειοι, Ενάλιος 2001, σελ. 47-67 7 Guthrie, W.K.C., ό.π., σελ. 47 8 Guthrie, W.K.C., ό.π., σελ. 46 9 Αναπολιτάνος, Δ.Α. , Εισαγωγή στη φιλοσοφία των Μαθημα­τικών, Νεφέλη, Αθήνα 1985, σελ. 11

ναι» ήταν διαφοροποιημένο, ενιαίο, πεπερασμένο. Αυτό το «είναι» ήταν για τους Πυθαγορείους ταυ­τόσημο με τον αριθμό ένα (1 )10•

Αναφέρουμε ότι οι πρώτοι αριθμοί με τους ο­ποίους ερχόμαστε σε επαφή, στα πρώτα παιδικά μας χρόνια, είναι οι λεγόμενοι φυσικοί ή θετικοί αριθμοί : Ι ,2,3 ,4, κ.λπ. Αυτοί οι αριθμοί είναι α­φαιρέσεις που προκύπτουν από τη διαδικασία α­ρίθμησης πεπερασμένων συνόλων αντικειμένων11•

Τι είναι όμως αυτό που διαφοροποιεί το «είναι» δεδομένου ότι είναι πεπερασμένο; Αυτό που το ο­ριοθετεί είναι το «μη είναι». Το «μη είναι» στους Πυθαγορείους, και γενικότερα στην αρχαιοελληνι­κή σκέψη, ήταν ισχυρότατα συνδεδεμένο με το κενό. Η οριοθέτηση του «είναι» από το «μη είναι» μπορεί να γίνει αντιληπτή με βάση το μοντέλο : το πεπερασμένο αδιαφοροποίητο ον περιορίζεται από το κενό που εκτείνεται επ' άπειρον. Αυτό, λοιπόν, το άπειρον και οριοθετούν κενό εισβάλλει στην αρχή της Πυθαγόρειας κοσμογονίας και διασπά το πεπερασμένο «είναι», παρεμβαλλόμενο ανάμεσα στα δύο του κομμάτια. Η εισβολή αυτή είναι που χαρακτηρίσαμε «εισπνοή απείρου», ακολουθώντας τους αρχαίους Έλληνες στις ανθρωπομορφικές με­ταφορικές περιγραφές τους. Έτσι δημιουργείται από την μονάδα η δυάδα, από την δυάδα ή τριάδα, από την τριάδα η τετράδα κ.τ.λ. , με τρόπο ώστε το πυθαγόρειο σύμπαν να αποτελεί ένα αντίγραφο αυτού που σήμερα θα ονομάζαμε σύνολο των φυ-

. θ . 12 σικων αρι μων .

Για τον Πυθαγόρα και τους μαθητές του, οι α­ριθμοί αποτελούσαν τα ουσιαστικά στοιχεία του κόσμου. Η Πυθαγόρεια οντολογία ήταν μία οντο­λογία αριθμολογική και οι ιδιότητες των όντων, καθώς και οι σχέσεις τους, δεν θα μπορούσαν να εκφραστούν με άλλους τρόπους, παρά με αυτούς που καθορίζονταν από τη δομή ενός αριθμητικού σύμπαντος. Από το «ένα» (1) δημιουργήθηκε με φυσική διαίρεση ο αριθμός «δύο» (2), που σύμφω­να με τους Πυθαγορείους είναι ταυτόσημος με την ευθεία γραμμή, και από το «δύο» ο αριθμός «τρί­α», που είναι στην ουσία το απλούστερο επίπεδο σχήμα, δηλαδή το τρίγωνο, και από τον αριθμό «τρία» ο «τέσσερα» που ταυτίζεται με το τετράε­δρο, το απλούστερο στερεό13 .

Σύμφωνα, λοιπόν, με τους Πυθαγορείους, οι α­ριθμοί είναι αιώνιοι, αθάνατοι, θεϊκοί και αποτε­λούν το κοσμολογικό «είναι» του σύμπαντος. Όλα διέπονται από μαθηματική αρμονία, όπως για παράδειγμα, η μουσική και η ψυχή μας αποτελεί

10 Αναπολιτάνος, Δ.Α. , ό.π., σελ. 14 11 Eνes, Η., Μεγάλες στιγμές των Μαθηματικών (έως το 1650), Τροχαλία, Αθήνα 1989, σελ. 57 12 Αναπολιτάνος, Δ.Α., ό.π., σελ. 15 13 Αναπολιτάνος, Δ.Α., ό.π., σελ. 19

ΕΥΚΛΕΙΔΗΣ Β ' 83 τ.3/2

Page 5: Ευκλειδης Β 83

σπίθα της παγκόσμιας ψυχής. Τα πάντα έχουν aρ­μονικούς λόγους. Και σύμφωνα με την διατύπωση του Φιλολάου «όλα τα πράγματα που μπορούν να γίνουν γνωστά έχουν από έναν αριθμό, γιατί είναι αδύνατον να συλλάβουμε ή να γνωρίσουμε οτιδή­ποτε χωρίς αριθμό>> .

Όλα τα πράγματα δεν διαθέτουν απλώς αριθμούς αλλά είναι αριθμοί. Ο Αριστοτέλης γράφει ότι «αυ­τοί οι στοχαστές φαίνεται πως θεωρούν ότι ο αριθ­μός είναι η αρχή της ύλης των πραγμάτων αλλά ε­πίσης, αυτό που συγκροτεί τις ιδιότητες και τις μό­νιμες καταστάσεις τους». Πράγματι, φαίνεται πως, κατά τον Αριστοτέλη, η θεωρία βασιζόταν αρχικά στην αναλογία μεταξύ των ιδιοτήτων των πραγμά­των και των αριθμών. «Πίστευαν ότι στους αριθ­μούς έβρισκαν περισσότερο από ότι στο πυρ, στη γη ή στο νερό, πολλές ομοιότητες προς τα πράγματα, τα οποία υπήρχαν και εξελίσσονται. Έτσι κάποια συγκεκριμένη ιδιότητα των αριθμών είναι η δι­καιοσύνη, μία άλλη είναι η ψυχή, μία τρίτη είναι η νόηση. Επίσης, στους αριθμούς αναγνώριζαν τις ιδιότητες και τους λόγους των μουσικών κλιμάκων. Οι αριθμοί δεν διακρίνονται από τα αντικείμενα αλλά από τα υπαρκτά αντικείμενα. Ακόμη, και οι αισθητές ουσίες αποτελούνται από αριθμούς. Οι αφηρημένες έννοιες και τα άυλα αντικείμενα είναι επίσης αριθμοί. «Κατασκευάζουν», έτσι, οι Πυθα­γόρειοι, ένα ολόκληρο σύμπαν από αριθμούς, κα­θώς δέχονται ότι «Οι αριθμοί έχουν μέγεθος». Αντι­μετώπιζαν τη μονάδα, η οποία είναι ένα σημείο χω­ρίς θέση (στιγμή «άθετος» ), ως σημείο, και το ση­μείο ως μονάδα (μονάς θέσιν έχουσα) 14 •

Πρέπει να αναφέρουμε ότι ο όρος «αριθμός» σή­μαινε για τους Πυθαγορείους «ακέραιος» (θετικός) αριθμός ή (γενικά) κλάσματα με τους όρους τους α­κέραιους, δηλαδή, κατά την μεταγενέστερη και τη σύγχρονη μαθηματική ορολογία «ρητός» (θετικός) αριθμός. Για την αριθμητική, λοιπόν, των Πυθαγο­ρείων, οι αριθμοί τους ήταν οι ρητοί (θετικοί) αριθ­μοί, δηλαδή πηλίκο δύο ακεραίων p/q, με q διάφορο του μηδενός, το σύστημα των ρητών είναι αρκετό για όλους τους ακεραίους και όλα τα κλάσματα.

Ασυμμετρία και Γεωμετρική θεωρία Οι ρητοί αριθμοί έχουν μία απλή γεωμετρική

παράσταση . Αν σημειώσουμε δύο διαφορετικά σημεία Ο και Ι σε μία οριζόντια γραμμή και πά­ρουμε το τμήμα ΟΙ ως μονάδα μήκους, τότε μπο­ρούμε να θεωρήσουμε ότι τα σημεία Ο και Ι παρι­στάνουν αντίστοιχα τους αριθμούς Ο και 1. Όλοι, λοιπόν, οι ακέραιοι αριθμοί, μπορούν να παραστα­θούν από ένα σύνολο σημείων πάνω στην ευθεία, που απέχουν μεταξύ τους κατά τη μονάδα μήκους. Τα κλάσματα με παρονομαστή q (p/q) παριστάνο-

14 Heath, Th.L., ό.π. σελ. 91-93

νται με τα σημεία που διαιρούν κάθε μοναδιαίο διάστημα ΟΙ σε q ίσα μέρη . Άρα για κάθε ρητό α­ριθμό υπάρχει ένα μοναδικό σημείο Q πάνω στην ευθεία15•

Όλο το οικοδόμημα της Πυθαγόρειας γνώσης είχε χτιστεί πάνω στους φυσικούς και ρητούς α­ριθμούς. Όμως η ανακάλυψη από τους ίδιους ότι δεν υπάρχει ρητός αριθμός που να μπορεί να αντι­στοιχεί σε σημείο Ρ της ευθείας των αριθμών, όταν αυτός ο αριθμός απεικονίζει τη διαγώνιο τετραγώ­νου πλευράς ίσης με τη μονάδα, αναιρεί την ίδια τους την θεωρία. Σύμφωνα με το Πυθαγόρειο θεώ­ρημα η διάμεσος ισούται με «δ ίσον με τετραγωνι­κή ρίζα του ένα εις την δευτέραν συν ένα εις την δευτέραν, ίσον με τετραγωνική ρίζα του δύο» α­ριθμός που δεν είναι ρητός διότι δεν μπορεί να γραφεί σαν πηλίκο δύο ακεραίων.

Η πρόταση αυτή, η «αποκάλυψη», τους βεβαίωσε ότι υπάρχουν μεγέθη ομοειδή μεταξύ τους, που γι αυτά δεν οριζόταν λόγος του ενός προς το άλλο, με­γέθη άλογα, ασύμμετρα μεταξύ τους, χωρίς κοινό μέτρο, χωρίς «κοινή» ομοειδή τους μονάδα μέτρησής τους - άρρητα - που ήταν αδύνατον να εκφραστούν, να ειπωθούν, να ρηθούν το ένα μέσω του άλλου.

Η αντίφαση αυτή επέφερε ένα «λογικό σκάνδαλο και δίλημμα μέγα» στο οικοδόμημα των Μαθημα­τικών, που το θεωρούσαν απρόσβλητο, και γενικότε­ρα στις θέσεις της Πυθαγόρειας Φιλοσοφίας και Κο-

θ ' 16 'Ε λο ' θ ' ' σμο εωριας . πρεπε, ιπον, να επινοη συν νεοι αριθμοί και αφού οι αριθμοί αυτοί δεν μπορούν αν είναι ρητοί ονομάστηκαν άρρητοι αριθμοί17•

Οι Πυθαγόρειοι έκαναν γνωστή την ουσία της συμμετρίας και της aσυμμετρίας και φανέρωσαν στους άλλους τα σχετικά με τις αναλογίες και τις aσυμμετρίες, όπως αναφέρει και ο Ίππασος στο δοξογραφικό απόσπασμα του Ιαμβλίχου 18• Εκείνοι θεώρησαν ότι από τη μονάδα και την αόριστη δυάδα έγιναν οι άλλοι αριθμοί, από τους αριθμούς τα σημεία, από τα σημεία οι γραμμές, από τις

1 5 Eves, Η., ό.π. σελ. 59 16 Heath, Th.L., ό.π. σελ. 153 1 7 Eves, Η., ό.π. σελ. 59 1 8 Ιάμβλιχος, Περί του Πυθαγορικού βίου, Ίππασος D-K 18 απόσπ. 4a, 246-247

ΕΥΚΛΕΙΔΗΣ Β ' 83 τ.3/3

Page 6: Ευκλειδης Β 83

----------------- Οι Πυθαγόρειοι -----------------γραμμές τα επίπεδα σχήματα, από τα επίπεδα σχή­ματα τα στερεά και από αυτά, τέλος, έγιναν τα αι­σθητά σώματα των οποίων τα υλικά στοιχεία είναι τέσσερα, πυρ, ύδωρ, γη και αήρ. Αυτά μεταβάλλο­νται και μετατρέπονται το ένα στο άλλο. Έτσι, από αυτά δημιουργείται και συντίθεται ο κόσμος, έμ­ψυχος, νοητός, σφαιροειδής, με κέντρο του τη γη , η οποία είναι και αυτή σφαιροειδής και κατοικεί­ται σε όλα τα μέρη της, όπως αναφέρει ο Διογένης ο Λαέρτιος19•

Οι Πυθαγόρειοι γνώριζαν διάφορες ιδιότητες των κανονικών πολυγώνων και των κανονικών στερεών. Είχαν δείξει με ποιο τρόπο μπορεί να καλυφθεί το επίπεδο με κανονικά σχήματα, τρίγω­να, τετράγωνα ή εξάγωνα, όπως επίσης, πως μπο­ρεί να καλυφθεί ο χώρος με κύβους20.

Όλο το σύμπαν σύγκειται σε αναλογίες αριθμών οι οποίες αντιστοιχούν σε γεωμετρικά σχήματα. Η αριθμητική σύλληψη τους οδηγεί σε γεωμετρική σύλληψη - σύσταση της πραγματικότητας του σύμπαντος όλου.

Και εφιστούν την προσοχή στο γεγονός ότι η κατασκευή αρχίζει από ένα σημείο, τη μονάδα, τη θεμελιώδη πρώτη αρχή της μονάδας που είναι η ψήφος ή ψηφίδα.

Αν στο σημείο - μονάδα προστεθούν δύο ψήφοι - ψηφίδες, θα έχουμε μία ευθεία γραμμή, αν προ­στεθούν τρεις ψήφοι, ένα επίπεδο σχήμα και αν προστεθούν τέσσερις, ένα στερεό.

Αν κατασκευάσουμε ένα τρίγωνο και προσθέ­σουμε έναν ακέραιο αριθμό ( 1 ψήφο - ψηφίδα) οι τέσσερις αριθμοί δίνουν το σχήμα που οι Πυθαγό­ρειοι αποκαλούν τετρακτύ, τετράδα, καθώς ο α­ριθμός τέσσερα εκπροσωπείται και στις τρεις πλευρές του ισόπλευρου τριγώνου. Με βάση αυτό το σχήμα, στο ποίο λέγεται ότι ορκίζονταν οι Πυ-θ ' ' ' λ ' λλ 21 αγορειοι, παραγονται ο α τα α α .

«Ου μα τον αμετέρα κεφαλά παραδόντα τετρα­κτύν παγάν αενάου φύσεως ριζώματ' έχουσαν»22

19 Διογένης Λαέρτιος, VIII 24-25, Ανώνυμοι Πυθαγόρειοι, Ο­Κ 58Β 1a 20 Eves, Η., ό.π. σελ. 75 21 Kahn, C.H., ό.π. σελ. 5 8 22 «Ορκίζομαι σε αυτόν που παρέδωσε στη σκέψη μας την τε­τρακτύ, την πηγή και την ρίζα της αέναης φύσης»

Πυθαγόρεια Κληρονομία Η επίδραση των πυθαγορείων αρχών στην ύστε­

ρη αρχαιότητα, έως το κατώφλι της σύγχρονης ε­ποχής είναι ανυπολόγιστη . Ο Κικέρων διατύπωσε τον ισχυρισμό ότι ο Νιγίδιος Φίγουλος, ο πρώτος γνήσιος Πυθαγόρειος, επανέφερε στη ζωή τη διδα­σκαλία (disciplina) «εκείνων των διάσημων Πυθα­γορείων που ήκμασαν επί αρκετούς αιώνες στην Νότιο Ιταλία και στην Σικελία» . Οι διάσημοι Πυ­θαγόρειοι περιλαμβάνουν τον Εμπεδοκλή από την Σικελία καθώς και τον Φιλόλαο και τον Αρχύτα από την Νότιο Ιταλία23 .

Ο Εμπεδοκλής έγραψε : «Υπήρξε ένας άντρας, ανάμεσα στους Πυθαγορείους, που υπερτερούσε σε γνώσεις, που κατείχε τα πιο πλούσια αποθέματα διανοητικού πλούτου και που ήταν στον πιο έξοχο βαθμό ο aρωγός των έργων της σοφίας. Γιατί όταν εξαντλούσε όλη τη δύναμη της διανοίας του, εύ­κολα διέβλεπε το καθετί, μέχρι δέκα ή είκοσι αιώνες της ανθρώπινης φυλής»24•

Το οικοδόμημα της επιστήμης του κόσμου επι­νοήθηκε από τον Πυθαγόρα και τους συντρόφους -μαθητές του . Το οικοδόμημα αυτό δεν καταστρά­φηκε ποτέ. Η σημερινή επιστήμη έχει εγκαταστα­θεί στο ισόγειο, στηριγμένη στα θεμέλια του οικο­δομήματος των Πυθαγορείων25•

Ο Αϊνστάιν έγραψε κάποτε ότι ο επιστήμονας είναι πολλά πράγματα αλλά βαmκά θα πρέπει να λογαριά­ζεται ως πλατωνικός ή πυθαγόρειος. Η Πυθαγόρεια κοσμοθεωρία ενσωματώθηκε δημιουργικά στο γνω­στικό corpus των ερχόμενων γενεώΥ6•

Στον 2 1 ο αιώνα, βασιζόμαστε και στις πλατωνι­κές ιδέες και στις πυθαγόρειες αρχές για να εξε­ρευνήσουμε τον άγνωστο χώρο της Γνώσης.

Βιβλιογραφία 1. Αναπολιτάνος, Δ.Α., Εισαγωγή στη φιλοσοφία των

Μαθηματικών, Νεφέλη, Αθήνα 1 985 2. Eνes, Η., Μεγάλες στιγμές των Μαθηματικών (έως

το 1650), Τροχαλία, Αθήνα 1 989 3. Guthrie, W.K.C. , Οι Έλληνες φιλόσοφοι, από τον

Θαλή ως τον Αριστοτέλη, Παπαδήμας 2001 4. Heath, Th.L., Ιστορία των ελληνικών Μαθηματικών,

τομ. Ι : Από τον Θαλή στον Ευκλείδη, Κέντρο Έ­ρευνας Επιστήμης και Εκπαίδευσης, Αθήνα 2001

5. Huson, Η., Πυθαγόρας, εκδ. Δαμιανός, Αθήνα 6. Kahn , C.H., Ο Πυθαγόρας και οι Πυθαγόρειοι, Ε­

νάλιος 2001 7. Πυθαγόρας- Πυθαγόρειοι, άπαντα, 3, Αρχαία Ελλη­

νική Γραμματεία, τόμος 8 1 1 , Κάκτος 1 999 8. Schrodinger, Ε., Η φύση και οι Έλληνες, Π. Τραυ­

λός, Αθήνα 2000

23 Kahn, C.H., ό.π. σελ. 199-200 24 Huson, Η., Πυθαγόρας, Δαμιανός, Αθήνα, σελ. 8 25 Πυθαγόρας - Πυθαγόρειοι, άπαντα, 3, Αρχαία Ελληνική Γραμματεία, τόμος 811, Κάκτος 1999, σελ. 239 26 Αναπολιτάνος, Δ.Α., ό.π., σελ. 27

ΕΥΚΛΕΙΔΗΣ Β ' 83 τ.3/4

Page 7: Ευκλειδης Β 83

Η. Ιερή Γεωμετρί-α '

οινωνια

πολλά έχουν γραφτεί για την ιερή γεωμετρία, _ . πολλά ερμηνεύονται και πολλά λέγονται ακό­

μα, . . . Εμείς θα μείνουμε στην καταγραφή και στην αναφορά τους, όσο πιο απλά γίνεται, με πολύ προ­σοχή, ... Έτσι θα λέγαμε ότι οι βασικοί ιεροί γεω­μετρικοί λόγοι της αρχαιότητας είναι οι αριθμοί:

π,.J2,J3,J5, φ . • π=3,14159 . . .

π= 3 ,Ι4Ι59 26535 89793 23 846 26433 83279 50288 4Ι97Ι 69399 375 1 0 5 8209 74944 59230 78Ι64 06286 20899 86280 34825 342ΙΙ 70679 82Ι48 0865Ι 32823 06647 09384 46095 505 82 23Ι72 53594 08Ι28 48ΙΙΙ 74502 84 1 02 70Ι93 852ΙΙ 05559 64462 29489 54930 38Ι96 44288 1 0975 66593 3446Ι 28475 64823 37867 83Ι65 27Ι20 Ι909Ι 45648 56692 34603 486 1 0 45432 66482 1 3393 60726 0249Ι 4Ι273 72458 70066 063Ι. . . Ο π βρίσκεται σε κάθε κύκλο (Γ = 2π R). Προκύπτει ως εξής: αν η διάμετρος είναι 1, τότε η περιφέρεια είναι 3, 1416 ...

• Η τετραγωνική ρίζα του Δύο: { .fi) =1,414 . . .

• Η τετραγωνική ρίζα του Τρία: { J3) =1 ,732 . . .

• Η τετραγωνική ρίζα του Πέντε: { J5) =2,236 . . .

• φ= Ι ,6Ι803 39887 49894 84820 45868 34365 63 8ΙΙ 77203 09Ι80 . .. Το φ είναι η Χρυσή Τομή.

Υπάρχουν άρρητοι αριθμοί. Ο π καταγράφτηκε σε πρώτη φάση σε Ι500 δεκαδικές θέσεις και συνεχί­στηκε, χωρίς κανέναν ορατό τύπο, . . . τώρα έχει ξεπεράσει τα 5 τρισεκατομμύρια . . . Ας δούμε ό­μως, καθένα από τους ειδικούς αυτούς αριθμούς και ίσως μπορέσουμε να τους βρούμε, . . . στην Ιε­ρή Γεωμετρία. Όλοι αυτοί, οι πέντε αριθμοί, απο­κτούν νόημα μόνο όταν εξαντληθούν έναντι του Ένα. Είναι όλοι λόγοι του χ : 1 . • Ο π και ο Κύκλος

Ο π βρίσκεται σε κάθε κύκλο. Σε έναν κύκλο με ακτίνα (ΓΔ) να είναι Ι, η διάμετρος (ΑΒ) θα

είναι 2, η περιφέρεια είναι π·2ακτίνα και το εμβα­

δόν π·ακτίνα στο τετράγωνο. Στην Ιερή Γεωμετρία, ο κύκλος συμβολίζει

τις ψυχικές σφαίρες. Ένας κύκλος, εξ αιτίας του π δεν μπορεί να περιγραφτεί με τον ίδιο βαθμό α­κριβείας, όπως ένα φυσικό τετράγωνο. Είναι ένα

Στέλιος Μπακολουκάς

σχήμα που πράττει όλα τα είδη των ψυχικών δραστηριοτήτων.

• Η .J2 και το Τετράγωνο Α�------------�Β

Δ Γ Η πλευρά του τετραγώνου είναι η (ΑΒ) = Ι, η

διαγώνιος του τετραγώνου είναι (ΑΓ) = Τετραγω­νική ρίζα του 2 (Ι ,4Ι4 . . . )

Στην Ιερή Γεωμετρία, το τετράγωνο συμβολί­ζει τον φυσικό κόσμο. Το τετράγωνο καθορίζει την ολότητα. Α ν η πλευρά του είναι Ι , τότε η πε­ρίμετρός είναι 4 και η επιφάνειά του Ι.

• Η J3 και η Κύστη Ιχθύος Δύο κύκλοι έχουν κοινή ακτίνα (ΑΒ). Η ακτί­

να ΑΒ είναι Ι. Οι τεμνόμενοι κύκλοι δημιουργούν μια Κύστη Ιχθύος. Ο μικρός άξονας της Κύστης είναι (ΑΒ) ίσος με Ι και ο μεγάλος άξονας είναι ο (ΓΔ) ίσος με τετραγωνική ρίζα του 3 ( 1 ,732 . . . ) .

\ \

\ \

\ \ z Ε Β Η

Πώς αποδεικνύεται; στο δεξιό τρίγωνο (ΕΒΓ),

ΕΒ = .!. ΑΒ ή 0,5. Η ΓΒ είναι επίσης ακτίνα του 2

κύκλου, με κέντρο Β . Έτσι, ΓΒ = ΑΒ = Ι. Η ΓΔ

ΕΥΚΛΕΙΔΗΣ Β' 83 τ. 3/5

Page 8: Ευκλειδης Β 83

------------- Η Ιερή Γεωμετρία και η κοινωνία ------------

είναι κάθετη στην ΑΒ. Ως εκ τούτου, η γωνία ΓΕΒ είναι 90°. Σύμφωνα με το Πυθαγόρειο θεώρημα α2+β2=γ2 τότε ΕΒ2 + ΓΕ2 = ΓΒ2 δηλαδή

(ΑΒ/2)2+ΓΕ2= 1 άρα 0,25+ΓΕ2= 1 τότε ΓΕ = .Jo, 75 .

Και ΓΕ=Ο,8660254038 . . . αλλά η ΓΕ είναι το μισό

της ΓΔ και έτσι ΓΔ=2·0,8660254038 . . . ή ΓΕ = 1, 732050808. . . . Η τιμή της ρίζας του 3 είναι:

J3 = 1,7320508076 . . . άρα ΓΔ= J3 .

Σε πολλούς Γοτθικούς Ναούς οι θόλοι και άλλα σημεία του ναού, είναι κτισμένα με αυτή την μορφή.

e Η .J5 και το Διπλό Τετράγωνο Το Διπλό Τετράγωνο βρίσκεται σε μερικά από

τα πιο γνωστά ιερά μέρη σε όλο τον κόσμο. Η τετραγωνική ρίζα του 5 αναλύεται ως εξής:

0,6 18 . . . + 1+0,618 . . . Το Διπλό Τετράγωνο. Μικρή πλευρά

(ΑΒ=ΓΔ)= 1, Μεγάλη πλευρά (ΑΔ=ΒΓ)=2, Διαγώ-

νιος (ΑΓ=ΒΔ)= ν'S = 2,236 ...

-,---------,Δ

• Ο φ ο Χρυσός Αριθμός Ο Χρυσός Αριθμός φ=1,618 . . . : Η μικρή ανα­

λογία στα δεξιά είναι = 3, η μεγάλη αναλογία είναι = 5, η μικρότερη προς τη μεγαλύτερη είναι όπως η μεγαλύτερη προς το σύνολο, 3 :5 και 5 :8. Αυτός ο λόγος δείχνει ότι είναι ένα τμήμα αυτής της σει­ράς: 1, 2, 3, 5, 8, 13, 21, 34, 55, 89, . . . κλπ. Αυτή είναι η σειρά Α Fibonacci. Ο κάθε αριθμός της σειράς προκύπτει αφού προστεθούν οι δύο προη­γούμενοι αριθμοί, δηλαδή, 5+8=13, 34+55=89, . . . . Ο λόγος δύο διαδοχικών αριθμών της ακολουθίας

τείνει προς την αποκαλούμενη Χρυσή Τομή, ή Χρυσή αναλογία, ή Αριθμός φ = 1.6 18033989 . . . . Ο αντίστροφος της Χρυσής Τομής 1/φ=

0.618033989 . . . , με αποτέλεσμα να ισχύει: 1/φ=φ-1 .

Α --------+Γ----------

8

ΑΓ:ΓΒ

Α -----Γ

προς Γ

------ Β

Γράφουμε ένα τετράγωνο ΑΒΓΔ, όπου ΑΒ= 1. Διαιρώντας τις γραμμές ΑΔ και ΒΓ στη μέση με την κάθετη ΖΕ, οπότε ΒΓ= 1 και ΕΓ=Ο,5. Υπάρχει το Διπλό Τετράγωνο ΕΓ ΔΖ και με τη διάμετρο ΕΔ σαν ακτίνα με κέντρο το Ε, γράφουμε τόξο από το σημείο Δ μέχρι αυτό να τμήσει την ΒΓ στην προέ­κτασή της, στο σημείο Θ. Προεκτείνουμε την πλευρά ΑΖΔ και υψώναμε κατακόρυφο στο ση­μείο Θ, η οποία τέμνει την ΑΖΔ το Η. Έτσι δημι­ουργούμε το φ ορθογώνιο ΑΒΘΗ. Ο λόγος των πλευρών είναι ίσος με 1/φ.

I I

I I

I I I

/ / I

"' "' "'

Α ��---:z---, Η ��-��-��--.

I �-------8 Γ Θ

Το φ ορθογώνιο, που καλείται επίσης Χρυσό Ορθογώνιο, χρησιμοποιήθηκε από τον Da Vinci και άλλους ζωγράφους στην Αναγέννηση.

Ο Χρυσός Αριθμός φ και η τετραγωνική ρίζα του 5 έχουν σχέση. Εκτεινόμενο το τόξο ΘΔ προς το Α θα τμήσει το ευθύγραμμο τμήμα ΘΒ σε ένα σημείο, στο Ι, όπου θα υπάρξει σχέση του φ και της τετραγωνικής ρίζας του 5 . . ..

Το φ δεν εφαρμόζεται στη Γεωμετρία μόνο στο ορθογώνιο αλλά και σε ένα ευθύγραμμο τμή­μα. Έχοντας ένα ευθύγραμμο τμήμα ΑΒ μήκους χ, η Χρυσή Τομή, γίνεται ως ακολούθως: Φτιάχνου­με ένα ορθογώνιο τρίγωνο με κάθετες πλευρές ΑΒ

μήκους χ και ΒΓ μήκους � , με υποτείνουσα την 2

ΑΒ. Με κέντρο το Γ και ακτίνα � γράφουμε κύ-2

κλο, ο οποίος περνά από το σημείο Δ της ΑΓ. Με κέντρο το Α γράφουμε κύκλο με ακτίνα μήκους ΑΔ. Ο κύκλος περνά από το σημείο Ε της ΑΒ [Χρυσή Τομή].

Εκτός από το Χρυσό Ορθογώνιο, υπάρχει και το Χρυσό Τρίγωνο. Έτσι λέγεται κάθε ισοσκελές τρίγωνο του οποίου ο λόγος της μεγάλης πλευράς

ΕΥΚΛΕΙΔΗΣ Β' 83 τ. 3/6

Page 9: Ευκλειδης Β 83

ΔΡΑΣΕΙΣ ΓΕΓΟΝΟΤΑ

Εκδήλtοοη γιο 1' ε(Ιι)μ&τρiα Σημαντική εκδήλωση, για τη I Ί:ωμι:τρίιι., με αρκε­

τό κόσμο και με κεντρικούς ομιλητές τον ι\ημiιτρη Χ ι ι ιστι ι διιt'Jλιιυ κ ω Λ ντιίινη Μ ι;λιί. διοργάνωσε το Ευ­γενίδειο Ίδρυμα με το περιοδικό «φ>>.

Αξιόλογη παρουσίαση του έργου του Δημήτρη Χρι­στοδούλου έγινε στο Ζάππειο Μέγαρο από τους αποφοί­τους του ΕΤΗ Zurich της Ελβετίας και της Ελλιινικής ΜαΟημu.τική; Εταψι:ίας, μέσα σε εγκάρδια ατμόσφαιρα, με παρευρισκόμενους αρκετούς σημαντικούς Έλληνες Μαθηματικούς, στις 16 Φι:βριηιιφίιιυ 2012.

Ομιλία του Προέδρου της Ε.Μ.Ε. Κυρίες και κύριοι, αγαπητοί συνάδελφοι Αποτελεί για μένα ιδιαίτερη τιμή και χαρά, εκπρο­

σωπώντας την Ελληνική Μαθηματική Εταιρεία, αλλά και το Μαθηματικό τμήμα του Πανεπιστημίου Αθηνών να κάνω, μια αναφορά στην επιστημονική βαρt'ιτητα

και σημασία του έργου, 1 του καθηγητή κ. Δημήτρη Χρι­στοδούλου. Στην διεθνή Επιστημονική σκηνή, ο Καθη­γητής Δημήτρης Χριστοδούλου2 θεωρείται σήμερα από τους πλέον διακεκριμένους Μαθηματικούς με έργο που έχει τεράστια απήχηση και στη Φυσική.

Το επιστημονικό του έργο, κινείται γύρω από τη μελέτη των Θεμελιωδών Εξισώσεων της Φυσικής (Η­λεκτρομαγνητισμός, Βαρύτητα, Ασυμπίεστα Ρευστά κ.λ.π. αλλά και άλλες περιοχές).

Α ν και οι εξισώσεις αυτές είναι γνωστές εδώ και πολλά χρόνια, εν τούτοις η ύπαρξη, οι ιδιότητες και η συμπεριφορά των λύσεων είναι ένα αντικείμενο που ακόμη ανακαλύπτεται. Η φύση των εξισώσεων αυτών

1 Η ΕΜΕ θα τιμήσει τον Δημήτρη Χριστοδούλου και σε ξε­fωριστή εκδήλωση, μέσα στο 2012.

Στο επόμενο τεύχος θα υπάρξει αποκλειστική συνέντευξη, στον «Ευκλείδη Β'», από τον Δημήτρη Χριστοδούλου.

ΑΠΡΟΟΠΤΑ

είναι έντονα μη Γραμμικiι και συνήθως πρόκειται για μερικές διαφορικί:ς εξισ<iισι:ις Υπερβολικού τύπου.

Εδώ είναι, που η δουλειά του Δημήτρη Χριστοδού­λου, έρχεται να ρίξει φώς. Αξίζει να σημειώσει κανείς την εργασία του, πάνω στην •:υστάΟι:ια του χώρου

Minkowski, ως λύση των εξισώσεων βαρύτητας αλλά και την πολύ σημαντική πρόσφατη δουλειά του, για την αυτόματη δημιουργία ιδιομορφιιίιν στα ρευστά

αλλά και στο κενό κάτω από ισχυρά βαρυτικά κύματα. (πρόκειται για μελανί;ς οπί;ς που δημιουργούνται χωρίς την ύπαρξη άστρου, δηλαδή στο κενό υπό την επήρεια ισχυρών βαρυτικών κυμάτων) .

Ας σημειωθεί ότι οι εργασίες αυτές, ολοκληρώνο­νται σε χιλιάδες σελίδες, με τεράστιες τεχνικές δυσκο­λίες για τις οποίες επινοεί πρωτότυπες μεΟόδους lJπέρ­

βασης. Χαρακτηριστική είναι η ισχυρή Γεωμετρική συ­νιστώσα που κυριαρχεί στις εργασίες του καθηγητή Χριστοδούλου, παρ ' ότι αρχικά τα προβλήματα αυτού του τύπου, διατυπώνονται αναλυτικά. Είναι αξιοσημεί­ωτο ότι στις διαλέξεις του ο Δημήτρης Χριστοδούλου πάντοτε έχει αφετηρία κάποιο πρόβλημα από τα Αρ­χαία Ελληνικ{ι ΜαΟηματικιi.

Θα ήθελα εδώ να τονίσω ότι ο Καθηγητής Χριστο­δούλου διαθέτει σπάνια αρχαιογνωσία, με υψηλή εξει­δίκευση και στιιzαστικό βάΟιις , θεωρεί ότι ο Αρχαίος Ελληνικός Πολιτισμός στον Επιστημονικό τομέα έκανε τέτοια τομή στην Ιστορία και την εξέλιξη της aνθρωπό­τητας, η οποία δεν επανελήφθη ποτέ, μέχρι σήμερα. Χωρίς λοιπόν να είμαστε υπερβολικοί μπορούμε να πούμε ότι ο Καθηγητής Χριστοδούλου, εκτός από την περίοπτη θέση που κατέχει στη διεθνή Μαθηματική Κοινότητα, συνεχίζει την παράδοση των Αρχαίων Ελ­λήνων Γεωμετρών. Η γενική συνέλευση του Τμήματος Μαθηματικών του ΕΚΠΑ καθώς και το Δ.Σ. της Ε.Μ.Ε. απεφάσισαν ομόφωνα να τον τιμήσουν, τόσο για το τε­λευταίο μεγάλο βραβείο στα Μαθηματικά (Shaw) που είναι ισότιμο με βραβείο Nobel (αφού είναι γνωστό ότι δεν υπάρχει βραβείο No­bcl στα Μαθηματικά), όσο και για το συνολικό πρωτο­ποριακό επιστημονικό του έργο, σε ειδικί:ς επίσημες

εκδηλ<iισ•:ις. Ο κ. Χριστο­δούλου πέρα από διακεκρι­μένος Μαθηματικός και Φυ­σικός είναι και ένας σπου­δαίος Έλληνας που τιμιί. και

αγαπά την πατρίδα του. ι\ημήτρης Χριστοδούλου Τ ο έργο του και η παρουσία του σε μια εποχή πα­

ρακμής και κατί�ιιρευσης αποτελεί πρότυπο και ελπίδα.

Του ευχόμαστε υγεία και δύναμη για μεγαλύτερες διακρί­σεις και ι:υίιδωση όλων των εmστημονικών στόχων του.

Γρηγίφως Καλιιγψίιπουλος Πρόεδρος του Μαθηματικού Τμήματος

Του Πανεπιστημίου Αθηνών και Πρόεδρος της Ε. Μ. Ε.

ΕΥΚΛΕΙΔΗΣ Β ' 83 τ.3/7

Page 10: Ευκλειδης Β 83

------------- Η Ιερή Γεωμετρία και η κοινωνία -------------προς τη μικρή είναι ίσος με φ. Έτσι, κάθε ισοσκε­λές τρίγωνο με κορυφή γωνίας 3 0° είναι Χρυσό Τρίγωνο. Στα κανονικά εγγεγραμμένα πολύγωνα, το Πεντάγωνο (Πεντάγραμμο των Πυθαγορείων) συναντάμε τον αριθμό φ. Δηλαδή, η διχοτόμος κά­θε τριγώνου του πολυγώνου που σχηματίζεται από μία κορυφή και την απέναντι πλευρά του πολυγώ­νου, τέμνει την απέναντι πλευρά κατά Χρυσή Το­μή.

Οι Ιεροί Αριθμοί και οι εφαρμογές τους 1. Ο Παρθενώνας είναι χτισμένος σύμφωνα

με την Ιερή Γεωμετρία. Αν το ύψος του Παρθενώ­να είναι 1 , το πλάτος του είναι φ και το μήκος του

είναι J5. Δηλαδή 1+1,618 . . . +2,236 . . . =3φ.

Έτσι θα μπορούσαμε να πούμε ότι υπάρχουν,

λοιπόν, πέντε ιεροί γεωμετρικοί λόγοι, το π, η J2, η J3 , η J5 και το φ. Η Ιερή Γεωμετρία βρίσκε­ται παντού. Στις αποστάσεις και σχέσεις μεταξύ των σωμάτων του ηλιακού μας συστήματος, στα χέρια μας, στο πρόσωπό μας, στο σώμα μας, στα φυτά, στα ψάρια κ.α.

2. Η εφαρμογή του φ στις πυραμίδες. Τελικά τετραγωνίζεται ο κύκλος; Το τετρά­

γωνο συμβολίζει το φυσικό κόσμο, την υλική κα­τάσταση . Ο κύκλος συμβολίζει τον πνευματικό κόσμο. Όλοι οι μελετητές της Ιερής Γεωμετρίας τείνουν να προσπαθούν τον τετραγωνισμό του κύ­κλου. Μια κλασική και γνωστή κατασκευή σύμ­φωνα με το φ, είναι η Μεγάλη Πυραμίδα στην Αίγυπτο.

Ο τετραγωνισμός του κύκλου γίνεται με ένα ορ-θογώνιο τρίγωνο που σχηματίζεται από το ευθύ­γραμμο τμήμα ΕΖ που συνδέει το μέσον μιας βάσης με την κορυφή της πυραμίδας, την κατακόρυφο ΕΗ από την κορυφή της πυραμίδας προς την βάση και το ευθύγραμμο τμήμα ΕΖ που ενώνει τα δύο πριν. Αυτό είναι το σχήμα της πυραμίδας με το ορθογώνιο τρί­γωνο. Υποθέταμε πως η πλευρά του τετραγώνου της βάσης είναι ίση με 2.

Κατασκευάζουμε τετράγωνο ΕΘΚΔ, έτσι δη­μιουργείται ένα διπλό τετράγωνο, το ΘΚΤΗ. Χρη­σιμοποιώντας το διαβήτη με κέντρο το Η και από-

σταση την ΗΛ, όπου Λ είναι το σημείο τμήσης της ΗΚ με τον προηγούμενο κύκλο κέντρου Ο, γρά­φουμε τόξο προς τα κάτω που τέμνει την ΚΜΤΓ στο σημείο Μ.

Φέρουμε τη διαγώνιο ΗΚ, που τέμνει την ΕΔ στο Ι. Η ΕΔ είναι ίση με 1 , ως εκ τούτου η διάμε-

τρος του κύκλου είναι επίσης 1. Η ΗΚ = J5 .

Β

Γ

Α Η

Αν HK= J5 και ΟΛ=ΟΔ=ΟΕ=Ο,5, η διάμε­τρος αυτού του κύκλου είναι 1 . Έτσι: ΗΛ=0,6 1 8 . . . +1, ή φ = 1 ,6 1 8 . . . .

Θ,-------.

Τώρα γράφουμε την ΗΜ που τέμνει την ΑΕΔ στο Ι . Βάζουμε τον διαβήτη στο Τ και εκτείνουμε ως το Μ . Βάζουμε πάλι στο σημείο Η και σχηματί­ζουμε τον κύκλο που έχει ακτίνα ΗΙ. Το τΜ είναι το ύψος της Μεγάλης Πυραμίδας. Αυτός ο κύκλος αξιοσημείωτα πλησιάζει να έχει την ίδια περιφέ­ρεια όπως η περίμετρος της βάσης ΑΒΓ Δ.

Είναι φανερό ότι αν στο αρχικό ορθογώνιο

τρίγωνο ΕΖΗ, όπου ΕΖ=1, ΖΗ=φ τότε ΕΗ=jφ .

ΕΥΚΛΕΙΔΗΣ Β' 83 τ. 3/8

Page 11: Ευκλειδης Β 83

Μ � �Δ I; βv&ιμστtΚοt ·· .. _ 1.a.yωvtOp!Ot Μαβqμαnκές Ολ"Uμπιά&ς;

Επιτροπή Διαγωνισμών της Ε.Μ.Ε.

72°ς ΠΑΝΕΛΛΗΝΙΟΣ ΜΑΘΗτΙΚΟΣ ΔΙΑΓΩΝΙΣΜΟΣ ΣΤΑ ΜΑΘΗΜΑτΙΚΑ

«0 ΕΥΚΛΕΙΔΗΣ» ΣΑΒΒΑΤΟ, 21ΙΑΝΟΥΑΡΙΟΥ 2012

ΕΝΔΕΙΚΤΙΚΕΣ ΛΥΣΕΙΣ Α' τάξη Λυκείου

Πρόβλημα 1 Να βρείτε το υποσύνολο των πραγματικών αριθμών στο οποίο συναληθεύουν οι ανισώ-

χ2 lxl -1 lxl+x2 x+l x(x+l) (χ+2) 2 σεις: -+-- � και --+ > . 4 3 4 2 4 4

Λύση xz lxl-1 lxl+xz

Έχουμε -+-- � � 3χ2 +4 (lxl - 1 )::;; 3 lxl+3x2 � lxl::;; 4 � -4::;; χ::;; 4. 4 3 4

χ+1 χ(χ+1) (χ+2)2 2

-+ > �2χ+2+χ(χ+1)>(χ+2) �2χ+2+χ2+χ>χ2+4χ+4�-χ>2�χ < -2. 2 4 4

Επομένως, οι δύο ανισώσεις συναληθεύουν στο διάστημα [ -4,-2 ) = {χ ε IR. : -4 ::;; χ < -2} . Πρόβλημα 2 Να προσδιορίσετε τις λύσεις της εξίσωσης

χ + χ2 [ ( 1 + αχ Υ -(α + χ Υ] αb . = ' 1-χ2 1-α2 (α-bΥ για τις διάφορες τιμές των πραγματικών αριθμών α,b με αb( α-b }( 1-α2) ::ι:. Ο.

Λύση Για να ορίζονται οι δεδομένες παραστάσεις πρέπει να ισχύουν:

1-χ2 ::ι:.Ο, 1-α2 ::ι:.Ο(υπόθεση) και α:;t:b(υπόθεση)�χ:;t:±1.

Για χ* ±1 , η δεδομένη εξίσωση είναι ισοδύναμη με την εξίσωση:

χ ( 1 + αz xz - αz - xz ) αb χ ( 1 -αz ) ( 1 - xz ) αb - · = � - · =----;:-1 - χ ( 1-α2) (α - b)2 1 - χ ( 1-α2) (α-b)2

� x(l +x)= ab 2 � (a - b)2x2+(a - b)2x - ab= O

(α- b) Επειδή είναι α :;t: b η τελευταία εξίσωση είναι δευτέρου βαθμού και έχει διακρίνουσα

Δ = (α- b )4 + 4αb (α- b )2 = (α- b )2 [ (α- b )2 + 4αb J = (α- b )2 (α+ b )2 = ( α2 - b2 )2 � Ο .

Άρα η εξίσωση έχει τις ρίζες (ίσες, αν α= -b ) :

- (α-b)2 +(α2 -b2 ) 2αb-2b2 2b (α-b) b

χ1 = = = = -- και

2 (α - b)2 2 (α -b )2

2 (α-b)2 α-b

ΕΥΚΛΕΙΔΗΣ Β ' 83 τ. 3/9

Page 12: Ευκλειδης Β 83

-------- Μαθηματικοί Διαγωνισμοί - Μαθηματικές Ολυμπιάδες ---------(α - b )2 - (α2 - b2 ) 2αb - 2α2 -2α(α - b) -α

χ = = = = 2 2(α - b )2 2(α - b)2 2(α - b)2 α - b

Παρατηρούμε ότι: b b -- = 1 <=> b = b - α <=> α = Ο και -- = -1 <=> b = -b + α <=> α = 2b,

b - α b - α -α -α -- = 1 <=> -α = α - b <=> 2α = b και -- = -1 <=> -α = b - α <=> b = Ο.

α - b α - b Επομένως, για τιμές των παραμέτρων α, b που ικανοποιούν τις υποθέσεις α -:1:- Ο , b -:1:- Ο, α -:1:- b

και α -:1:- ± 1 , έχουμε :

Αν (α - 2b )( 2α - b) -:1:- Ο , η εξίσωση έχει δύο ρίζες (ίσες με _!_ , αν α = -b ) : 2

b -α χ1 = -- και χ2 = -- .

α - b α - b Α ν α = 2b , τότε η εξίσωση έχει μόνο τη ρίζα χ2 = -2 .

Α b ' ξ' ' ' 'ζ 2 ν α = - , τοτε η ε ισωση εχει μονο τη ρι α χ? = - . 2 -

Δίνεται ορθογώνιο τρίγωνο ΑΒΓ με Β = 90° και Α< 45° . Θεωρούμε τα μέσα Δ και Ε των πλευρών ΒΓ και ΑΓ, αντίστοιχα, και σημείο Μ -:1:- Α στο ευθύγραμμο τμήμα ΑΕ. Αν η μεσο­κάθετη του ευθύγραμμου τμήματος ΒΜ τέμνει την ευθεία ΔΕ στο Ζ και την ευθεία ΑΓ στο

Θ, να αποδείξετε ότι: ΒΜΖ = Α . Η ευθεία ΒΖ διχοτομεί τη γωνία ΘΒΕ.

Α

..................... ---

Β Δ Σχήμα 1

Γ

Επειδή το Ζ ανήκει στη μεσοκάθετη του ΒΜ θα

είναι Ζ Β = ΖΜ και ΒΜΖ = ΜΒ Ζ = ω . Επειδή είναι ΔΕ 11 ΑΒ και ΑΒ l_ ΒΓ έπεται ότι

ΔΕ l_ ΒΓ , δηλαδή η ευθεία ΔΕ είναι μεσοκάθετη της

πλευράς ΒΓ . Αφού Ζ ε ΒΓ θα είναι ΖΒ = ΖΓ και

ΖΒΓ = Zf'B = φ . Επειδή ΜΖ = ΒΖ =Γ Ζ θα είναι και

zMr =ztM =θ . Από το τρίγωνο ΒΜΓ, λόγω των προη­

γούμενων ισοτήτων, έχουμε ΜΒΓ +BfM + rMB =180° =>

2ω + 2φ + 2θ = 1 80° => ω + φ + θ = 90° . ( 1 ) Από το ορθογώνιο τρίγωνο ΑΒΓ λαμβάνουμε

φ + θ =f' = 90° -Α. (2) Από τις σχέσεις ( 1 ) και (2) με αφαίρεση κατά μέλη

λαμβάνουμε: ΒΜ Ζ = ω = Α . Διαφορετικά, θα μπορούσαμε να εργαστούμε θεωρώντας το μέσον Κ της ΒΜ . Αν η ΔΚ

τέμνει την πλευρά ΑΒ στο σημείο Ν, τότε η ΔΝ είναι παράλληλη προς την ΑΓ και το τετρά-

πλευρο ΑΕΔΝ είναι παραλληλόγραμμο, οπότε θα έχουμε: ΝΔΕ =Α . (3)

Επίσης, από το εγγράψιμο τετράπλευρο ΒΔ ΖΚ (έχει Κ =Δ =900) έχουμε: ΝΔΕ =κΔΕ =zΒ Κ(4) Από το ισοσκελές τρίγωνο Ζ ΒΜ ( Ζ Β = ΖΜ ) έχουμε: ΖΒ Κ = ΒΜΖ . (5)

Από τις σχέσεις (3), (4) και (5) έπεται ότι: ΒΜΖ =Α Επειδή το σημείο Θ ανήκει στη μεσοκάθετη του ΒΜ η ΘΖ είναι διχοτόμος της γωνίας

ΒΘΕ . Επίσης, επειδή η ΒΕ είναι διάμεσος του ορθογώνιου τριγώνου ΑΒΓ προς την υποτεί-

ΕΥΚΛΕΙΔΗΣ Β ' 83 τ. 3/10

Page 13: Ευκλειδης Β 83

--------- Μαθηματικοί Διαγωνισμοί - Μαθηματικές Ολυμπιάδες -------­νουσα, θα είναι ΒΕ = ΑΓ = ΕΓ, οπότε το τρίγωνο ΒΕΓ είναι ισοσκελές με την ΕΔ ύψος και δι-

2 χοτόμο της γωνίας ΒΕ Γ , άρα και της γωνίας ΒΕ Θ . Επομένως στο τρίγωνο ΒΘΕ τοΖ είναι το

σημείο τομής των διχοτόμων του, οπότε και η ΒΖ διχοτομεί τη γωνία Θ Β Ε . 4

Αν υπάρχουν ακέραιοι χ,y,α που επαληθεύουν την εξίσωση JX2 +(/ -a2)x+ y(y-a)2 =0, να αποδείξετε ότι ο αριθμός xy είναι τέλειο τετράγωνο ρητού αριθμού.

Έστω ότι οι ακέραιοι χ,y,α επαληθεύουν την εξίσωση : yx2 + (y2- α2)x + y(y- α)2=0. Μετά τις πράξεις και αναδιάταξη των όρων η εξίσωση, ως προς άγνωστο το α, γράφεται:

(y-χ)α2 - 2/α+ y( χ2 +xy+ y2) =Ο. Σύμφωνα με την υπόθεση, η εξίσωση αυτή με άγνωστο το α έχει ακέραια λύση, αλλά και α­

κέραιους συντελεστές. Επομένως, η διακρίνουσα της είναι τέλειο τετράγωνο ακεραίου, δηλαδή

υπάρχει κ Ε Ζ τέτοιο, ώστε Δ=4/ -4y(y-x)(x2 +λ)!+/) =4y[/ - (/ -x3)]=4yx3 =λJ!(2χ)2 =Κ.

Από την τελευταία ισότητα προκύπτει ότι: xy = (.!!.._) 2, όπου ο αριθμός .!!.._ είναι ρητός. 2χ 2χ

Να προσδιορίσετε τις τιμές της παραμέτρου α* Ο για τις οποίες η εξίσωση ι ι 2a + 6 , δ , , 'ζ δ , 4

2a +αχ 2χ _ χ2 =

χ3 _ 4χ , εχει υο πραγματικες ρ ι ες με ιαφορα .

Μετά τις παραγοντοποιήσεις των όρων των κλασμάτων η εξίσωση γράφεται: 1 + 1 = 2(α+3) (1)

α(χ+2) χ(χ- 2) χ(χ- 2)(χ + 2) ·

Πρέπει να ισχύουν χ * Ο, ±2 , δηλαδή η εξίσωση θα λυθεί στο σύνολο IR- {-2, Ο, 2}. Η εξίσωση ( 1) στο σύνολο IR- {-2, Ο, 2} είναι ισοδύναμη τελικά με την εξίσωση

χ2 + ( α- 2) χ- ( 2α2 + 4α) =Ο , η οποία έχει διακρίνουσα Δ = ( 3α + 2 )2 και ρίζες χ1 =α + 2 και

χ2 = -2α. Επειδή α+ 2 Ε {-2,0,2} �α Ε {-4,-2,0} και -2α Ε {-2,0,2} �α Ε {1,0,-1} και αφού

από την υπόθεση είναι α * Ο , συμπεραίνουμε ότι η εξίσωση ( 1) έχει δύο ρίζες δεκτές, τις

χ, = α-2 και χ2 =-2α, όταν είναι α-:!;-1, +1,-2,-4. Επειδή είναι lα+2-{-2a)l=4<=:

� l3α + 21 = 4 � 3α + 2 = 4 ή 3α + 2 = -4 �α=� ή α= -2, η τιμή του α που ζητάμε είναι η α = � . 3 3

f1J μό}Ω.ημια 2 Α ν y ακέραιος και χ Ε R , να προσδιορίσετε όλα τα ζευγάρια (χ, y) που είναι λύσεις του

συστήματος {1 +Υ -ix2- 3χ +ι Ι> 0} .

(Σ) y-2+lx-21 < Ο

Να παραστήσετε γραφικά στο Καρτεσιανό επίπεδο Oxy, το σύνολο των σημείων

Μ (χ, y) , όπου (χ, y) λύση του συστήματος (Σ).

ΕΥΚΛΕΙΔΗΣ Β' 83 τ. 3/1 1

Page 14: Ευκλειδης Β 83

--------- Μαθηματικοί Διαγωνισμοί - Μαθηματικές Ολυμπιάδες --------ΛίJση

Έχουμε { 1 + Υ - Ιχ2 - 3χ + 1 1 > ο}

� { 1 + Υ > Ιχ2 - 3χ + ψ : : ο}

' y - 2 + lx - 21 < Ο y - 2 < - Ι χ - 21 s Ο

Από τις δύο τελευταίες εξισώσεις προκύπτει ότι: 1 + y > Ο και y - 2 < Ο � -1 < y < 2 . Επομένως οι δυνατές τιμές του y είναι y = Ο ή y = 1 .

• Για y = Ο, το σύστημα γίνεται: { Ιχ2 - 3χ + 1 1 < 1} � {- 1 < χ2 - 3χ + 1 < 1}

� {χ2 - 3χ + 2 > ο και χ2 - 3χ < ο}

lx - 21 < 2 - 2 < χ - 2 < 2 - 2 < χ - 2 < 2 {(χ < 1 ή χ > 2) και Ο < χ < 3} , � � ο < χ < 1 η 2 < χ < 3 . 0 < χ < 4

• Για y = 1, το σύστημα γίνεται: { Ιχ2 - 3χ + 1 1 < 2} � {-2 < χ2 - 3χ + 1 < 2} �

{χ2 - 3χ + 3 > ο και χ2 - 3χ - 1 < ο} lx - 21 < 1 - 1 < χ - 2 < 1 1 < χ < 3 { 3 - JU 3 + J13 } --- < χ < ---� 2 2 � 1 < χ < 3 .

1 < χ < 3 Για τη γεωμετ ική αναπαράσταση του συνόλου των λύσεων του συστήματος έχουμε :

3

2

o t--- ��--�- --ο 2 3 - 1

-2

Π ρόβλημα 3

4 5 6 7 8 > 9

Σχήμα 2

Δίνεται οξυγώνιο σκαληνό τρίγωνο ΑΒΓ με ΑΒ < ΑΓ , εγγεγραμμένο σε κύκλο c(O , R) . Η διχοτόμος της γωνίας Α τέμνει τον κύκλο c(O , R) στο σημείο Μ. Ο κύκλος c1 (Μ,ΑΜ) τέ­

μνει την προέκταση της ΑΓ στο σημείο Δ. Να αποδείξετε ότι ΓΔ = ΑΒ . ΛίJση ( 1 ο � τρόπος) Έστω Ε το δεύτερο κοινό σημείο των περιφε­

ρειών ( c) και ( c1 ) . Τότε η ΑΕ είναι η κοινή χορ­

δή των δύο κύκλων, άρα η ΟΜ είναι μεσοκάθετη της ΑΕ . Το Μ είναι το μέσο του τόξου ΒΓ (διότι

η ΑΜ είναι διχοτόμος της γωνίας Α). Άρα η ΟΜ είναι μεσοκάθετη και της ΒΓ. Επειδή οι χορδές ΒΓ και ΑΕ έχουν την ΟΜ κοινή μεσοκάθετη,

Δ συμπεραίνουμε ότι το τετράπλευρο ΑΒΓΕ είναι Μ ισοσκελές τραπέζιο, οπότε : ΑΒ = ΕΓ . ( 1 )

Σχήμα 3 Το τρίγωνο ΜΑΔ είναι ισοσκελές, αφού ΜΑ = ΜΔ ως ακτίνες του κύκλου ( c1 ) • Άρα είναι

ΕΥΚΛΕΙΔΗΣ Β ' 83 τ. 3/12

Page 15: Ευκλειδης Β 83

--------- Μαθηματικοί Διαγωνισμοί - Μαθηματικές Ολυμπιάδες � � Α ι = Δ ι = Α

. Ισχύει επίσης Α ι = Ε ι = Α (εγγεγραμμένες στον κύκλο ( c) και βαίνουν στο τό-

2 2

ξο Mr ) . Από τις τελευταίες ισότητες γωνιών συμπεραίνουμε ότι Ε ι = Δ ι = Α και σε συνδυα-

2 σμό με την ισότητα ΜΔΕ = ΜΕ Δ (που προκύπτει από το ισοσκελές τρίγωνο ΜΔΕ ), καταλή­

γουμε στην ισότητα των γωνιών Γ ΔΕ = ΓΕ Δ και στην ισότητα των ευθυγράμμων τμημάτων: ΕΓ = ΔΓ. (2) Από τις σχέσεις (1) και (2) έχουμε το ζητούμενο.

zος Τρόπος Το τρίγωνο ΑΜΔ είναι ισοσκελές ( ΜΑ = ΜΔ ακτίνες του κύκλου ( c1 ) ) . Η ΑΜ είναι διχο-

τόμος της γωνίας Α , οπότε έχουμε : (3)

Από το ισοσκελές τρίγωνο ΑΜΔ ' έχουμε: Μι + ω = ΑΜΔ = 1 80° -Α ι - Δι = 1 80" -Α

� Μ ι + ω = 1 80° -Α <=> Μ ι = 1 80" -Α- ω ο ( 4) Επίσης, ισχύουν οι ισότητες γωνιών: Β = ω

(είναι εγγεγραμμένες στο κύκλο ( c) και βαίνουν στο ίδιο τόξο), Μ2 = Γ (είναι εγγεγραμμένες

στο κύκλο ( c) και βαίνουν στο ίδιο τόξο). Άρα έχουμε Μι = 1 80" -Α- Β = Γ = Μ2 • (5)

Μ

Σχήμα 4

Από τις ισότητες: Μι = Μ2 , ΜΒ = ΜΓ (διότι το .---._ Μ είναι μέσο του τόξου ΒΓ) και ΜΑ = ΜΔ (διότι ΜΑ, ΜΔ ακτίνες του κύκλου ( c1 ) ), συμπεραίνουμε

ότι τα τρίγωνα ΜΑΒ και ΜΔΓ (*) είναι ίσα, οπότε ΓΔ = ΑΒ . (*) Η ισότητα των τριγώνων, μπορεί να αποδειχθεί και με άλλους τρόπους.

Π αρατηρήσης: Διαφορετικά, θα μπορούσαμε να απο­δείξουμε ότι τα σημεία Μ, Γ και το μέσο της ΔΕ είναι

συνευθειακά. Ο κύκλος ( c1 ) τέμνει και τη προέκταση της

ΑΒ . Αν ονομάσουμε Λ το σημείο τομής, τότε θα ισχύει ΒΛ = ΑΓ . Έτσι δημιουργείτε το ισοσκελές τρίγωνο ΑΔΛ με ΑΔ = ΑΛ = ΑΒ + ΑΓ και στη συνέχεια, μπορούμε να αποδείξουμε ότι ΑΜ l_ ΔΛ .

Π ρόβλη μα 4 Βρείτε όλες τις ρητές τιμές του χ για τις οποίες είναι ρητός ο αριθμός .J χ2 + αχ + b , όπου

a,b ρητοί τέτοιοι ώστε a2 < 4b .

ΛίJση Επειδή από υπόθεση α2 - 4b < Ο , έπεται ότι χ2 +αχ+ b > Ο , για κάθε χ ε IR . Α ν υποθέσουμε

ότι οι αριθμοί χ , � χ2 +αχ+ b = y είναι και οι δύο ρητοί, τότε και η διαφορά τους y - χ = r θα εί-

, 'Ε , Ι 2 b Ι 2 b 2 b 2 2 2 r2 -b ναι ρητος. τσι εχουμε ν χ +αχ+ -χ= r<=>νχ +αχ+ =x+r�x +αχ+ =χ + l'X+r �χ=--,

' α Ο ' δ ' ' α

Α ' ' εφοσον r -:f::- - και χ + r ?: η ισο υναμα, εφοσον r > - . ντιστροφα, αν ειναι 2 2

r2 - b , , α , , χ = --, οπου r ρητος με r > - , τοτε εχουμε:

α - 2r 2

α-2r

2 b [ r2 - b J2 r2 - b b r4 - 2αr3 + α2r2 + 2br2 - 2αbr + b2 (r2 - αr + b )2 χ + αχ + == -- + α-- + == 2 =

α - 2r α - 2r (α - 2r) (α - 2r)2

ΕΥΚΛΕΙΔΗΣ Β ' 83 τ. 3/13

Page 16: Ευκλειδης Β 83

--------- Μαθηματικοί Διαγωνισμοί - Μαθηματικές Ολυμπιάδες -------­οπότε, αφού από υπόθεση α2 - 4b < Ο , θα είναι y = .J χ2 +αχ+ b = r

2

1- ar +I

b = r2 - ar + b

, r > !!._, a- 2r 2r- a 2

δηλαδή ο y είναι ρητός.

Γ' τάξη Λ υ κ ε ίου Π ρόβλη μα 1 Να βρεθεί η αριθμητική πρόοδος αν , v = 1, 2,3, . . . που έχει πρώτο όρο α1 = α ;;�; Ο , διαφορά

ω ;;�; Ο και είναι τέτοια ώστε ο λόγος του αθροίσματος α1 + · · · + αν των v πρώτων όρων της

προς το άθροισμα αν+ ι + · · · + α3ν των επόμενων 2v το πλήθος όρων της είναι σταθερός, δη­

λαδή ανεξάρτητος του v . ΛίJση

, , θ δ Σv α, + . . ·α ξ Απο την υπο εση ίνεται ότι: = v = c ( ανε άρτητο του v) .

Σ 3v - Σv αv+ι + · · ·α3v (1)

[ 2α + ( v - 1) ω J · v Επειδή είναι Σv = α , + · · · + αv = και

2 [ 2α + ( 3v - 1) ω J · 3v [ 2α + ( v - 1) ω J · v [ 4α + ( 8v- 2) ω J · v Σ - Σ = - = =---------='--3v v 2 2 2

'

2α +(v- 1)ω η σχέση (1) γίνεται

( ) = c <=> ( 8cω- ω) v + 4αc- 2α - 2cω + ω = Ο

4α + 8v- 2 ω <=> (8c -1) ωv + (2c -1)(2α- ω) = Ο, για κάθε v = 1 , 2, 3, . . . (2)

<=> ( 8c -1) ω = Ο και ( 2c -1) ( 2α- ω)= Ο <=> 8c - 1 = Ο και 2c -1 ή 2α - ω = Ο <=> c =_!_ ,ω= 2α , 8

αφού ω ;;�; Ο. Επομένως η αριθμητική πρόοδος που ζητάμε είναι η : α, 3α , 5α , . . . . ,( 2v - 1)α, . . . . Διαφορετικά. στην εξίσωση ( 8c- 1)ωv + (2c - 1 )(2α- ω)= Ο , που ισχύει για κάθε

v = 1 , 2,3, . . . μπορούμε να θεωρήσουμε v = 1 και v = 2 και να αφαιρέσουμε κατά μέλη τις ισό­

τητες που προκύπτουν, οπότε λαμβάνουμε ( 8c- l)ω= 0 και από αυτή ( 2c-1)(2α- ω)= 0 , ο-

' ' , λ , { ( 8c - 1 ) ω = Ο } { c = _!_ } ποτε εχουμε πα ι το συστημα: <:::::> 8 , αφού ω =t- Ο .

(2c - 1 ) ( 2a - ω) = O ω = 2α Εύκολα επαληθεύουμε ότι οι τιμές που βρήκαμε ικανοποιούν την εξίσωση (2) για κάθε

v ε Ν* , οπότε η αριθμητική πρόοδος που ζητάμε είναι η: α,3α, 5α, . . . . ,( 2v - 1)α, . . . . Π ρόβλη μα 2

Ν λ , , θ , , _ _2 Sz4 α υσετε στους πραγματικους αρι μους το συ στη μα:

χ = 16+ z4 ,

ΛίJση 2 8z4 2 8z 2 2 , , 8z 2

Παρατηρούμε ότι χ = 4 = z · 2 ::::;; z , αφου ισχυει : 2 ::::;; 1 , και ομοίως 16 + z 42 +(z 2) 42 + (z 2)

λ β , , 2 < 2 2 < 2 Ε , , 2 2 2 Τ ' , , αμ ανουμε οτι: z _ y και y _χ . πομενως, εχουμε: χ = y = z . οτε απο την πρωτη ε-

ξίσωση λαμβάνουμε:

8χ4 2 χ2 =--4 <=:>χ2 ( χ4 - 8χ2 + 16) =0<=>χ2(χ2 -4) =Ο<=>χ=Ο ήχ = -2ή χ = 2 (όλες με πολλαπλότητα 2). 16+χ

ΕΥΚΛΕΙΔΗΣ Β ' 83 τ. 3/14

Page 17: Ευκλειδης Β 83

--------- Μαθηματικοί Διαγωνισμοί - Μαθηματικές Ολυμπιάδες

Για χ = Ο , προκύπτει η λύση ( Ο, Ο, Ο) . Για χ = 2 , προκύπτουν οι λύσεις: ( 2, 2, 2 ) , ( 2, -2, 2 ) , ( 2, 2, -2) και ( 2, -2, -2) .

Για χ = -2 , προκύπτουν οι λύσεις: ( -2, 2, 2 ) , ( -2, -2, 2 ) , ( -2, 2, -2 ) και ( -2, -2, -2) .

Δίνεται τρίγωνο ΑΒΓ εγγεγραμμένο σε κύκλο c(O, R) . Τα ύψη του ΑΔ,ΒΕ,ΓΖ τέμνουν

τον περιγεγραμμένο κύκλο στα σημεία Α1 ,Β 1 ,Γ 1 αντίστοιχα. Α ν Az ,B z ,Γ z είναι τα μέσα

των ευθυγράμμων τμημάτων ΟΔ, ΟΕ, ΟΖ αντίστοιχα, να αποδείξετε ότι οι ευθείες

A1Az , B1Bz , Γ1Γz περνάνε από το ίδιο σημείο.

ι'μiηω�} Παρατηρούμε ότι το σημείο Α, είναι συμμετρικό

του ορθοκέντρου Η ως προς την πλευρά ΒΓ. Πράγμα­τι, αν θεωρήσουμε το σημείο Η, συμμετρικό του Η ως προς την πλευρά ΒΓ, τότε έχουμε

Β Η, Γ = ΒΗΓ = 1 80° -Α . Άρα το τετράπλευρο

ΑΒΗ1Γ είναι εγγεγραμμένο στον περιγεγραμμένο

κύκλο του τριγώνου ΑΒΓ, οπότε το σημείο Η, συμπί­

πτει με το σημείο Α, . Έστω Κ το aντιδιαμετρικό του σημείου Α1 και

Μ το σημείο τομής της Α1Α2 με την ΗΚ . Τότε στο τρίγωνο Α1ΗΚ έχουμε ότι το σημείο Ο εί­

Β Γ

Σχήμα 5

ναι μέσο της πλευράς Α1Κ και ότι το σημείο Δ είναι μέσο της πλευράς Α,Η . Άρα το τμήμα

ΟΔ ' ' ' λλ λ ' ΗΚ Ε δ ' ' ΟΔ ΗΚ Α Α ' δ ' ειναι ισο και παρα η ο με το τμημα -- . πει η τωρα = -- και η 1 ? ειναι ια-2 2 -

μεσος στο τρίγωνο ΑρΔ , συμπεραίνουμε ότι η Α, Μ είναι διάμεσος του τριγώνου Α,ΗΚ . Έ-στω ότι οι διάμεσες Α, Μ και ΗΟ (του τριγώνου Α,ΗΚ ) τέμνονται στο σημείο G . Τότε θα ι­

σχύει GH = 200 , δηλαδή το σημείο G χωρίζει το τμήμα ΗΟ σε δύο τμήματα με λόγο 2 : Ι . Με ανάλογο τρόπο αποδεικνύουμε ότι και οι Β,Β2 , Γ ,Γ 2 διέρχονται από το σημείο G .

2" � τρόπος { Χρησιμοποιώντας τη πρόταση : "Τα συμμετρικά του

ορθοκέντρου τριγώνου, ως προς τις πλευρές του, βρί­σκονται στο περιγεγραμμένο κύκλο του", που αποδεί­ξαμε στην αρχή της προηγούμενης λύσης, συμπεραί­νουμε ότι το Δ είναι μέσο του Α1Η , το Ε είναι μέσο

του Β1Η και το Δ είναι μέσο του Γ ,Η . Άρα το τρίγωνο Α1Β1Γ 1 είναι ομοιόθετο του (ορθι­

κού) τριγώνου ΔΕΖ στην ομοιοθεσία με κέντρο το ορ­θόκεντρο Η και λόγο 2 , ( ΗΑ1 = 2ΗΔ ) .

Το Α2 είναι μέσο του ΟΔ , το Β2 είναι μέσο του ΟΕ και το Γ2 είναι μέσο του ΟΖ .

ΕΥΚΛΕΙΔΗΣ Β ' 83 τ. 3/1 5

Page 18: Ευκλειδης Β 83

--------- Μαθηματικοί Διαγωνισμοί - Μαθηματικές Ολυμπιάδες ---------Άρα το ορθικό τρίγωνο ΔΕΖ , είναι ομοιόθετο του τριγώνου Α2Β2Γ 2 στην ομοιοθεσία με κέ­

ντρο το Ο και λόγο 2 , ( ΟΔ = 20Α2 ) . , δηλαδή το τρίγωνο Α2Β2Γ 2 είναι ομοιόθετο του τριγώ­νου Α1Β1Γ 1 • Άρα οι ευθείες Α1Α2 , Β1Β2

, Γ1Γ 2 (που συνδέουν τις ομόλογες κορυφές) θα συντρέ­

χουν στο κέντρο της ομοιοθεσίας (έστω Κ ) το οποίο θα βρίσκεται επάνω στην ΟΗ .

Π μόβλημα 4 Βρείτε όλες τις ρητές τιμές του χ για τις οποίες είναι ρητός ο αριθμός -J 4χ2 - αχ + b , ό­

που a,b ρητοί τέτοιοι ώστε a 2 < 1 6b .

Λ �)lί.ϊ1ti ( � ο: i:' �] �;:π:ος) Επειδή από υπόθεση α2 - 1 6b < Ο , έπεται ότι 4 χ2 - αχ + b > Ο , για κάθε χ ε ffi. . Α ν υποθέ-

σουμε ότι οι αριθμοί χ , .J 4χ2 - αχ + b = y είναι και οι δύο ρητοί, τότε και η διαφορά y - 2χ = r θα είναι ρητός.

Έτσι έχουμε

-J 4χ2 - αχ + b - 2χ = r <=> -J 4χ2 - αχ + b = 2χ + r => 4χ2 - αχ + b = 4χ2 + 4rx + r2 => χ = b - r2 ,

α + 4r

' α 2 Ο ' δ ' ' α εφοσον r =1:- -- και χ + r � , η ισο υναμα εφοσον r > - - . 4 4

Α , , b - r2 , , α , , ντιστροφα, αν ει ναι χ = -- , οπου r ρητος με r > -- , τοτε εχουμε ( 2 J2 b - r 4χ2 - αχ + b = 4 --α + 4r

α + 4r 4 α (b - r2 )

+ b = 4r4 + α2r2 + 4b2 + 8br2 + 4αbr + 4αr3 α + 4r (α + 4r)2

οπότε, αφού από υπόθεση α2 - 1 6b < Ο , θα είναι

( 2r2 + αr + 2b) 2 (α + 4r)2

Υ = .J4x2 - αχ + b = 2r2 + αr + 2b = 2r2 + αr + 2b Jα + 4r J 4r + α ' r > - α , δηλαδή ο y είναι ρητός.

4

2": τρ<ίπος ( � ΗΗ'iλαος Κολλ�Μιουλος) Θα αποδείξουμε πρώτα ότι όλα τα ζητούμενα χ δίνονται από έναν συγκεκριμένο τύπο που

περιέχει μια ρητή παράμετρο, και στη συνέχεια θα δείξουμε ότι κάθε χ που προκύπτει από το συγκεκριμένο τύπο ικανοποιεί το ζητούμενο, οπότε έτσι θα έχουμε έναν τύπο που θα μας δίνει ακριβώς όλα τα ζητούμενα χ μέσω μιας ρητής παραμέτρου.

'Εστω λοιπόν ότι για κάποιο χ ρητό, ο αριθμός .J 4χ2 - αχ + b είναι ένας θετικός ρητός αριθ­μός, έστω m , οπότε ισχύει: 4χ2 - αχ + b = m2 <=> 4χ2 - αχ + (b - m2 ) = Ο .

Αφού το παραπάνω τριώνυμο έχει ρητούς συντελεστές και μια ρητή ρίζα χ , πρέπει η διακρί­νουσά του να είναι το τετράγωνο ενός ρητού αριθμού, έστω n , και να ισχύει δηλαδή :

Επειδή τώρα οι m, n είναι ρητοί αριθμοί, έχουμε ότι ο αριθμός k = n - 4m είναι επίσης ρητός, και διάφορος του Ο αφού διαφορετικά θα είχαμε α 2 = 1 6b που είναι άτοπο από την υπόθεση .

π , , , 4 α2 - 1 6b α 2 - 1 6b ροφανως τωρα ισχυει και n + m = ---n - 4m k

ΕΥΚΛΕΙΔΗΣ Β ' 83 τ. 3/16

Page 19: Ευκλειδης Β 83

--------- Μαθηματικοί Διαγωνισμοί - Μαθηματικές Ολυμπιάδες --------

Λύνοντας το σύστημα: { n- 4m=k, n + 4m= α' �l 6b}

, 1 α 2 - 1 6b 1 α 2 - 16b ως προς (m, n) θα λαμβανουμε (m, n) = (- ( - k),- ( + k)) ,

8 k 2 k

και επειδή ο αριθμός χ είναι ρίζα του τριωνύμου που έχει διακρίνουσα n 2, θα έχουμε τελικά:

1(α 2- 16b k) α ± - + α ± n 2 k χ = --= ---=---:.:._ __ _

8 8 Α ντίστροφα, αν ο k είναι ο τυχαίος μη μηδενικός ρητός αριθμός, τότε είναι θέμα απλών πρά­

ξεων να επαληθεύσουμε ότι για τους ρητούς αριθμούς χ, m που δίνονται από τους παραπάνω

τύπους για το συγκεκριμένο k , ισχύει πράγματι

4χ 2 - ax + (b- m 2)= 0 <;:::;> .J4x 2 - ax + b = lml και άρα ο χ ικανοποιεί το ζητούμενο.

Άρα ένας τύπος που μας δίνει ακριβώς τα ζητούμενα χ είναι ο ακόλουθος:

α ± _!_ (α2 - 1 6b + k) α ± n 2 k χ=-- = --=------'-'---

8 8 όταν ο k είναι ένας τυχαίος μη μηδενικός ρητός αριθμός.

Π α ρ ατη ρι1 σε ις :

1 . Για να μη προκύπτει η ίδια λύση για διαφορετικά k , αρκεί να κρατήσουμε μόνο το + στον

παραπάνω τύπο και να περιοριστούμε στη συνέχεια στα k με απόλυτη τιμή μικρότερη η ίση

από .Jι 6b - α 2 • Αυτό επειδή μια λύση που προκύπτει για κάποιο k = k0 από τον τύπο που

έχει -, προκύπτει και από τον τύπο που έχει + θέτοντας όπου k το - k0 , ενώ επειδή για συ-

α + - + 1 ( α2 - 1 6b k)

γκεκριμένο χ η εξίσωση χ = 2 k ως προς k ανάγεται σε δευτεροβάθμια με 8

σταθερό συντελεστή 1 6b- α 2 κατ' απόλυτη τιμή και συντελεστή του k 2 τη μονάδα, είναι απλό να δούμε ότι αν έχει 2 διακεκριμένες πραγματικές ρίζες, τότε ακριβώς μια θα έχει από-

λυτη τιμή μικρότερη η ίση από .J 1 6b - α 2 ( αφού το γινόμενο των ριζών θα είναι ίσο με το

σταθερό συντελεστή)

2 . Είναι θέμα απλών πράξεων να δούμε ότι στον τύπο που δίνει τις λύσεις στον πρώτο

τρόπο επίλυσης του προβλήματος, αν αντικαταστήσουμε το r με το - α - k ,

4 λαμβάνουμε τον τύπο που βρήκαμε στην παραπάνω λύση με έναν διαφορετικό

περιορισμό για τον ρητό k , ο οποίος όμως μας δίνει πάλι όλες τις λύσεις.

3 . Η λύση αυτή μπορεί να χρησιμοποιηθεί αναλόγως και για το 4° πρόβλημα της Β ' Λυκείου.

ΕΥΚΛΕΙΔΗΣ Β ' 83 τ. 3/17

Page 20: Ευκλειδης Β 83

--------- Μαθηματικοί Διαγωνισμοί - Μαθηματικές Ολυμπιάδες

Επιτροπή Διαγωνισμών της Ε.Μ.Ε.

Λύσεις των ασκήσεων τεύχους 82/2011 Δίνεται τρίγωνο ABC με βαρύκεντρο G και περίκεντρο Ο τέτοια ώστε OG .l A G . Έστω Α1

το δεύτερο σημείο τομής της ευθείας A G με τον περιγεγραμμένο κύκλο του τριγώνου ABC . Έστω ακόμη D το σημείο τομής των ευθειών ΒΑ1 και Α C , και Ε το σημείο τομής των ευθειών CA1 και ΑΒ . Να αποδείξετε ότι το περίκεντρο του τριγώνου ADE βρίσκεται πάνω στον περιγεγραμμένο κύκλο του τριγώνου ABC . [Εσθονία, 2006]

-- --- ------- -- --- ----- ----- -- -- ----------- -- -- -- -----Σχήμα 1

Έστω Ζ το μέσον της πλευράς BC , Η το μέσον της πλευράς AC και Θ το μέσον της πλευράς ΑΒ . Επειδή η OG περνάει από το κέντρο του περιγεγραμμένου κύκλου του τριγώνου ABC και είναι κάθετη προς τη χορδή ΑΑ1 ,

έπεται ότι το σημείο G είναι το μέσον της ΑΑ1 • Όμως το

G είναι το βαρύκεντρο του τριγώνου ABC , οπότε

GZ = G: =

G�ι , δηλαδή το Ζ είναι μέσον του GA1 ,

όπως επίσης είναι μέσον και της πλευράς BC . Επομένως, οι διαγώνιοι του τετραπλεύρου GBA1C διχοτομούνται,

οπότε αυτό είναι παραλληλόγραμμο. Έτσι έχουμε ΒΗ 1 1 EC και CΘ 1 1 BD , οπότε

ΑΒ ΑΗ 1 AC ΑΘ 1 - = - = - και - = - = -ΑΕ AC 2 AD ΑΒ 2 '

από τις οποίες προκύπτει ότι τα τρίγωνα ABC και AED είναι ομοιόθετα ως προς κέντρο ομοιοθεσίας το

Α και λόγο λ = _!_ . Α ν Κ είναι το κέντρο του περιγεγραμμένου κύκλου του τριγώνου ABC , τότε θα ισχύ-2

ει ΑΚ = 2 · ΑΟ , οπότε το Κ βρίσκεται πάνω στον περιγεγραμμένο κύκλο του τριγώνου ABC . Λ Σε ισοσκελές τρίγωνο ABC με ΑΒ = A C , η διχοτόμος της γωνίας Β τέμνει την πλευρά A C

στο σημείο D . Αν επιπλέον ισχύει η ισότητα B C = BD + AD , να βρείτε όλες τις γωνίες του τριγώ-νου ABC . [Καναδάς, 1996]

Θεωρούμε σημείο Ε πάνω στην πλευρά BC τέτοιο ώστε CE = AD, οπότε, λόγω της υπόθεσης, θα είναι και BE = BD.

Α

Σχήμα 2

ΕΥΚΛΕΙΔΗΣ Β' 83 τ. 3/18

Page 21: Ευκλειδης Β 83

--------- Μαθηματικοί Διαγωνισμοί - Μαθηματικές Ολυμπιάδες -------­Επειδή η BD είναι διχοτόμος της γωνίας Β , από το θεώρημα της διχοτόμου λαμβάνουμε:

CD BC CD CD BC BC - = - => - = - = - = -AD ΑΒ CE AD ΑΒ AC

.

Έτσι τα τρίγωνα ABC και EDC , έχουν κοινή τη γωνία της κορυφής C και τις πλευρές που περιέχουν

την κοινή γωνία ανάλογες, οπότε είναι όμοια. Επομένως θα έχουν και CED = Α . Αν τώρα θέσουμε

Β = C = 2φ, τότε CBD = DBA = φ και CED = A = 1 80° - 4φ. ( 1 ) Επειδή το τρίγωνο BED είναι ισοσκελές, θ α έχουμε ότι: BED = 90° - φ . (2) 2 Από τις ( ι ) και (2) και την ισότητα BED + DEC = ι 8Οο λαμβάνουμε:

90° - φ + ι 8Οο - 4φ = ι 8Οο <=> 9φ = 90° <=> φ = 20° , οπότε θα είναι Β = C = 40° και Α = 1 00°. 2 2 Να προσδιορίσετε όλους του πραγματικούς αριθμούς α για τους οποίους έχει μοναδική λύση

( x,y) Ε JRz το σύστημα: 2ΙχΙ + lxl = xz + y + α, xz + yz = 1 . [Μόσχα, 2006]

Λ Παρατηρούμε ότι, αν ( χ, y) είναι μία λύση του συστήματος , τότε και το ζευγάρι (-χ, y) είναι επίσης

λύση . Επομένως, αν το σύστημα έχει μοναδική λύση, τότε αυτή πρέπει να είναι της μορφής (Ο, y ) , y Ε JR.

{ι = y + α

} {α = Ο

} { α = 2 }

Για χ = Ο το σύστημα γίνεται: 2 <=> ή . y = ι y = 1 y = -ι Για α = Ο , το δεδομένο σύστημα γίνεται: {2ΙχΙ + l x l = χ 2 + y, χ2 + y2 = ι} ,το οποίο προφανώς έχει τη λύ­

ση (χ, y) = ( Ο, ι ) . Θα αποδείξουμε ότι η λύση αυτή είναι μοναδική . Πράγματι, από τη δεύτερη εξίσωση

λαμβάνουμε Ο ::::; l x l ::::; ι και Ο ::::; I YI ::::; ι , οπότε θα είναι και l x l ;::: χ2• ( 1 )

Επίσης έχουμε 2Ιχ Ι ;::: 2° = ι ;::: I YI ;::: y , (2) οπότε από τις ( ι ) και (2) προκύπτει ότι 2Ιχl + l x l ;::: χ 2 + y , ενώ η ισότητα ισχύει όταν l x l = Ο και

I YI = y <=> χ = Ο και y ;::: Ο . Τότε από τη δεύτερη εξίσωση προκύπτει ότι y = ι και το σύστημα έχει μόνο

τη λύση ( x, y) = (ο, ι ) . Για α = 2 , προκύπτει το σύστημα { 2ΙχΙ + l x l = χ 2 + y + 2, χ2 + y2 = ι} το οποίο έχει

τη λύση ( χ, y) = (Ο, - ι ) , που βρήκαμε στην αρχή, αλλά και άλλες δύο τουλάχιστον λύσεις, τις

(χ, y) = ( ι, Ο) και ( χ, y) = (-ι , Ο) . Οι δύο τελευταίες λύσεις προκύπτουν εύκολα, αν πάρουμε χ = Ο . Επομένως, η τιμή α = 2 δεν είναι δεκτή και το σύστημα έχει μοναδική λύση, αν, και μόνο αν, α = Ο.

Λ5. Βρείτε όλες τις συναρτήσεις f : Ζ � Ζ για τις οποίες ισχύει ισότητα

f {f (a) + f (b)) = a + b - 1, (1) για κάθε a,b E Z. [Κροατία, 2011]

Αν θέσουμε b = ι στη σχέση ( ι ), λαμβάνουμε: f ( f ( a ) + f ( ι )) = a, για κάθε ακέραιο a. (2) Έστω τώρα k τυχαίος ακέραιος. Θέτουμε a = f ( k) + f ( ι ) , b = 2f ( ι ) , οπότε από την ( 1 ) θα έχουμε

f (a) = k και f (b) = ι και επιπλέον f (k + ι ) = f (k ) + 3f ( ι ) - 1 . (3) Θέτουμε f ( ι ) = c , οπότε από τη σχέση (3) λαμβάνουμε f (0) = 1 - 2f ( 1 ) = -2c + ι , f (- ι ) = -5c + 2 και

επιπλέον f ( k + ι ) - f ( k) = 3c - 1 , για κάθε μη αρνητικό ακέραιο k . ( 4)

f ( -k) - f ( -k + ι ) = - (3c - 1) , για κάθε μη αρνητικό ακέραιο k . (5)

Άρα η ακολουθία f (k ) , k = 0, 1 , 2, . . . είναι αριθμητική πρόοδος με πρώτο όρο f (O) = -2c + ι και διαφορά

ω = 3c - ι , οπότε θα είναι f (η ) = -2c + ι + η ( 3c - ι ) = ( 3η - 2) c - η + ι, η = Ο, ι, 2, 3, . . . ( 4)

Ομοίως η ακολουθία f ( -k ) , k = 0, 1 , 2, . . . είναι αριθμητική πρόοδος με πρώτο όρο f (O) = -2c + 1 και δια­

φορά -(3c - ι ) , οπότε θα είναι f ( -η ) = -2c + ι - η (3c - ι ) = ( -3η - 2) c + η + ι, η = 0, 1 , 2, 3, . . . ΕΥΚΛΕΙΔΗΣ Β' 83 τ. 3/19

Page 22: Ευκλειδης Β 83

--------- Μαθηματικοί Διαγωνισμοί - Μαθηματικές Ολυμπιάδες -------­Επομένως, για κάθε η Ε Ζ καταλήγουμε στον ενιαίο τύπο f (η ) = ( 3η - 2) c - η + 1, η Ε Ζ, c = f ( 1 ) . Από την αρχική σχέση για a = Ο, b = 1 λαμβάνουμε

f ( f (Ο) + f ( 1 ) ) = Ο <=> f ( -c + 1 ) = Ο <=> ( 3 ( -c + 1 ) - 2) c - ( -c + 1 ) + 1 = Ο<=> c (-3c + 2) = Ο <=> c = Ο,

αφού c = f ( 1 ) Ε Ζ . Επομένως έχουμε τη συνάρτηση f (η ) = -η + 1, η Ε Ζ , η οποία εύκολα επαληθεύουμε

ότι ικανοποιεί τη δεδομένη συνθήκη . Ν3 . Να λύσετε στους ακέραιους την εξίσωση: χ2 {y - 1) + y2 {χ - 1) = 1 .

ΛίJση e (Πολωνία, 1996]

'Ε χουμε χ 2 ( y - 1 ) + y2 (χ - 1 ) = 1 <=> χ 2 y - χ 2 + xy2 - y2 = 1 <=> xy (χ + y ) - [ (χ + y )2 - 2xy J = 1 .

Θέτουμε u = χ + y και ν = χy, οπότε λαμβάνουμε την εξίσωση

uν - (u2 - 2ν) = 1 <:::> uν + 2ν = u2 + 1 <::> ν = u2 + 1 = u - 2 +-5- . u + 2 u + 2 Επειδή ζητάμε ακέραιες λύσεις, πρέπει ο u + 2 να είναι διαιρέτης του 5 , οπότε:

U + 2 Ε {-5, - 1, 1, 5} <:::> U Ε {-7, - 3, - 1, 3} . Επομένως λαμβάνουμε τα ζευγάρια: ( u, ν) = ( -7, - 1 0) , ( u, ν) = ( -3, - 10) , ( u, ν) = ( -1, 2) , (υ, ν) = (3, 2) , οπότε τα χ , y είναι αντίστοιχα ο ι λύσεις των εξισώσεων

ω2 + 7ω - 1 0 = Ο, (έχει ρίζες άρρητες, αφού Δ = 89 =t- κ2 , κ Ε Ζ),

ω2 + 3ω - 1 0 = Ο <=> ω = -5 ή ω = 2, ω2 + ω + 2 = 0, ( αδύνατη στο JR), ω2 - 3ω + 2 = 0<=> ω = 1 ή ω = 2.

Άρα έχουμε τις λύσεις: (x, y ) = ( 1, 2 ) ή ( x, y) = (2, 1 ) ή ( x, y) = ( -5, 2) ή ( x, y) = (2, -5) . Ν 4 . Έστω p { k) ο μεγαλύτερος περιττός διαιρέτης του θετικού ακέραιου k . Ν α αποδείξετε ότι, για

2η p (l) p ( 2) p (η) 2(η + ι) κάθε θετικό ακέραιο η , ισχύει: - < -- + -- + ... + -- < . 3 1 2 η 3 ΛίJση

p ( 1 ) p ( 2) p (η ) 2η 2 (η + 1 ) Έστω S( η ) =--+--+ . . . +--. Τότε πρέπει να αποδείξουμε ότι: - < S( η ) < . (1) Ι 2 η 3 3 Θα εφαρμόσουμε μία ισχυρή μορφή της μαθηματικής επαγωγής, ως εξής: • Για η = 1 και η = 2 , η πρόταση αληθεύει. • Υποθέτουμε ότι αληθεύει η πρόταση για όλους τους θετικούς ακέραιους που είναι μικρότεροι ενός

θετικού ακέραιου k . • Θα αποδείξουμε ότι η πρόταση αληθεύει και για η = k . Πράγματι, η σχέση ( 1 ) αληθεύει για η = 1 και η = 2 , αφού είναι

� < S ( 1 ) = Ρ ( Ι ) = 1 < i και i < S ( 2) = Ρ ( 1 ) + Ρ ( 2) = 1 + _!._ = � < � = 3 . 3 Ι 3 3 1 2 2 2 2 Υποθέτουμε ότι η πρόταση αληθεύει για όλους τους θετικούς ακέραιους η που είναι μικρότεροι του k και θα αποδείξουμε ότι αληθεύει και για η = k. Παρατηρούμε ότι για τους άρτιους θετικούς ακέραιους ισχύει p ( 2k) = p ( k ) , ενώ για τους θετικούς πε-

ριττούς ακέραιους ισχύει p ( 2k + 1 ) = 2k + 1 . Στη συνέχεια υποθέτουμε ότι ο k είναι περιττός, έστω

k = 2m + Ι . Τότε έχουμε s (2m + Ι) = ( p(l ) + Ρ (3) + . . . + P (2m + l ) J + ( P (2 ) + Ρ ( 4) + . . . + P (2m) J Ι 3 2m + 1 2 4 2m = ( 1 + 1 + . . . + 1 ) +

_!_( p (1 ) + p (2 ) + . . . + p (m) J = m + I + s (m) . 2 1 2 m 2 Όμως, από την υπόθεση της επαγωγής έχουμε

ΕΥΚΛΕΙΔΗΣ Β' 83 τ. 3/20

Page 23: Ευκλειδης Β 83

---------- Μαθηματικοί Διαγωνισμοί - Μαθηματικές Ολυμπιάδες

m s (m) m + 1 4m + 3 4m + 4 m + 1 +- < m + 1 + -- < m + 1 + -- <=> < s (2m + 1 ) < ---3 2 3 3 3 2 (2m + 1 ) 4m + 3 ( ) 4m + 4 2 (2m + 2 )

=> < < s 2m + 1 < = , 3 3 3 3 που είναι η σχέση που ζητούσαμε. Ομοίως εργαζόμαστε για την περίπτωση που ο k είναι άρτιος. Δ Ι . Σε έναν 5 χ 7 πίνακα έχουμε χρωματίσει ένα μόνο μοναδιαίο τετράγωνο κόκκινο, ενώ όλα τα υπόλοιπα είναι λευκά. Διαθέτουμε πλακίδια επικάλυψης του πίνακα σχήματος L, όπως στο παρα-

Εbο κάτω σχήμα, τα οποία μπορούν να περιστρέφονται κατάλληλα. Τα πλακίδια πρέπει να επικαλύπτουν ακριβώς τέσσερα μοναδιαία τετράγωνα και μπορούν να επικαλύπτονται στο κόκκινο τετράγωνο, όχι όμως στα λευκά τετράγω­να. Βρείτε όλες τις δυνατές θέσεις του κόκκινου τετραγώνου για τις οποίες είναι δυ-

νατή η πλήρης κάλυψη του πίνακα με L-πλακίδια. [Σλοβενία ΜΟ, 2009] Λύση

Ο πίνακας έχει 35 μοναδιαία τετράγωνα, οπότε για την πλήρη κάλυψη του θα πρέπει να χρησιμοποιή­σουμε 9 πλακίδια και ταυτόχρονα το κόκκινο τετράγωνο να καλυφθεί δύο φορές.

Αριθμούμε τις γραμμές του πίνακα από το 1 μέχρι το 5 και τις στήλες του πίνακα από το 1 μέχρι το 7 .

-τ τ-l -+-� ' J

r- 1

l Σχήμα 3

Ας υποθέσουμε ότι σκιάζουμε όλα τα τετράγωνα των περιττών γραμμών του πίνακα, δηλαδή συνολικά σκιάζουμε 2 1 τετράγωνα. Κάθε τοποθετούμενο πλακίδιο μπορεί να καλύψει τρία σκιασμένα τετράγωνα και ένα λευκό ή τρία λευκά και ένα σκιασμένο. Έστω για την πλήρη κάλυψη του πίνακα χρησιμοποιούμε k πλακίδια που καλύπτουν τρία σκιασμένα τετράγωνα, οπότε 9 - k πλακίδια θα καλύπτουν ένα μόνο

σκιασμένο τετράγωνο. Έτσι συνολικά θα καλύπτονται 3k + 1 · ( 9 - k ) = 2k + 9 σκιασμένα τετράγωνα, δη­

λαδή περιττός αριθμός τετραγώνων, όπως και ο αριθμός 2 1 . Επομένως δεν είναι δυνατόν κάποιο από τα σκιασμένα τετράγωνα να είναι το κόκκινο τετράγωνο.

_,. "'

Σχήμα 4 Ομοίως, αν σκιάσουμε τα τετράγωνα των άρτιων στηλών του πίνακα με το ίδιο σκεπτικό μπορούμε να

αποδείξουμε ότι δεν είναι δυνατόν κάποιο από τα σκιασμένα τετράγωνα να είναι το κόκκινο τετράγωνο.

ΕΥΚΛΕΙΔΗΣ Β' 83 τ. 3/21

Page 24: Ευκλειδης Β 83

--------- Μαθηματικοί Διαγωνισμοί - Μαθηματικές Ολυμπιάδες --------­Επομένως τα πιθανά τετράγωνα που μπορεί να είναι κόκκινα είναι αυτά που ανήκουν ταυτόχρονα σε άρ­τια γραμμή και περιττή στήλη, δηλαδή τα τετράγωνα (2, 1 ), (2,3),(2,5), (2,7), (4, 1 ), (4,3),(4,5), (4,7). Με απ' ευθείας επαλήθευση μπορούμε να αποδείξουμε ότι καθένα από αυτά μπορεί να είναι το κόκκινο τε­τράγωνο. Στο σχήμα φαίνεται η επαλήθευση για ένα από αυτά και συγκεκριμένα για το τετράγωνο (2, 1 ). ; ! οφαη)ρηση : (Από τ.82, σελίδα 16) Στην άσκηση Αι η f που βρίσκουμε, είναι ορισμένη προφανώς στο

ll( και όχι στο ffi. .

Ασκήσεις για λύση Να προσδιορίσετε τη συνάρτηση f : ffi. � ffi. , η οποία ικανοποιεί την ισότητα

f (xf (y) + f (x ) ) = 2f (x ) + xy, για κάθε x, y ε ffi..

\7. Να βρεθούν όλες οι τριάδες θετικών ακέραιων ( χ, y , z ) , που είναι λύσεις της εξίσωσης ( Χ + y ) ( 1 + xy) = 2z .

Α8. Θεωρούμε τους μη αρνητικούς πραγματικούς αριθμούς a, b, c και τους θετικούς πραγματικούς αριθ­μούς χ, y, z τέτοιους ώστε a + b + c = χ + y + z . Ν α αποδείξετε ότι:

aJ bJ cJ

-2 + -2 + ----z ::Ξ': a + b + c. χ Υ z

'\ 5 . Να βρείτε όλους τους πρώτους διαιρέτες του αριθμού Α = 3ι 6 - 2ι 6 • "\ 6 Να προσδιορίσετε όλα τα ζεύγη θετικών ακέραιων (m, n) που είναι λύσεις της εξίσωσης

3 · 2m + 1 = n2 •

Α ' b ' ' ' a b c 5 δ 'ξ ' θ ' b ' 'λε \ 7. ν οι ακεραιοι a, , c ει ναι τετοιοι ωστε - + - + - = , να απο ει ετε οτι ο αρι μος a c ειναι τε ι-b c a

ος κύβος.

r ,ι Δίνεται τρίγωνο ABC και σημείο Χ στο εσωτερικό του. Από τις κορυφές Α, Β, C θεωρούμε ευθείες �.�,ε;,αντίστοιχα, τέτοιες ώστε ει .lXA, � .lXB κm � .lXC Α ν είναι ε2 n ε3 =Αι , ε3 ί'ι ει = Α2 , ει ί'ι ε2 = Α3 και Ηι , Η2 , Η3 είναι τα ορθόκεντρα των τριγώνων ΑιΒC, A2CA, Α3ΑΒ, αντίστοιχα, να αποδείξετε ότι τα τρίγωνα ABC και Η ιΗ2Η3 είναι ίσα.

r5. Δίνεται παραλληλόγραμμο ABCD με AC > BD και BAC = 45° . Έστω kι ο κύκλος διαμέτρου AC και k2 ο κύκλος διαμέτρου DC . Ο κύκλος k ι τέμνει την ευθεία ΑΒ στο Ε , ο κύκλος k2 τέμνει την ευ­θεία AC στα σημεία C και Ο , και την ευθεία AD στα σημεία D και F . Αν είναι ΑΟ = a και FO = b,

να βρείτε το λόγο των εμβαδών των τριγώνων ΑΟΕ και COF.

Γ6. Θεωρούμε τρίγωνο ABC με Α :Ξ>: 60° . Αν BD και CE είναι οι διχοτόμοι των γωνιών του Β και C , αντίστοιχα, να αποδείξετε ότι: AD + ΑΕ � BC .

Λ Λ ! " 7 . Σε κυρτό τετράπλευρο ABCD οι διχοτόμοι των γωνιών Α και C τέμνονται στο σημείο Ι . Να απο-δείξετε ότι το τετράπλευρο είναι περιγράψιμο περί κύκλο (όλες οι πλευρές του εφάπτονται σε ένα κύκλο, αν, και μόνον αν, ( ΑΙΒ) + ( CID) = ( AID) + (BIC) .

'Ω: . Να προσδιορίσετε το συντελεστή του χ2 στο πολυώνυμο Ρ ( χ ) = ( 1 + χ ) ( 1 + 2χ ) ( 1 + 4χ ) · · · ( 1 + 2" χ) .

113 . Θεωρούμε επτά σημεία στο επίπεδο, ανά τρία μη συνευθειακά. Οποιαδήποτε δύο σημεία από αυτά συνδέονται με ευθύγραμμο τμήμα. Είναι δυνατόν να χρωματίσουμε αυτά τα ευθύγραμμα τμήματα με κά­ποια χρώματα έτσι, ώστε να υπάρχουν τρία ακριβώς ευθύγραμμα τμήματα από κάθε χρώμα που να σχη­ματίζουν τρίγωνο με κορυφές κάποια από τα επτά σημεία;

ΕΥΚΛΕΙΔΗΣ Β' 83 τ. 3/22

Page 25: Ευκλειδης Β 83

--------- Μαθηματικοί Διαγωνισμοί - Μαθηματικές Ολυμπιάδες ---------

Στη μνήμη Peter Ο' Halloran

Ένα μεσογειακό πολυώνυμο έχει μόνο πραγματικές ρίζες και είναι της μορφής Ρ( χ) = χ10 - 20χ9 + 135χ8 + a χ 7 + a χ6 + a χ5 + a χ4 + a χ3 + a χ2 + a χ + a 1 6 s 4 3 2 ι Ο '

με πραγματικούς συντελεστές a0 , a1 , ••• , a7 • Ν α προσδιορίσετε τον μεγαλύτερο πραγματικό αριθμό

που μπορεί να είναι ρίζα ενός μεσογειακού πολυωνύμου.

Θεωρούμε ένα μεσογειακό πολυώνυμο πού έχει τις πραγματικές ρίζες α και χ ι , χ2 , • • • , χ9 . 'Εστω 9 9

s = Σxi , t = Σx� και u = Σ x ix j . Από τους τύπους Vieta έχουμε ότι i = ι

s = 20 - α, u = 1 3 5 - sα = 1 3 5 - 20α + α2 και t = s2 - 2u = 1 30 - α2 , οπότε λαμβάνουμε ο � Σ (x i - xj )2 = 8t - 2u = ιο (α + 7) ( 1 1 - α) .

IS i <j:-ς9 Άρα είναι α � 1 1 . Επειδή για α = 1 1 και χ ι = χ2 = . . . = χ9 = 1 προκύπτει ένα μεσογειακό πολυώνυμο Ρ (χ ) = (χ - 1 1 ) ( χ - 1 )9 = (χ - 1 1 ) ( χ9 - 9χ8 + 36χ7 - 84χ6 + 126χ5 - 1 26χ4 + 84χ3 - 36χ2 + 9χ - 1 )

1 0 20 9 1 35 8 7 6 5 4 3 2 = χ - χ + χ + a7x + a6x + a5x + a4x + a3x + a2x + aι x + a0 •

'Εστω Α ένα πεπερασμένο σύνολο θετικών πραγματικών αριθμών, έστω Β = {� : x, y ε Α} και έ­

στω C = { xy : x, y ε Α} . Αν συμβολίσουμε τον πληθικό αριθμό του συνόλου Χ με cardX = /Χ/ , να

αποδείξετε ότι: /A I · IB I � /C/ 2 •

Ί • r. - ,

Για το στοιχείο b ε Β , έστω b = ��:� , με x (b ) , y (b ) ε A είναι μία αναπαράσταση του με χ (b) ελάχι­στο. Θεωρούμε τη συνάρτηση f : Α χ Β ---+ C χ C,( a, b ) ---+ f ( a, b ) := ( ax (b ) , ay (b ) ) . Θα αποδείξουμε ότι η συνάρτηση f είναι 1 - 1 . Πράγματι, αν θεωρήσουμε ( a1 , b1 ) και ( a2 , b2 ) στο σύνολο Α χ Β τέτοια ώ­στε f ( aι , bι ) = f ( aυ b2 ) , τότε

( a ι χ ( bι ) , a ι y (bι ) ) = ( a2x (b2 ) , a2y (b2 )) � aι χ (bι ) = a2x (b2 ) και a ι y (bι ) = a2y (b2 ) οπότε με διαίρεση κατά μέλη προκύπτει ότι

χ (( bι )

) = χ

(( bz )

) � bι = b2 • Υ bι Υ bz

Τότε από την ισότητα aι χ (b ι ) = a2x (b2 ) προκύπτει ότι , οπότε θα είναι και ( a ι , bι ) = ( aυ b2 ) . Επειδή η

συνάρτηση f είναι 1 - 1 ' έπεται ότι /Α χ Β / = ιr (Α χ Β )I � ιc χ cι � /A I · IB I � /C /2 •

Από κανονικό τετράεδρο ύψους h αποκόπτεται με επίπεδο παράλληλο προς τη βάση του ένα τε­τράεδρο ύψους xh . Όταν το στερεό που απομένει τοποθετείται με τη μία από τις παράπλευρες έ­δρες του πάνω σε οριζόντιο επίπεδο, τότε η προβολή του κέντρου βάρους του G στο οριζόντιο επί­πεδο είναι ένα σημείο της μικρής βάσης της παράπλευρης έδρας του. Να αποδείξετε ότι το χ είναι ρίζα της εξίσωσης χ3 + χ2 + χ = 2 .

ΕΥΚΛΕΙΔΗΣ Β ' 83 τ. 3/23

Page 26: Ευκλειδης Β 83

--------- Μαθηματικοί Διαγωνισμοί - Μαθηματικές Ολυμπιάδες

ΛίJση :

Έστω Ο η κοινή κορυφή των δύο τετραέδρων και Α , Α1 τα κέντρα των δύο βάσεων (το Α1 είναι το κέντρο της βάσης του αποκομμένου τετραέδρου ύψους xh . Οι όγκοι των δύο τετραέδρων έχουν λόγο 1 : χ3 και οι αποστάσεις των κέντρων βάρους τους από την

κορυφή Ο είναι i h και i xh, αντίστοιχα. Α ν G είναι το κέντρο βάρους της κόλουρης πυραμίδας, τότε 4 4

έχουμε 3 3 4 -h - -x h 3 ( I + x ) ( l + xz ) OG = χ = 4 4 = -h --'------'------'--------=-...:....

1 - χ3 4 l + x + x 2

ο

Σχήμα 1

h (3 - x - x2 - χ3 ) οπότε θα είναι Α G = χ - xh = ( 1 ) I 4( 1 + χ + χ2 ) Έστω ΑΝ η κάθετη από το Α προς τη μεγάλη βάση της έδρας που εφάπτεται του οριζόντιου επιπέδου

και έστω ότι η ΟΝ τέμνει τη μικρή βάση στο σημείο Ν' . Σύμφωνα με τα δεδομένα του προβλήματος η

GN, είναι κατακόρυφη . Τώρα, επειδή η ακμή του μεγαλύτερου τετραέδρου είναι � h , θα είναι

ι J3 J3 h ' xh ΑΝ = 3 -2 .

J2 h = 2J2

, Οποτε Α,Ν, = 2J2

. Επειδή είναι .LONιG = 90" και η Ν,Α, είναι ύψος

τριγώνου ΟΝρ , έπεται ότι προς την υποτείνουσα του ορθογώνιου

Αρ · ΟΑ, = (Νρ)2 � Αρ · χh = (2�)2 � Aρ = lxh

ικανοποιεί την εξίσωση : χ3 + χ2 + χ = 2 .

(2). Από τις ( 1 ) και (2) λαμβάνουμε ότι ο χ

Σχήμα 2 ΕΥΚΛΕΙΔΗΣ Β ' 83 τ. 3/24

Page 27: Ευκλειδης Β 83

--------- Μαθηματικοί Διαγωνισμοί - Μαθηματικές Ολυμπιάδες

Π αρατή ρηση

Η τελευταία εξίσωση έχει μία πραγματική ρίζα και δυο συζυγείς μιγαδικές. Η πραγματική ρίζα είναι η

1 r;;;;:; � 6 1 2 1 3 -ν108ν 1 39 + - - -- = 0 8 1 054 1 08 54 9 3 -1-JlOSJlli + � 3

,

1 08 54

Π ρόβλη μα 4 Έστω D το σημείο τομής της εσωτερικής διχοτόμου της γωνίας Α τριγώνου ABC με την απένα­

ντι πλευρά του. Η ευθεία που περνάει από τα κέντρα των εγγεγραμμένων κύκλων των τριγώνων ABD και ACD τέμνει τις ΑΒ και AC στα σημεία Μ και Ν, αντίστοιχα. Να αποδείξετε ότι οι ευ­θείες ΒΝ και CM τέμνονται πάνω στη διχοτόμο AD .

ΛίJση Για τη λύση του προβλήματος θα χρησιμοποιήσουμε μία βοηθητική πρόταση που είναι γνωστή ως θεώ­

ρημα των διατεμνουσών.

Λήμμα : Δίνεται τρίγωνο ΑΒΓ και έστω Ι το σημείο τομής των διχοτόμων του. Μία διατέμνουσα ε των πλευρών του τριγώνου ΑΒΓ περνάει από το Ι και τέμνει τις πλευρές του ΑΒ, ΑΓ και ΒΓ στα σημεία

Ε, Ζ και Κ αντίστοιχα. Τότε ισχύει: ΕΒ · ΑΓ + ΖΓ · ΑΒ = ΒΓ . ΕΑ ΖΑ

Απ66ειξη

Έστω ΑΔ η διχοτόμος της γωνίας Α . Από το θεώρημα του Μενελάου στα τρίγωνα ΑΒΔ και ΑΔΓ με διατέμνουσα την ευθεία ε λαμβάνουμε:

Α

κ Β Δ

Σχήμα 3

Από το θεώρημα του Μενελάου στα τρίγωνα ΑΒΔ και ΑΔΓ με διατέμνουσα την ευθεία ε λαμβάνου-ΒΕ ΑΙ ΔΚ ΓΖ ΑΙ ΔΚ ΒΕ ΑΙ ΚΒ ΓΖ ΑΙ ΚΓ , με: - · - · - = 1 και - · - · - = 1 <=> - · - = - και - · - = -. Επειδη από το θεώ-ΕΑ ΙΔ ΚΒ ΖΑ ΙΔ ΚΓ ΕΑ ΙΔ ΚΔ ΖΑ ΙΔ ΚΔ '

' δ ' ' ΑΒΔ ΑΔΓ ' ' ΑΙ ΑΒ ΑΓ ρημα της εσωτερικης ιχοτομου στα τριγωνα και ισχυει οτι: ΙΔ

= ΒΔ

= Γ Δ

,

, , , λ β , ΒΕ ΑΓ ΚΒ ΓΖ ΑΒ ΚΓ απο τις προηγουμενες σχεσεις αμ ανουμε: ΕΑ

· Γ Δ

= ΚΔ

και ΖΑ

· ΒΔ

= ΚΔ

ΒΕ ΑΓ ΚΒ ΓΖ ΑΒ ΚΓ ΒΕ ΚΒ ΓΖ ΚΓ � - · - · ΓΔ = - - ΓΔ και - · - · ΒΔ = - · ΒΔ� - · ΑΓ = - · ΓΔ και - · ΑΒ = - · ΒΔ. ΕΑ Γ Δ ΚΔ ΖΑ ΒΔ ΚΔ ΕΑ ΚΔ ΖΑ ΚΔ

Μ , θ , 'λ δ ' λ , , , ΒΕ ΑΓ ΓΖ ΑΒ ΚΒ · ΓΔ + ΚΓ · ΒΔ ε προσ εση κατα με η των υο τε ευταιων σχεσεων εχουμε: - · + - · = -------ΕΑ ΖΑ ΚΔ

Επειδή ισχύει ότι: ΚΒ · ΓΔ + ΚΓ · ΒΔ = ΚΒ · (ΒΓ -ΒΔ) + (ΚΒ + ΒΓ) · ΒΔ = ΚΒ · ΒΓ + ΒΓ · ΒΔ = ΒΓ · ΚΔ,

τελικά λαμβάνουμε: ΒΕ . ΑΓ + ΓΖ . ΑΒ = ΚΔ · ΒΓ = ΒΓ . ΕΑ ΖΑ ΚΔ

ΕΥΚΛΕΙΔΗΣ Β ' 83 τ. 3/25

Page 28: Ευκλειδης Β 83

--------- Μαθηματικοί Διαγωνισμοί - Μαθηματικές Ολυμπιάδες ---------ΛίJση προβλή ματος Έστω 1 1 , 12 τα σημεία τομής των διχοτόμων των τριγώνων ABD, ACD , αντίστοιχα, και έστω Q το

σημείο τομής των ευθειών 1 1 12 και AD . Επειδή η ευθεία MQ περνάει από το έκκεντρο 11 του τριγώνου ABD , από το προηγούμενο λήμμα λαμβάνουμε:

ΜΒ · AD + QD · c = BD . ( 1 )

ΜΑ QA "Α

Σχήμα 4

Ομοίως, επειδή η ευθεία QN περνάει από το 12 , από το προηγούμενο λήμμα λαμβάνουμε:

QD · b + NC

· AD = DC . (2) QA ΝΑ

Με διαίρεση των δύο μελών της ( 1 ) με c και της (2) με b , λαμβάνουμε: ΜΒ . AD + QD = BD ΜΑ c QA c

QD NC AD DC , , , , 'λ λ β , - + - · - = - , απο τις οποιες με αφαιρεση κατα με η αμ ανουμε: QA ΝΑ b b

ΜΒ . AD _ NC . AD = BD _ DC = Ο ' ΜΑ c ΝΑ b c b

(3)

όπου η τελευταία ισότητα προκύπτει από το θεώρημα της εσωτερικής διχοτόμου στο τρίγωνο ABC. Από τη σχέση (3), λαμβάνοντας υπόψη και την ισότητα

AD = 2bc sin Α (4)

b + c 2 '

ΜΒ 2bc . Α NC 2bc . Α ΜΒ NC Λαμβάνουμε -- · sιn- = - · sιn- <=> -- · b = - · c . (5) ΜΑ c (b + c) 2 ΝΑ b (b + c) 2 ΜΑ ΝΑ

Για να αποδείξουμε ότι οι ευθείες ΒΝ και CM τέμνονται πάνω στη διχοτόμο AD , αρκεί να αποδεί-

ξ , ΒΜ ΑΝ CD , ζ , , , , θ , ου με οτι -- · - · - = 1 , οποτε το ητουμενο προκυπτει με εφαρμογη του αντιστροφου εωρηματος

ΜΑ NC DB του Ceνa. Πράγματι, από τη σχέση (5) και το θεώρημα της εσωτερικής διχοτόμου στο τρίγωνο ABC έ­χουμε:

ΜΒ ΑΝ b ΜΒ ΑΝ CD - · - · - = 1 <=> - · - · - = 1 . ΜΑ NC c ΜΑ NC DB

ΕΥΚΛΕΙΔΗΣ Β' 83 τ. 3/26

Page 29: Ευκλειδης Β 83

ΗΟΜΟ MATHEMATICUS Η Homo Mathematicus είναι μια στήλη στο περιοδικό μας, με σκοπό την ανταλλαγή απόψεων και την ανάπτυξη προβληματισμού πάνω στα εξής θέματα: 1 ) Τι είναι τα Μαθηματικά, 2) Πρέπει ή όχι να διδάσκονται, 3) Ποιοι είναι οι κλάδοι των Μαθηματικών και ποιο το αντικείμενο του καθενός, 4) Ποιες είναι οι εφαρμογές τους, 5) Ποιες επιστήμες ή κλάδοι επιστημών απαιτούν καλή γνώση των Μαθηματικών για να μπορέσει κάποιος να τους σπουδάσει.

Για τους συνεργάτες της στήλης: παράκληση ! τα κείμενα της στήλης αυτής, ως προς το περιεχόμενό τους και ως προς το επίπεδό τους, θα πρέπει να είναι συμβιβαστά με τα ενδιαφέροντα και το επίπεδο κατανόησης από μέρους των παιδιών.

επιμέλεια : Γιάννης Κερασαρίδης Ι α . "τι είναι τ α ιWαΟηματικά; "

Προλεγ{ιμενα . Είναι γνωστό σε όλους πως ο αείμνηστος καθηγητής Δημήτρης Κάππος ήταν ένας από τους πρωταγωνιστές για την εγκαθίδρυση στη χώρα μας της "σύγχρονης μαθηματικής σχολής". Τιμώντας τον, δημοσιεύσουμε μέρος του «Προλόγου Πρώτης Έκδοσης» του βιβλίου του «ΑΠΕΙΡΟΣτΙΚΟΣ ΛΟΓΙΣΜΟΣ». Η γλώσσα αυτού του κειμένου είναι η καθαρεύουσα· εμείς, όμως, θα το αποδώσουμε στη Δημοτική, γιατί έτσι θα γίνει κατανοητό. Το κείμενο « . . . Όπως τονίσαμε και στην πρώτη έκδοση τον πυρήνα της Μαθηματικής Ανάλυσης αποτελούν οι έννοιες της παραγώγου (διαφορικού), και του Ολοκληρώματος, οι οποίες εισήχθησαν στα Μαθηματικά το πρώτο κατά τον 1 7° αιώνα από τους Leibniz και Newton με τη βοήθεια της θεωρίας των aπειροστών μεγεθών. Η θεωρία όμως των aπειροστών μεγεθών των ανωτέρω μνημονευθέντων ερευνητών και γενικά οι μέθοδοι αυτών, θεμελίωσης του διαφορικού και του ολοκληρώματος, δεν διακρίνονταν για την ακρίβεια και αυστηρότητα, την οποία απαιτούν οι μαθηματικές θεωρίες και το πνεύμα των δημιουργών της μαθηματικής επιστήμης Ελλήνων μαθηματικών της αρχαιότητας. Ήσαν βεβαρυμμένες με εξωμαθηματικές έννοιες και λογικά δεν είχαν επαρκή δικαιολόγηση . Τα επιτεύγματα όμως των νέων θεωριών κατά τις εφαρμογές τους στη Μηχανική και τη Γεωμετρία aποστόμωσαν την εναντίον τους θεωρητική κριτική. Κατά τους δύο αιώνες που ακολούθησαν ο Ι β. "Ευκλείδεια Γεωμετρία, αγάπη μου "

κλάδος αυτός των Μαθηματικών εξελίσσεται και συμπληρώνεται με νέες γνώσεις, παίρνει από τον Eϋler τον γενικότερο τίτλο <<Ανάλυση», συγκεντρώνει δε και απασχολεί σαν ερευνητές του τις μεγαλύτερες μαθηματικές διάνοιες της εποχής εκείνης, όπως τους Eϋler, Lagrange, Fourier, Gauss και άλλους. Εξακολουθεί όμως να παραμένει βεβαρυμμένος με τη θεωρητική κριτική, όσον αφορά τη σταθερότητα των βασικών εννοιών. Ο Γάλλος μαθηματικός Α. Cauchy κατά το πρώτο μισό του 1 8ου αιώνα και αργότερα οι Weierstrass, Peano, Cantor, Dedekind και άλλοι πέτυχαν με αυστηρή κριτική, να θεμελιώσουν τη θεωρία των πραγματικών αριθμών και να διατυπώσουν αυστηρά λογικά τις έννοιες της Ανάλυσης. Έτσι ο κλάδος της Μαθηματικής Ανάλυσης, περί το τέλος του 1 9ου αιώνα πήρε ισάξια θέση κοντά στις μαθηματικές θεωρίες της αρχαίας Ελληνικής περιόδου . . . » [πηγή: Δημ. Κάππου, «Απειροστικός Λογισμός», αυτοέκδοση,

Αθήνα 1962]

Προλεγόμενα . Δεν πρόλαβε να κυκλοφορήσει το δεύτερο τεύχος του περιοδικού μας κι αμέσως λάβαμε δύο ηλεκτρονικά μηνύματα με τα οποία μας δήλωναν την αντίθεσή τους σε κάθε αλλαγή του χαρακτήρα της τοποθεσίας "Αυτό το ξέρατε; ". Ο ένας απ' αυτούς μας προτείνει τη δημιουργία μιας νέας τοποθεσίας. Εμείς δεχθήκαμε την πρόταση και τη νέα τοποθεσία την ονομάζουμε "Ευκλείδεια Γεωμετρία, αγάπη μου ". Στο τεύχος αυτό θα ασχοληθούμε με:

• την "Ευθεία του Newton (ή του Gauss) " • το ''Ισογώνιο σημείο τριγώνου "

• τον "Κύκλο του Fuhrmann "

•Ευθεία το υ Newton (ιί το υ Gαιι,\'.\) (σχήμα Ι) Τα μέσα των τριών διαγωνίων πλήρους

τετραπλεύρου βρίσκονται στην ίδια ευθεία (ε). Η (ε) ονομάζεται ,ευθεία του Newton (ή του

Gauss) '' Σημείωση: Ορισμός πλήρους τετραπλεύρου:

Δίνεται ένα κυρτό τετράπλευρο ΑΒΓ Δ, με τις απέναντι πλευρές του μη παράλληλες. Οι πλευρές ΑΔ, ΒΓ τέμνονται στο Ζ και οι ΑΒ, ΔΓ στο Ε. Το σχήμα που προκύπτει, ονομάζεται πλήρες τετράπλευρο ΑΒΓ Δ.ΕΖ. Διαγώνιοι του πλήρους τετραπλεύρου είναι τα ευθύγραμμα τμήματα ΑΓ,

ΕΥΚΛΕΙΔΗΣ Β' 83 τ.3/27

Page 30: Ευκλειδης Β 83

------------- ΗΟΜΟ MATHEMAτiCUS ------------­ΒΔ, ΕΖ και τα Κ,Λ,Μ τα αντίστοιχα μέσα τους. Τα ι (ε) είναι η ευθεία του Newton (ή του Gauss]

σημεία Κ,Λ,Μ βρίσκονται σε μια ευθεία (ε) . Η

Η ευθεία . . . . . είναι του Newton

(ή του Gauss) !

(σχήμα 1)

Το Κ είναι «ισογώνιο σημείο» του τριγώνου

(σχήμα 2)

� fσο;ι(u ι•ω σημ;;ίο τριι(ύ νου (σχήμα 2) Ονομάζεται ένα σημείο στο εσωτερικό του

τριγώνου το οποίο "βλέπει , τις τρεις πλευρές με την ίδια γωνία.

l:η μει(ί)σεις: 1) Η ύπαρξη αυτού του σημείου εξασφαλίζεται από το παρακάτω θεώρημα: «Πάνω στις πλευρές ενός τριγώνου κατασκευάζουμε ισόπλευρα τρίγωνα, τα οποία βρίσκονται και τα τρία εκτός του τριγώνου ή και τα τρία προς το μέρος κάθε πλευράς προς το

• Κι!ιιJ.ος nJ /) FΝfιπηωιη (σχήμα 3) Ονομάζεται ο κύκλος που γράφεται με διάμετρο

το ευθύγραμμο τμήμα με άκρα το ορθόκεντρο του τριγώνου και το "σημείο Nagel" αυτού.

Σημειώσεις: 1 . Τι είναι το σημείο Nagel τριγώνου; Αν σε ένα τρίγωνο συνδέσουμε κάθε κορυφή του με το σημείο επαφής της απέναντι πλευράς με τον αντίστοιχο παρεγγεγραμμένο κύκλο, αυτά τα τρία ευθύγραμμα τμήματα διέρχονται από το ίδιο

I ι "Α /JΠJ το ξ/ραπ.'; 1 1

Ο κύκλος (F,FH) είναι «κύκλος του Fuhrmann»

οποίο βρίσκεται η τρίτη κορυφή . Οι περιγεγραμμένοι κύκλοι περί τα τρία ισόπλευρα τρίγωνα διέρχονται από το ίδιο σημείο Κ. Το σημείο αυτό "βλέπει" τις τρεις πλευρές με την ίδια γωνία, δηλ. γωνΒΚΓ=γωνΓΚΑ=γωνΑΚΒ» 2) Οι περιγεγραμμένοι κύκλοι περί τα ισόπλευρα τρίγωνα ονομάσθηκαν από τον Neuberg "κύκλοι του Toricelli"

σημείο, που το ονομάζουμε σημείο Nagel του τριγώνου 2. Τι είναι παρεγγεγραμμένος κύκλος τριγώνου. Ο κύκλος που εφάπτεται σε μια πλευρά τριγώνου και στις προεκτάσεις των άλλων δύο πλευρών ονομάζεται παρεγγεγραμμένος κύκλος του τριγώνου αντίστοιχος της πλευράς αυτής. Είναι φανερό πως σε κάθε τρίγωνο υπάρχουν τρεις παρεγγεγραμμένοι κύκλοι

Ποιοι πήραν το βραβείο Fields Medal (γνωστό και σαν Nobel των Μαθηματικών) για το έτος 2010 ; [η απάντηση στο τέλος της στήλης]

! Η . "οι συνιγηάτες της σηjl.ηr.: ;ιράφυυν-ι:ρωτο ι! ν " Θ{;μα I 0 • r ωι την ί:'ο·νιΗα τηc εξίσωσης Π ρολr.γόμενα. Συνεχίζουμε με το δεύτερο μέρος όπως είχαμε προαναγγείλει στο προηγούμενο τεύχος. Υπενθυμίζουμε πως κάνουμε αποσπασματική παράθεση κειμένων από το βιβλίο του Sander Bais «01 ΕΞΙΣΏΣΕIΣ»

δε-ίJηφο μ{;ρος (η:λ::υτα �ο) • «Οι φυσικοί χρησιμοποιούν τα Μαθηματικά ως γλώσσα της Φύσης, μια γλώσσα η οποία έπρεπε να επεκτείνεται όποτε αποκαλύπτονταν νέα επίπεδα της φυσικής πραγματικότητας. Και γενικότερα, ο φυσικός επιστήμονας τείνει να χρησιμοποιεί τα Μαθηματικά ως γλώσσα . . . . . . »

• «Η λέξη εξίσωση (equatίon) προέρχεται από το λατινικό aequo (εξισώνω, στα ελληνικά) το οποίο παράγεται με τη σειρά του από το επίθετο aequus (ίσος)» • «Μολονότι οι εξισώσεις δεν χρησιμεύουν παρά για να αποδίδεται κάποια τιμή σε μια μεταβλητή, στο

ΕΥΚΛΕΙΔΗΣ Β ' 83 τ.3/28

Page 31: Ευκλειδης Β 83

------------- ΗΟΜΟ MATHEMAτiCUS -------------γενικό πλαίσιο της πραγμάτευσής μας εκφράζουν υπάρχουν παγκόσμιες σταθερές, όπως η ταχύτητα σχέσεις μεταξύ φυσικών μεταβλητών οι οποίες του φωτός, c, ή το φορτίο του ηλεκτρονίου, e · από χαρακτηρίζουν το εκάστοτε εξεταζόμενο φυσικό την άλλη, υπάρχουν άλλες παράμετροι που σύστημα και καθορίζουν τις επιτρεπτές μεταβολές λαμβάνουν σταθερές αλγεβρικές τιμές σε κάποιο τους στον χώρο και στον χρόνο . . . » δεδομένο πλαίσιο . . . » • «Η εύρεση των εξισώσεων προϋποθέτει γνώση του • «Αντί στις εξισώσεις να γράφουμε τις αριθμητικές συστήματος το οποίο προσπαθούμε να τιμές των παραμέτρων, τις παριστάνουμε κι αυτές με μοντελοποιήσουμε, και αυτό αποτελεί έργο κυρίως γράμματα. Έτσι, διακρίνουμε αρκετά σύνολα του φυσικού επιστήμονα · στην εξαγωγή των λύσεων, γραμμάτων στις εξισώσεις μας: μεταβλητές, όμως, ο μαθηματικός είναι απαραίτητος συνεργάτης» παγκόσμιες σταθερές και παραμέτρους του • «Ας στραφούμε τώρα σε μερικές από τις συστήματος. μαθηματικές συμβάσεις και σύμβολα που Ας προσθέσουμε τώρα και άλλα επίπεδα στην μεταχειρίζονται οι φυσικοί επιστήμονες. Συνηθίζεται έννοια των μεταβλητών. Οι μεταβλητές μπορεί να τα μεγέθη να παριστάνονται με γράμματα - η εξαρτώνται η μία από την άλλη, και σε αυτή την θερμοκρασία με τ, ο χρόνος με t, . . . - και τα εν λόγω περίπτωση τις αναφέρουμε ως συναρτήσεις ή μεγέθη μπορούν να λαμβάνουν διάφορες τιμές, εξ ου πεδία . . . »

και ονομάζονται μεταβλητές. Στις εξισώσεις [πηγή: Sander Bais «01 ΕΞΙΣΩΣΕΙΣ», εκδ. «ΚΑ ΤΟΠΤΡΟ», εμφανίζονται και ορισμένες σταθερές: από τη μία , Αθήνα 2009]

Θ{;μα 2" f:πέτεω ι μεγάλων μαθη μαηκιον

Π ρ ολεγόμενα . Ο φίλος της στήλης Δημητράκης Κώστας [Χανιά] , μας έστειλε τον παρακάτω πίνακα, στον οποίο σημειώνονται οι επέτειοι από την ημέρα των γενεθλίων ή την ημέρα του θανάτου, τεσσάρων μεγάλων μαθηματικών κατά το μήνα Μάρτη .

rJ ίνωω-; με μιρ ικές εππείους το ΙVi iφτη T(JQJ 20 ι 2 α.α. ΟΝΟΜΑΤΕΠΩΝΥΜΟ Έτος Έτος Μέχρι το Μάρτη 2012

γέννησης θανάτου συμπληρώνονται: 1 Rene Descartes 3 1 /3/ 1 596 � 416 έτη από τη γέννησή του 2 Isaac Newton . . . . . . . . . . 20/3/ 1 727 � 285 έτη από το θάνατό του 3 Pieπe-Simon Laplace 23/3/ 1 749 5/3/ 1 827 � 263 έτη από τη γέννησή του

� 185 έτη από τον θάνατό του 4 Joseph Fourier 2 1 /3/ 1 768 � 244 έτη από τη γέννησή του

[πηγή : «01 ΜΑΘΗΜΑτΙΚΟΙ», ΕΤ BELL, πανεπιστημιακές εκδόσεις Κρήτης, τ. Ι, 1 995] Θi:μα J". Π έτρου Σοφ ιανο ύ : f:φαρμογi:ς π(lνω στην επ:':κταση της αρχα ίας Λφπεδ6νης. Π ρολεγό μενα . Ο φίλος της στήλης και ειδικός στον Πυθαγόρα, Πέτρος Σοφιανός, μας έστειλε ένα σημείωμά του πάνω στο θέμα της «αρχαίας αρπεδόνης». Επειδή η στήλη μας πάσχει από έλλειψη χώρου και επειδή ο όγκος του υλικού που αναμένει δημοσίευση είναι μεγάλος, θα το δημοσιεύσουμε σε δύο συνέχειες. n ρ (iJτη συνf:ι.r: ια

((Σε 12 ίσα τμήματα χώρισαν το κορδόνι

απ ' την αρχαία Αίγυπτο και το 'παν

((αρπεδόνη>ι!

«Αρπεδονάπτης» λέγονταν ο απτόμενος του νήματος και δάσκαλος

μπορεί με αυτό Γεωμετρίας σχήματος . . . » Κατά τον Ηρόδοτο βέβαια οι αρπεδονάπτες

ήσαν εκείνοι οι τεχνίτες της Γεωμετρίας που μετρούσαν τα χωράφια των αγροτών στις όχθες του Νείλου υπολογίζοντας και τους φόρους που αντιστοιχούσαν για τους Φαραώ .

((Με 12 ίσα τμήματα αν κλείσεις το κορδόνι

πλέον σήμερα θεώρημα του Πυθαγόρα το τρίγωνο με μήκη πλευρών 3 ,4 , 5 πρέπει να είναι ορθογώνιο αφού οι πλευρές του αυτές ικανοποιούν τη σχέση : 32 + 42 = 52 .Εκτός όμως του ορθογώνιου σκαληνού (ετυμολογικά: χωλού ανισοσκελούς), κατασκευάζουμε: � Ισόπλευρο-ισογώνιο (με πλευρά 1 2 : 3 = 4) � ισοσκελές- οξυγώνιο (με βάση 2 και σκέλη ίσα με 5) � Τρίγωνα μ' αυτή φτιάχνεις, τετράπλευρα (είδους παντός), πολύγωνα και σχήματα πολλά

κι έτσι . . . πόσα άραγε; γωνία ορθή ποιείς λέγε το ((αρπεδόνφι " ! με σταθερή περίμετρο τους κόμπους καθώς Πράγματι αφού 1 2 =3+4+5 από το γνωστό πιάνεις , βλέποντας πως αλλάζουνε σε τούτα τα

ΕΥΚΛΕΙΔΗΣ Β' 83 τ.3/29

Page 32: Ευκλειδης Β 83

------------- ΗΟΜΟ MATHEMAτiCUS -------------εμβαδά !

Το παιχνίδι αυτό ( η κλειστή αρπεδόνη με περίμετρο 1 2 ) «παίζεται» από δύο τουλάχιστον έως έξη το πολύ άτομα που μπορούν να κατασκευάζουν όλα τα είδη των γεωμετρικών σχημάτων , να υπολογίζουν και να συγκρίνουν μεταξύ τους τα εμβαδά καταλήγοντας σε προτάσεις όπως π. χ. οι παρακάτω : � Από όλα τα παραλληλόγραμμα που έχουν

σταθερή περίμετρο 1 2 και την ίδια βάση, το μέγιστο εμβαδόν έχει το ορθογώνιο παραλληλόγραμμο, αφού σε τούτο μεγιστοποιείται το ύψος. � Από όλα τα ορθογώνια που έχουν σταθερή

περίμετρο η σταθερό άθροισμα διαστάσεων, το μέγιστο εμβαδό έχει το τετράγωνο.

Η πρόταση αυτή μπορεί να δειχθεί γενικά και με μηδενισμό της πρώτης παραγώγου του Εμβαδού σαν συνάρτησης αρχικά δύο μεταβλητών Ε = χ y αλλά αφού x+y = 6 ή y = 6 - χ έχουμε Ε(χ) =χ (6-χ) = 6χ - χ2 οπότε Ε ' (χ) = 6 - 2χ και τέλος λύοντας την εξίσωση Ε '(χ) = Ο προκύπτει το χ = 3 και άρα y = 3 .

Στην περίπτωση της κλειστής αρπεδόνης καταλήγει κανείς στο παραπάνω συμπέρασμα μετρώντας απλά και συγκρίνοντας τα εμβαδά των τριών ορθογωνίων με σταθερή περίμετρο 1 2 ( l x5 , 2χ4, 3χ3) βλέποντας ότι το μεγαλύτερο εμβαδόν έχει το τετράγωνο.

Θέμα 4° Ενός φίλος της στήλης με τα . . . «Μ αθη ματικά στην κουζίνα»

Με μεγάλη μας χαρά πληροφορηθήκαμε πως ο φίλος της στήλης, Δημ. Χασάπης, έγραψε ένα νέο βιβλίο με τίτλο «"ΜΑΘΗΜΑτΙΚΆ ΣΤΗΝ ΚΟΥΖΙΝΑ" - Μαθηματικές δραστηριότητες για παιδιά και τους γονείς τους» [εκδόσεις ΜΕΤΑΙΧΜΙΟ, Αθήνα 20 1 1 ] . Το βιβλίο έφτασε στα χέρια μας και το διαβάσαμε. Είναι προικισμένο με χάρη και ευρηματικότητα, και προορίζεται για παιδιά του Δημοτικού.

Θέμα 5". Στη χιi)ρα του Ευκλείδη

Στο ηλεκτρονικό περιοδικό «Journal of classical Geometry» [ Volume 1 (20 1 2) ] διαβάσαμε την παρακάτω είδηση : <<Nguyen Mίnh Ηα, Α proof of Vittas ' Theorem and its conνerse, pp. 32-39» . πράγμα που σημαίνει πως αναγνωρίζεται, στον Έλληνα αρχιτέκτονα και εραστή της Γεωμετρίας, Κώστα Βήττα η πατρότητα ενός θεωρήματος που, διεθνώς, θα φέρει το όνομα «Θεώρημα του Βήπα». Είμαστε υπερήφανοι. Αυτή η μεγάλη επιτυχία είναι η φωνή της συνείδησης, που εγκαταλείψαμε ουσιαστικά την Ευκλείδεια Γεωμετρία στη χώρα του Ευκλείδη .

Θέμα 6° Εκδήλωση για την Ευκλείδεια Γεωμετρία

Η εκδήλωση για τη Γεωμετρία [του περιοδικού «Φ», του Βασίλη Βισκαδουράκη] πραγματοποιήθηκε στο Ευγενίδειο Ίδρυμα και διήρκεσε περισσότερο από τέσσαρες ώρες, σε ένα αμφιθέατρο κατάμεστο από εραστές της Γεωμετρίας. Ήταν τέσσαρες μαγικές ώρες στη διάρκεια των οποίων ανέπτυξαν την άποψή τους ο Δημήτρης Χριστοδούλου (καθηγητής στο Πολυτεχνείο της Ζυρίχης) και ο Αντώνης Μελάς (καθηγητής στο πανεπιστήμιο Αθήνας), άριστοι κάτοχοι του θέματος και οι δυο τους, μας ξενάγησαν στην ιστορία και σε περιστατικά της Γεωμετρίας, τόσο απλά και όμορφα που είμαι σίγουρος πως κι ένας διαβασμένος μαθητής θα τα καταλάβαινε.

1 1 11 • Η απάντηση στο : " Αυτό το ξέρατε ; "

3/9!1970 Ρωσία

Petersburg �State UniversityCaltech

ΕΥΚΛΕΙΔΗΣ Β' 83 τ.3/30

της

Page 33: Ευκλειδης Β 83

------------ Μαθηματικά για τη Α' Λυκείου ------------

ΑΛΓΕΒΡΑ Α I ΛΥΚΕΙΟΥ

ΣΥΝΑΡΤΗΣΕΙΣ

Άσκηση t η Να βρείτε τα πεδία ορισμού των παρακάτω

συναρτήσεων:

.J25 - 4x2 α) r με r<χ> = ι

I 2χ - 3 - 6

β) g με g(x) = {/χ2 - 3χ + 2 - �l2x - 5l - 4 γ) h με h(x) = Jixi + .Jx2 + 1 - �-x + .Jx2 + 1

Λύση :

{ 25-4xz ;;::: o } f(x) ε IR ς:> l2x-3l-6:;t:O ς:>

{-4χ2 �-25 } {χ2 � 25 } ς:> 12x-�:;t:6 ς:>

2χ -3:6 και 2x -3:;t: -6 ς:>

1 1 - 2 ζ:? 9 3 ς:> { χ 2 < (�)2 } { J χ Ι � � } χ:;t:-καιχ:;t:--

2χ :;t:9και 2χ:;t: -3 2 2

ς:>{-1��� }ς:> χ ε [-� ,-�)υ (-� ,�]

x:;t:2 και χ :;t:-� 2 2 2 2 2 2

Επομένως πεδίο ορισμού της f είναι το D = [-� -�) υ (-� �] f 2 ' 2 2 ' 2

β) g(x) =ε IR ς:> ς:> { χ2-3χ+2� } J2x-5J ;;:::4

{ xg ή χ� } !xg ή χ�

I 2χ-5<ο4 ή 2χ-5,;-4 <ο> χ<ο� ή χ,;� .

Επομένως πεδίο ορισμού της g είναι:

D = (-οο _!_] υ [2. +οο) . g ' 2 2 '

Γιάννης Κυριαζής

γ) Επειδή i +l>Ο, για κάθε χ ε ΙR άρα Ji+l >Ο και lx l � Ο , επομένως lx l + � > Ο για κάθε χ ε IR . Επίσης χ 2 + 1 > χ 2 άρα και Jx2 +l >N ς:> Jx2 + l > lxl � x και τελικά -x+Ji +1 >0 για κάθε χ ε IR . Άρα Dg = IR . Άσκηση 2'1

Δίνεται η συνάρτηση f με f(x)= �2χ+λ

, λεΙR χ-1

α)Να βρεθεί το πεδίο ορισμού της f

β)Να βρείτε το λ ε ΙR ώστε f(3) = � . 2

γ)Να υπολογισθούν οι τιμές r(%) και f ( f(l l)) .

Λύση :

{ 2χ+3;;::: Ο } - 2 ! χ>_]_ I α) f(x) ε IR ς:> x-l:;t:O ς:> x:;t: l ς:>

χ ε [-% , ι) υ (l, +οο) . Άρα Dr =[ -% , ι)υ(Ι,+οο) . 3 .J2 . 3 + λ 3 r;::-:-;

β) f(3) = - ς:> = - ς:> νλ + 6 = 3 . Στο 2 3 - 1 2 σύνολο ορισμού της Α = [ -6, +οο) έχουμε:

Jλ+6=3��λ+6)2 =Υ ς:>λ= 3 . Δεκτή αφού 3 ε Α .

γ) f (�) = * = J6 = 2J6 . 2 l _ ι _!_

2 2

f(l l) = .J2 · 1 1 + 3 = 2_ = _!_ . 1 1 - 1 1 0 2

( ) f-F· f(f(l l)) = f _!_ = 2 =�= -4 . 2 _!_ _ 1 _ _!_

2 2 ΕΥΚΛΕΙΔΗΣ Β ' 83 τ.3/31

Page 34: Ευκλειδης Β 83

------------- Μαθηματικά για τη Β ' Λυκείου -------------

Λ σ uςη ση 3 11 Δίνεται η συνάρτηση f : JR � JR για την ο­

ποία ισχύει: f(x) + 4χ � χ2 + 3 � f(x + 2) + 4 για κάθε χ ε JR . α) Να βρείτε τον τύπο της f .

β) Να μελετηθεί η f ως προς την μονοτο­νία και τα ακρότατα.

Λ (Jση : Από την υπόθεση έχουμε:

f(x) + 4x � x2 + 3 => f(x) � x2 + 3 - 4χ ( 1 ). για κάθε χ Ε JR . Επιπλέον χ2 + 3 � f(x + 2) + 4 =>

=> f(x + 2) ;;:: x 2 + 3 - 4 => f(x + 2) ;;:: x 2 - 1 (2). Θέτουμε όπου χ το χ - 2 οπότε έχουμε:

f(x - 2 + 2) ;;:: (x - 2)2 - 1 => f(x) ;;:: x 2 + 3 - 4χ (3)

για κάθε χ Ε JR . Από (1 ) και (3) βρίσκουμε:

f(x) = χ2 - 4χ + 3 , (αντισυμμετριιcή ιδιότητα) .

β ) Στο τριώνυμο f(x) = x2 - 4x + 3 έχουμε

α = 1 > Ο β = -4 γ = 3 -β = - -4 = 2 και ' ' ' 2α 2 - 1 -� = (-4)2 - 4 - 1 - 3 = -i = -1 4α 4 - 1 4 ·

Επομένως η f είναι γνησίως φθίνουσα στο διάστημα

( -οο, 2] και γνησίως αύξουσα στο [ 2, +οο) . Παρουσιάζει

ολικό ελάχιστο στο χ0 = 2 που ισούται με f(2) = - 1 , ό­

πως φαίνεται στον παρακάτω πίνακα μεταβολών:

χ -οο +2 +οο �- ---- -Ι--- -�-- --- - -----·---- ----- ---� --

-1 / f(χ)

Ολικό Ελάχισιο

4 η Δίνεται η συνάρτηση f : Α � JR με Α -φ. JR

χ2 - 2 και f(x) = ι:;ι:;ι , λ Ε JR . xz - 2ν 2 1λ l · χ + λ2 + 1

Να δείξετε ότι lλ l = 1 .

jβ ) Ν α μελετήσετε την f ως προς την μονοτονία και τα ακρότατα.

γ) Να βρείτε τα σημεία τομής της Cr με τους άξονες. Λ (Jση : Παρατηρούμε ότι:

f(x) E JR <:::;> χ2 - 2[2ίλί - χ + λ2 + 1 -::F- 0 . Το α' μέλος είναι τριώνυμο με α = 1 > Ο ,

β = -2{2ίλί και γ = λ2 + 1 . Δ=( -ψj�γ -4Κ +1) = 8 lλl - 4λ2 - 4 = -4( lλl2 - 2 lλl + 1 ) = -4( lλl - 1)2 .

Αν �� -::F-1 τότε Δ <Ο , οπότε i -�·χ+Κ +1>0, για κάθε χ Ε R αφού α = 1 > Ο . Συνεπώς θα είναι και

χ2 - 2{2ίλί - χ + λ2 + 1 -::F- Ο για κάθε χ Ε JR . Άρα Dr = Α = JR , άτοπο, αφού Α :;t: JR ,άρα

lλl = 1 , δηλαδή λ Ε { -1 , 1 } . β) Για lλl = 1 ο τύπος της f γίνεται

f(x) = χ2 - 2 χ2 - ( ,J2γ x2 - 2J2-i - x + 12 + 1 x 2 - 2xJ2 + (J2)2

= ( x + Ji)(x - Ji) ( x - J2)2

x + Ji r;:; ' με X - '-,J L.

A = Dr = 1R - {J2} = (-oo·,J2) υ (J2,+oo) Επιπλέον τροποποιούμε τον τύπο της f και έχουμε

f(x) = x + Ji = x - J2 + 2J2 = 1 + 2J2 x -Ji x - Ji x - Ji = 1 + :� .όπου g ( x ) = x -Ji

Αλλά: g! ( -oo,J2) και g! ( Ji,+oo) . Επομένως

1 η - και συνεπώς η f θα είναι γνησίως φθίνουσες στα g διαστήματα ( -οο, J2) και ( J2, +οο) . Όμως

f(0 ) = 1 - 2 = - 1, και (2J2) = 1 + 2 = 3 , δηλαδή

f ( Ο) < f ( 2J2) , με Ο < 2J2, πράγμα που αναιρεί την

έννοια της φθίνουσας συνάρτησης. Άρα η f δέν είναι μονότονη στο Dr . Μέγιστα και ελάχιστα δεν υπάρχουν. γ) Για χ=Ο παίρνουμε y=-1

Άρα το κοινό σημείο της Cr και του άξονα y ' y είναι το Α(Ο, - 1) . Οι συντεταγμένες των σημείων το­

μής της cf με τον άξονα χ ' χ επαληθεύουν το σύστη-

μα {: : ; : �} , δηλαδή τελικά { �=�JZ} . Άρα έχουν ένα μόνο κοινό σημείο το Β ( -J2, Ο) . Ασκη ση 5'1 Θεωρούμε δύο συναρτήσεις f, g : R � JR για

τις οποίες ισχύει: f(x) = g2 (x) - 6g(x) + 1 1 για κά­θε χ ε JR . Αν η γραφική παράσταση Cg της g τέ­μνει την ευθεία y = 3 , να δείξετε ότι η f παρουσιά­ζει ολικό ελάχιστο, το οποίο να υπολογίσετε.

ΛίJση : Έχουμε: f(x) = g2 (χ) - 6g(x) + 1 1 = = g2 (x) - 2g(x) - 3 + 32 + 2 = [g(x) - 3]2 + 2 ;::: 2

ΕΥΚΛΕΙΔΗΣ Β ' 83 τ.3/32

Page 35: Ευκλειδης Β 83

------------- Μαθηματικά για τη Β' Λυκείου -------------

για κάθε χ Ε R . Επιπλέον η cg τέμνει την ευθεία

y = 3 . Άρα υπάρχει χ0 Ε R τέτοιο ώστε g ( χ0 ) = 3 , δηλαδή f ( χ0 ) = [g(x0 ) - 3Υ + 2 = 2 . Επομένως

f (χ) � f ( χ0 ) = 2 για κάθε χ Ε R , δηλαδή η f παρου­

σιάζει ολικό ελάχιστο στο Χο ER με την τιμή f(Xo) =2 . Άσκηση 6" Έστω η συνάρτηση f : R � R , τέτοια ώστε

3r[f(x)+f(-x)+2r]= �f(-x), για κάθε χ ε JR . α )Να

δειχθεί η f είναι περιττή. β) Να βρεθεί ο τύπος της f.

Λ ί1ση : Προφανώς για κάθε xER ισχύει (-χ) ER.

α) Στην: 3x5 [f(x) + f(-x) + 2x2 ] = -% f(-x) ( 1 ), θέτουμε όπου χ το -χ και έχουμε:

3( -χ)5 [ f( -χ) + f(x) + 2( -χ)2 J = -% f(x) δηλαδή

-3χ5 [ f( -χ) + f(x) + 2χ 2 J = -� f(x) (2)

Προσθέτοντας τις σχέσεις ( l ) και (2) κατά μέλη πέρ­

νουμε: [f(x) + f(-x) + 2x2 } (3x5 - 3χ5 ) = 2 2 2 2 = --f( -χ) --f(x) ::::> Ο = --f(-x) - -f(x) 3 3 3 3

2 2 ::::>- f(-x) =-- f(x) ::::> f(-x) = -f(x) για κάθε χ Ε R . 3 3 Άρα η f είναι περιττή. β ) Από την σχέση ( 1 ) αφού η f είναι περιττή παίρνου-

με : 3χ5 [ f(x) - f(x) + 2χ 2 J = � f(x) ::::>

7 2 7 6χ = -f(x) ::::> f(x) = 9χ , για κάθε χ Ε R . 3 :\σκηση 7' 1 Θεωρούμε τη συνάρτηση f(x)=(λ-2)i +�-10.

α) Να βρεθεί ο αριθμός λ ε R ώστε η γραφική της παράσταση Cf να διέρχεται από το Α(1, 4) . β) Να μελετηθεί η f ως προς την μονοτονία. γ) Να επιλυθεί η ανίσωση r[r(x)-6r -s]-4>0 (1).

Λ\Jση : α) Η Cf διέρχεται από το σημείο A(L4)<=>f(1) =4

<=> (λ - 2) · 1 3 + 3λ · 1 - 1 0 = 4 <=> λ - 2 + 3λ - 1 0 = 4 <=> 4λ = 1 6 <=> λ = 4 . β) Για λ = 4 έχουμε: f(x) = 2χ3 + 1 2χ - 1 0 ,

ΊΙD ·ο 3 3 * 2 3 2 3 Χ Ε m.. . μως Χ 1 < Χ2 ::::? Χ ι < Χ2 ::::? Χ 1 < Χ2 , 1 2χ ι < 1 2χ 2 => 2χ/ + 12χι - 10 < 2χ/ + 12χ2 - 10::::> ::::> f(x 1 ) < f(x2 ) . Επομένως η f είναι γνησίως αύ­ξουσα στο R .

γ) f [ f(x) - 6χ2 - 5 J > 4 <=> f[ f(x) - 6χ2 - 5 J > f(l)

και επειδi1 η f είναι γνησίως αύξουσα θα έχουμε: ( 1 ) <=> f(x) - 6x2 - 5 > 1 <=> 2χ3 + 1 2χ - 1 0 - 6χ2 - 5 - 1 > 0 <=> χ3 - 3χ2 + 6χ - 8 > ο <=> ( χ3 - 23 ) - 3χ(χ - 2) > ο (χ - 2)(χ2 + 2χ + 4) - 3χ(χ - 2) > ο <=> (χ - 2)(χ2 - χ + 4) > ο <=> χ - 2 > ο <=> χ > 2 , αφού χ 2 - χ + 4 > Ο για κάθε χ Ε R , ( α = 1 > 0 και Δ = (-1)2 - 4 · 1 · 4 = - 1 5 < 0 ) .

Δίνεται η συνάρτηση f : JR�JR με f(x)= l�3=2 ·

υ) Να εξετασθεί η f ως προς την μονοτονία. ' ' r -3x+6 r -3x+8 ) ! Να επιλυθει η ανισωση r > r (Ι · +3 +5

α) Αν χ � Ο , τότε f(x) =�=3-__i_ και αν χ+2 χ+2 ' 3χ 3χ 6 • • χ s Ο , τοτε f(x) =-- = ---= -3 ----χ+ 2 χ - 2 χ - 2

1 1 Επομένως χ, � :5:0=>χ, -2<Χ:ι -2<-2=>------ >-=>

χ, -2 Χ:ι -2 6 6 6 6

=> --- < ---=>-3--- < -3---xl -2 χ2 -2 Χι -2 χ2 -2

Ομοίως: Ο s Χ ι < χ2 => 2 < χ 1 + 2 < χ2 + 2 => . . . => f(χ ι ) < f(x2 ) => ft ( -οο, Ο] => f t [ Ο , +οο)

Λ Λ

Αφού το άκρο Χο =0 περιλαμβάνεται και στα δύο δι­

αστήματα μονοτονίας θα έχουμε: f��ψ.-{O,�=JR

!) ) Είναι χ2 - 3χ + 6 > 0 για κάθε χ Ε R , διότι α = 1 > 0 και Δ = (-3)2 - 4 · 1 · 6 = - 1 5 < 0

και χ2 -3χ +8 = (χ2 -3χ + 6) + 2 > 0 για κάθε x E JR

Ά ( Ι ) χ2 - 3χ + 6 χ2 + 3

ρα <=> > -- <=> χ2 - 3χ + 8 χ2 + 5 3 ( χ 2 - 3χ + 6) 3 ( χ2 + 3) <=> > <=> Ιχ2 - 3χ + 61 + 2 Ι χ2 + 31 + 2

f( χ2 - 3χ + 6) > f(x2 + 3) <=>χ2 -3χ+6 > χ2 +3<=> -3χ > 3 - 6 <=:> -3χ > -3 <=> χ < 1 , αφού η f είναι γνησίως αύξουσα.

( )3 ( )3 3 3 3 3 => - Χ ι > -χ2 => -Χ ι > -Xz => Χ ι < Xz και

Χι :5: Ο < χ2 => Χ ι3 < Ο < χ/ •• Οι μορφές αυτές μπορούν να προκύψουν άμεσα από τις διαιρέσεις 3y:(y+2) και -3y:(y-2), όπου y = l x l

ΕΥΚΛΕΙΔΗΣ Β ' 83 τ.3/33

Page 36: Ευκλειδης Β 83

------------- Μαθηματικά για τη Α ' Λυκείου -------------

Α ' ΛΥΚΕΙΟΥ Εγγεγραμμένα Σχήματα - Αναλογί ες - Ομο ιότητα

Ένα τετράπλευρο ΑΒΓ Δ είναι εγγεγραμμένο σε κύκλο. Αν Ε, Ζ, Η, Θ είναι οι προβολές του τυχαίου σημείου Μ του κύκλου πάνω στις ΑΒ, ΒΓ, ΓΔ, ΔΑ αντίστοιχα, να δειχτεί ότι: ΜΕ · ΜΗ=ΜΖ · ΜΘ.

Α

Α , δ 'ξ , ΜΕ ΜΘ , , ρκει να ει ου με οτι: - = -- η οτι τα ΜΖ ΜΗ

τρίγωνα ΜΕΖ και ΜΗΘ είναι όμοια. Είναι

Δ Θ Μ=Μ Η Δ= l L οπότε ΜΘΔΗ εγγράψιμο άρα

Θ1 = Δ1 ( 1 ) . Ακόμη Δ1 = Β 1 (2) γιατί βαίνουν στο

ίδιο τόξο. Είναι Μ Ζ Β= Μ Ε Β= 1 L οπότε ΜΖΒΕ

εγγράψιμο, άρα Β1 = Ε1 (3) . Από ( 1 ), (2), (3) έχουμε

Θ1 = Ε1 ( 4 ) . Η Α Δ Γ είναι απέναντι εξωτερική της

Η Μ Θ στο εγγράψιμο τετράπλευρο ΜΘΔΗ, οπότε

Η Μ Θ =Α Δ Γ. Ακόμη στα εγγράψιμα τετράπλευρα

ΑΒΓ Δ και ΕΒΖΜ έχουμε Α Δ Γ= Β2 και

Β2 =Ε Μ Ζ, οπότε Η Μ Θ= Ε Μ Ζ (5). Από (4) και

(5) τα τρίγωνα ΜΕΖ και ΜΗΘ είναι όμοια. Δίνονται τρίγωνο ΑΒΓ εγγεγραμμένο σε

κύκλο, το aντιδιαμετρικό Α' του Α, και το ύψος του ΑΗ. Αν Μ, Ν οι προβολές των Β, Γ, στην Α Α' , τότε να δείξετε ότι: Α Η2 =ΑΜ · ΑΝ.

Α ' δ 'ξ ' ΑΗ ΑΝ δ λ δ ' ρκει να ει ου με οτι: -- = - , η α η

ΑΜ ΑΗ αρκεί τα τρίγωνα ΑΗΜ και ΑΝΗ είναι όμοια και

επειδή Αι κοινή αρκεί Νι = Η ι . Το τετράπλευρο ΑΗΝΓ είναι εγγράψιμο (η

πλευρά του ΑΓ φαίνεται από τις κορυφές Η και Ν

υπό ίσες γωνίες) � Νι = fΊ ( 1 ) . Το τετράπλευρο

ΑΒ Α' Γ είναι εγγεγραμμένο � fΊ = Λ; (2).

Πανδής Χρήστος

Είναι ΑΒ j_ Β Α' και ΒΜ j_ Α Α' � Λ; = Βι (3). Α

τ

Α ' Το τετράπλευρο ΑΒΗΜ είναι εγγράψιμο (η

πλευρά του ΑΒ φαίνεται από τις κορυφές Η και Μ

υπό ίσες γωνίες) � Β ι = Η ι (4) . Από τα

( 1 ),(2),(3 ),(4) προκύπτει ότι Νι = Ηι .

Ένας κύκλος (0) με σταθερό μήκος ακτίνας R εφάπτεται σε σταθερό σημείο Μ μιας σταθερής ευθείας (ε). Στα άκρα Α,Β μεταβλητής διαμέτρου αυτού φέρνουμε τις εφαπτόμενες που τέμνουν την (ε) στα Δ, Γ αντιστοίχως. Δείξτε ότι:_Ι\!Δ · ΜΓ=σταθ.

Επειδή Ο Μ Δ=Ο Α Δ=l L το τετράπλευρο ΟΑΔΜ είναι εγγράψιμμο. Αυτό έχει σαν συνέπεια

πώς 01 = Α1 ( l ) .H γωνία Β1 είναι εγγεγραμμένη

στον κύκλο (Ο) και βλέπει το τόξο ΑΜ , ενώ η

γωνία Α1 σχηματίζεται από την χορδή ΑΜ και την

εφαπτομένη ΑΔ. Άρα αυτές οι δύο γωνίες θα είναι

ίσες, δηλαδή Β, =Α (2). Επειδή Ο Β Γ =Ο Μ Γ= 1 L το τετράπλευρο ΟΒΓΜ θα είναι εyyράψιμμο, άρα θα

είναι: Β1 =fΊ (3). Από ( 1 ) , (2), (3) έχουμε ότι

<\ = fΊ , άρα τα τρίγωνα ΜΟΔ και ΜΟΓ είναι

όμοια, οπότε: ΜΔ · ΜΓ=Μ 02 ή ΜΔ · ΜΓ= R 2 • ο Ένα τετράπλευρο ΑΒΓ Δ είναι εγγεγραμμένο

σε κύκλο διαμέτρου ΑΓ. Αν Μ είναι το σημείο τομής των διαγωνίων του και Ε, Ζ οι προβολές του Μ πάνω στις ΑΒ, ΑΔ αντίστοιχα, να δείξετε

ΕΥΚΛΕΙΔΗΣ Β ' 83 τ.3/34

Page 37: Ευκλειδης Β 83

------------- Μαθηματικά για τη Α ' Λυκείου -------------

ότι ΕΖ I I ΔΒ. Δ

Α

, δ 'ξ , ΑΕ ΑΖ ( 1 ) Αρκει να ει ω οτι -=- . ΕΒ ΖΔ

Επειδή η Β είναι ορθή και η ΜΕ l_ ΑΒ είναι ΜΕ 1 1 ΓΒ, οπότε : ΑΕ = ΑΜ (2) . Επειδή η Δ είναι ΕΒ ΜΓ ορθή και ΜΖ l_ ΑΔ θα είναι ΜΖ/ /Γ Δ, οπότε είναι ΑΖ = ΑΜ (3) . Από (2), (3) συνεπάγεται η ( 1 ) . ΖΔ ΜΓ

Δίνεται τυχαίο τετράπλευρο ΑΒΓ Δ και Μ, Ν τα μέσα των πλευρών ΑΔ, ΒΓ αντίστοιχα. Η ευθεία ΜΝ τέμνει τις ευθείες ΑΒ και ΔΓ στα Ε,

'ξ , , ΕΒ ΖΓ

Ζ αντίστοιχα. Δει τε οτι ισχυει: - = - . ΕΑ

ΖΔ

Α

Μ Β

:'\ Ε Η �/Θ

Δ Γ Ζ Φέρνω τις ΒΗ, ΓΘ παράλληλες στην

ΑΔ. Τότε είναι ΒΗ 1 1 ΓΘ, οπότε Β ι = fΊ (ως εντός εναλλάξ). Είναι ακόμη Νι =Ν2 (ως κατακορυφήν) και ΝΒ=ΝΓ. Οπότε τα τρίγωνα ΝΒΗ και ΝΓΘ είναι ίσα. Άρα ΒΗ=ΓΘ. Τα τρίγωνα ΑΕΜ και ΒΕΗ είναι

όμοια οπότε ΕΒ = ΒΗ . Τα τρίγωνα ΔΖΜ και ΓΖΘ ΕΑ ΑΜ

, , , ΖΓ ΓΘ ειναι ομοια οποτε -= -- . ΖΔ ΔΜ

, ΕΒ ΖΓ Αλλά ΒΗ=ΓΘ και ΑΜ=ΔΜ θα ειναι -=-ΕΑ ΖΔ

Δίνονται δύο παράλληλες ευθείες ( ει ), ( ε2 ) και σταθερό σημείο Ο εκτός της ταινίας τους και προς το μέρος της ( ει ). Ένα σημείο Α γράφει την ( ε2 ) και η ΟΑ τέμνει την ( ει ) στο Β. Κάθε φορά θεωρούμε το ορθογώνιο τρίγωνο Α ΟΑΓ ( Α =1 L ) όπου ΑΓ=ΟΒ. Δείξτε ότι ΟΑ - =σταθ. ΟΓ

0

ν μ-'-( :.!!ι!.__----"'Β'--------- (ει ) Γ

-,ΝJ.,.Ι-----;----- (ε2)

Έστω Μ, Ν οι προβολές του Ο στις (ε ι ) , ( ε2 ) αντιστοίχως. Τότε από τα όμοια τρίγωνα ΟΜΒ και

, ΟΒ ΟΜ μ θ ( 1 ) 'Ε ΟΝΑ εχουμε : -=--=- =στα . . χουμε ΟΑ ΟΝ ν

ΟΒ=ΑΓ (2) . Από ( 1 ), (2) : ΑΓ = μ (3) . Είναι και ΟΑ ν

Γ Α 0= 1 L (4) . Οι (3), (4) σημαίνουν πως το τρίγωνο ΟΑΓ μένει όμοιο προς τον εαυτό του (Ομοιο δηλαδή στο σταθερό ορθογώνιο τρίγωνο με κάθετες πλευρές μ,ν) . Άρα ο λόγος δύο οποιονδήποτε γραμμικών στοιχείων μένει

σταθερός. Δηλαδή ΟΑ =σταθ. ΟΓ

Δίνεται παραλληλόγραμμο ΑΒΓ Δ. Μια ημιευθεία από το Α τέμνει την διαγώνιο ΔΒ στο Ε, την πλευρά ΒΓ στο Ζ και την ευθεία ΔΓ στο

, , 1 1 1 Η. Δειξτε οτι: - = - + - . ΑΕ ΑΖ ΑΗ

Α Β

Δ

ΑΕ ΑΕ Αρκεί να δείξουμε ότι: -+-= 1 ( 1 ). Τα ΑΖ ΑΗ

Β , , , FA ΕΔ τρίγωνα ΕΔΑ και Ε Ζ ειναι ομοια, οποτε: - = - , ΕΖ ΕΒ ΑΕ ΕΔ ΑΕ ΕΔ άρα: = και -=- (2).

ΑΕ+ΕΖ ΕΔ+ ΕΒ ΑΖ ΔΒ Επίσης τα ΕΒΑ και ΕΔΗ είναι όμοια, οπότε:

ΑΕ ffi ΑΕ ΕΒ ΑΕ ΕΒ -=- άρα: =-- και-=- (3). ΕΗ ΕΔ' ΑΕ+ΕΗ ΕΒ+ΕΔ ΑΗ ΔΒ Τις (2), (3) τις προσθέτουμε κατά μέλη και

έχουμε: ΑΕ + ΑΕ = ΕΔ + ΕΒ _ ΕΔ + ΕΒ = ΔΒ = 1 . ΑΖ ΑΗ ΔΒ ΔΒ ΔΒ ΔΒ

Δίνεται τραπέζιο ΑΒΓ Δ (ΑΒ 11 Γ Δ). Από το σημείο Ο της τομής των διαγωνίων του περνά ευθεία, παράλληλη στις βάσεις, που τέμνει τις ΑΔ, ΒΓ στην Ε, Ζ αντίστοιχα. Δείξτε ότι: ΟΕ=ΟΖ.

, , , ΔΕ ΓΖ Είναι AB I I EZ I I ΔΓ. Απο Θ. Θαλη ειναι -=­ΔΑ ΓΒ

ΕΥΚΛΕΙΔΗΣ Β' 83 τ.3/35

Page 38: Ευκλειδης Β 83

------------- Μαθηματικά για τη Α' Λυκείου -------------

( 1 ) . Ακόμη τα τρίγωνα ΔΑΒ και ΔΕΟ είναι όμοια , ΔΕ ΕΟ οποτε: -=- (2) .

ΔΑ ΑΒ Ar.o--------;o;

Δ Γ Επίσης τα τρίγωνα ΓΑΒ και ΓΟΖ είναι όμοια

οπότε: ΓΖ = ΟΖ (3) . Από ( 1 ) , (2), (3) έχουμε: ΓΒ ΑΒ

ΕΟ ΑΒ

ΟΖ ΑΒ

Στην τελευταία σχέση οι

παρανομαστές είναι ίσοι, άρα θα είναι και οι αριθμητές, δηλαδή ΟΕ=ΟΖ. 9. Δίνεται τραπέζιο ΑΒΓΔ (AB I I ΓΔ) και σημεία Ε,Ζ πάνω στις ΑΔ, ΒΓ αντιστοίχως. Α ν είναι ΕΖ Ι Ι ΔΓ και ΕΑ = μ , να υπολογιστεί το τμήμα ΕΔ ν ΕΖ συναρτήσει των βάσεων και των μ, ν.

Α Β

Δ Γ Απόδειξη : Έστω Η η τομή των ΕΖ και ΑΓ. Τότε θα έχουμε:

ΕΗ Ι Ι ΔΓ, οπότε τα τρίγωνα ΑΕΗ και ΑΔΓ είναι , , ΕΗ ΑΕ ( 1 ) Α , ' θ ομοια, αρα: -=- . πο την υπο εση

ΔΓ ΑΔ έχουμε: ΑΕ =� άρα ΑΕ =-μ- (2) . Από ( 1 ), (2)

ΕΔ ν ΑΔ μ + ν

έχουμε: ΕΗ =_μ_ οπότε ΕΗ= μ. ΔΓ (3 ). Επίσης

ΔΓ μ + ν μ + ν έχουμε: ΗΖ Ι Ι ΑΒ οπότε τα τρίγωνα ΓΗΖ και ΓΑΒ , , , ΗΖ ΓΖ (4) Α , , ' θ ειναι ομοια, αρα: -=- . πο την υπο εση

ΑΒ ΓΒ , ΖΒ ΕΑ μ , ΓΒ μ + ν , εχουμε: -=-=- αρα -= -- οποτε

ΖΓ ΕΔ ν ΓΖ ν ΓΖ = _ν_ (5) . Από (4), (5) έχουμε: ΗΖ =-ν- , ΓΒ μ + ν ΑΒ μ + ν

ή ΗΖ= ν · ΑΒ (6) . Από (3), (6) με πρόσθεση έχουμε: μ + ν

ΕΗ+ΗΖ= μ · ΔΓ + ν · ΑΒ ' ή ΕΖ

μ·ΔΓ +ν·ΑΒ μ + ν μ + ν μ+ν

t Ο . Δίνεται κύκλος (0), σταθερό σημείο Μ στο εσωτερικό του κύκλου και μια μεταβλητή χορδή ΑΒ που περνά από το Μ. Οι εφαπτομένες

του (Ο) στα Α, Β τέμνονται στο Γ. Αν Δ, Ε είναι οι προβολές του Μ πάνω στις ΒΓ, ΑΓ τότε θα ' 1 1 θ ει ναι: - +- =στα .

ΜΔ ΜΕ

Απόδειξη : Έστω Ζ το αντιδιαμετρικό του Α. Επειδή το

τρίγωνο ΑΒΓ είναι ισοσκελές θα ισχύει Α = Β . Επειδή ΖΑ .l Γ Α και ΖΒ .l ΑΒ θα είναι

Α Ζ Β= Ε Α Μ. Άρα τα ορθογώνια τρίγωνα ΕΑΜ, ΜΒΔ και ΖΑΒ είναι όμοια. Άρα θα έχουμε:

ΜΑ = ΜΒ ( 1 ) . Επίσης ΜΑ = ΖΑ (2) . Από ( 1 ) , (2) ΜΕ ΜΔ ΜΕ ΑΒ

, ΜΑ ΜΒ ΖΑ , μ λ 2R εχουμε: - = - = - η - = - = -- . ΜΕ ΜΔ ΑΒ ΜΕ ΜΔ μ + λ Από την τελευταία σχέση παίρνουμε τις

2R (3) και 1 2R (4)

ΜΕ μ (μ + λ ) ΜΔ λ (μ + λ) Γ

Θ ΜΑ ΜΒ ΖΑ Από ( 1 ) , (2) έχουμε: - = - = - ή ΜΕ ΜΔ ΑΒ

μ λ 2R A , λ ' ' ' - =- =- . πο την τε ευταια σχεση εχουμε : 1\ι:Ε 1\ιΒ μ+ λ

1 2R 1 2R , = ( )

(3), - =-λ( ) (4) Απο (3), (4) : ΜΕ μ μ + λ 1\ιΒ μ+λ _1_ +-1-= 2R + 2R =�( _!_+_!_] ΜΔ ΜΕ λ (μ + λ) μ ( μ + λ) μ + λ l λ μ 2R μ + λ , , 1 1 2R =--·-- η τελικα -+-=- (5) . μ + λ μ · λ ΜΔ ΜΕ μ · λ Τώρα φέρνω τη διάμετρο ΗΘ που περνά από το

Μ. Τα τρίγωνα ΜΑΗ και ΜΘΒ είναι όμοια επειδή Μ1 =Μ2 (ως κατακορυφήν) και Η ΑΜ=Μθ Β (βαίνουν στο ίδιο τόξο). Από την ομοιότητα:

ΜΑ = ΜΗ ή ΜΑ · ΜΒ=ΜΗ · ΜΘ ή ΜΑ · ΜΒ= ΜΘ ΜΒ =(R-MO)(R+MO) ή μ · λ= R2 -ΜΟ2 (6). Από (5), (6) :

1 1 2R δ λ , , ΜΟ

ΜΔ +ΜΕ

= R z - MOz

η . την αρχικη

, αφου

είναι η σταθερή απόσταση του Μ από το κέντρο Ο.

ΕΥΚΛΕΙΔΗΣ Β' 83 τ.3/36

Page 39: Ευκλειδης Β 83

Β ' ΛΥΚΕΙΟΥ ΑΣ ΚΗΣ ΕΙΣ ΑΛΓΕΒΡΑ Σ

Εκθετική - Λογαριθμική Συνάρτηση Χρήστος Λαζαρίδης

Το παρακάτω άρθρο αναφέρεται στις εκθετικές και λογαριθμικές συναρτήσεις και αποτελείται κυρίως, από επαναληπτικές ασκήσεις με αυξημένες απαιτήσεις.

Άσκηση 1

Έστω η συνάρτηση f(χ) = ( α - 2 )χ, χ ε R .

α - 1

α) Για ποιες τιμές του α η συνάρτηση είναι

γνησίως φθίνουσα.

β) Αν α > 2, να λυθεί η f(χΧα-1/ < (α-2/ .

Λύση

α)Γι' αυτό πρέπει και αρκεί: { ::� , δηλαδή 0<- <1

α-1 α:;t1 ( 1) α-2 ( 2) - < 1 α-1 α-2

(3) - >0 α-1

Με α :;t 1 έχουμε: α - 2 1 ( 2 ) � 1 - -- > Ο � -- > Ο � α > 1 οπότε α - 1 α - 1

(3 ) � α > 2 . Τελικά πρέπει και αρκεί α>2 .

β) Η aνίσωση έχει σύνολο ορισμού το R

οπότε : f(χΧα-1)χ2 <(α-2)χ2 �f(x) < (α-2)χ2 � α-1

(α) � f(x) < f(x2 ) �χ > χ2 �χ2 -χ < Ο <::::> O < x < l .

Άσκηση 2

Έστω η συνάρτηση f(x) = -1- , χ ε R .

2χ + 1 α) Να δειχθεί ότι: f(-x)+f(x) = 1, για κάθε χ�.

β) Να λυθεί: 9f(x) = f{ �7-4� ) + r( � - 2) (1).

γ) Να λυθεί η ανίσωση -1- -� + 4 < 0 (2).

f(2x) f(x)

Λύση

1 1 1 2χ α) f(x) +f(-x) = --+-- =--+-- = 2χ + 1 τ χ + 1 2χ + 1 2χ + 1

2χ + 1 -- = 1, για κάθε χ Ε R . 2χ + 1 β) Η εξίσωση έχει σύνολο ορισμού το R. Αρχικά παρατηρούμε ότι:

�7 - 4f3 = /!Γ + 22 - 2 -.fi · 2 =

= �( .J3 - 2 )2 = I.J3 - 2 1 = 2 - .fi. Άρα:

f ( � 7 - 4.J3 ) + f ( .J3 - 2) = f ( 2 -.J3) + f ( .J3 - 2 ) ι:; 1 οπότε : ( 1 ) � -9 - = 1 � 2 χ + 1 = 9 � 2χ = 8

2χ + 1 � γ = 23 � χ = 3 . γ) Η aνίσωση έχει σύνολο ορισμού το R, αφού f(x) :;t Ο , για κάθε χ Ε R , οπότε : ( 2 ) � 22χ + 1 - 3(2χ + 1) + 4 < ο

� 22χ - 3 · 2χ + 2 < 0 { y = 2x { y = Y � � �

y2 - 3 y + 2 < Ο 1 < Υ < 2

1 < 2χ < 2 � 2° < 2χ < 21 � ο < χ < 1 . Άσκη ση 3 Να λύσετε τις εξισώσεις:

α) αχ + α-χ = 2k (Ι ) , όπου 0 < α :;e 1, k ε R

παράμετροι. β) ex + e -χ = 3 (11). γ) 7χ + τχ + 1 = 3συν[lη(χ2 + 1)] (111).

Λύση α) Η εξίσωση έχει σύνολο ορισμού το R,

οπότε: αχ + α-χ = 2k � αχ + -1 = 2k � αχ

αχ = y } ( 1 ) ( ) . Για να έχει η (2) ρίζες

α2χ - 2kαχ + 1 = Ο 2

y Ε JR πρέπει και αρκεί Δ � Ο, όπου Δ=� κ" -1) ,

οπότε Δ2 0�κ ε (-οο,-1]υ[ 1,-tω) . Αν λοιπόν κ Ε ( --οο, -1] υ [ 1 , -too) , τότε οι ρίζες της y1 , y 2

ΕΥΚΛΕΙΔΗΣ Β ' 83 τ.3/37

Page 40: Ευκλειδης Β 83

------------- Μαθηματικά για την Β ' Λυκείου ------------είναι ομόσημες, αφού Ρ = y1 y2 = 1 > Ο .

Επομένως για να έχει λύση η ( 1 ) οφείλουν να είναι θετικές, δηλαδή S = y1 + y2 ::::: ο , η 2k > O

και τελικά k :::::: 1 . Ειδικά για k= 1 έχουμε: ( Ι ) <=> χ = 1οg

α 1 <=> χ = Ο

Από το προηγούμενο ερώτημα, για α = e

k 3 1 ' ' και = - :::::: , προκυπτει οτι: 2

( 11 ) <c> χ = ln ( Ξ ±�J <c> χ = ln ( 3 ±2J5}

γ) Έχουμε 3συν[lη(χ 2 + 1)] ::;; 3 με το ίσον μόνον όταν ln(x2 + 1) = Ο , δηλαδή χ=Ο. Δείξαμε στο πρώτο ερώτημα ότι για κάθε αριθμό χ που επαληθεύει την αχ + α-χ = 2k, ισχύει k :::::: 1 , δηλαδή αχ + α-χ :::::: 2 , με το ίσον μόνον όταν χ=Ο. Επομένως 7χ + Τχ + 1 :::::: 3 με το ίσον μόνον όταν χ=Ο. Άρα (ΠΙ ) <=> χ = Ο .

Έστω α, θ > Ο και α * 1 , θ * 1 , να αποδείξετε

ότι: θ = α logθ α

α log2 α·log3 α = 6 .

θ = α logα θ = α logo α = α logθ α

α log2 α·log3 α = α log2 α·log3 α α log2 α· log3 α

_Ι _ __ Ι _ ( α) = α log3 α α log2 α = 3 . 2 = 6 .

Έστω α > Ο ώστε α201 1 = 2012 , να

υπολογίσετε τις παραστάσεις: α α α log(α - -) + log(α - -) + · · · + log(α - --) 2 3 2012 α α α log(α + -) + log(α + -) + · · · + log(α + --) 2 3 2012

α α α 1og( α - -) + 1og( α - -) + · · · + 1og( α - --) =

2 3 20 1 2 α 2α 20 1 1α

1og- + 1og- + · · · + 1og-- = 2 3 20 1 2

1 1 - j · - · 20 1 1 20 1 1

1 1 20 1 2 1 1 ο og α = og -- = og =

j - 3 · · · 20 1 2 20 1 2

α α α log( α + -) + log( α + -) + · · · + 1og( α + --)

2 3 20 1 2 3α 4α 20 1 3α

log- + 1og- + · · · + log-- = 2 3 20 1 2

' . � . . . 20 1 3 20 1 1 20 1 3 . 20 1 2 1og α = 1og .

2 · ' · · · 20\2 2

Έστω O<k * 1 τέτοιος ώστε -1-� =e. Να

·�k hιk ln(IOe)

αποδείξετε ότι: k = e e

1 1 1 1 -- + -- = e => -- + - = e log k ln k ln k ln k

ln l O ln 1 0 1 => -- + -- = e => ln 1 0 + ln e = e ln k => ln k ln k

=> ln(l Oe) = ln ke => 1 Oe = ke

ke = l Oe => ln(ke ) = ln(l Oe)

ln(lOe) ln( l Oe) => e ln k = ln(l Oe) => ln k = -- => k = e e

e

'Εστω η συνάρτηση f(x) = (ημα)Χ , α Ε (0, π

) 2

Να αποδείξετε ότι η f είναι γνησίως

φθίνουσα στο R.

Να λύσετε την ανίσωση : (ημα/-χ < (ημα)2χ-z .

Ν α λύσετε την εξίσωση :

(συνα)1"

2 χ = (συνα)-ιη χ+z , α Ε ( Ο,�) .

α Ε (0, π ) => Ο < ημα < 1 , άρα η η f είναι 2

γνησίως φθίνουσα στο R. Η ανίσωση έχει σύνολο ορισμού το R, οπότε:

f(x2 - χ) < f(2x - 2) <=> χ2 - χ > 2χ - 2 <=> Ο

χ 2 - 3χ + 2 > Ο <=> χ ε (-οο, l) υ (2, +οο) .

Η συνάρτηση g(x) = (συναγ , χ Ε R αντίστοιχα, είναι γνησίως φθίνουσα στο R οπότε και 1 - 1 . Η εξίσωση έχει σύνολο ορισμού το (0, +οο) , οπότε :

(συνα) 1"2 χ = (συνα)- Ιη χ+2 <=>

ln 2 χ = - ln χ + 2 <=> ln 2 χ + ln χ - 2 = Ο <=>

ΕΥΚΛΕΙΔΗΣ Β' 83 τ.3/38

Page 41: Ευκλειδης Β 83

------------ Μαθηματικά για την Β' Λυκείου -----------( ln x = l ή ln x = -2 ) <=> x = e ή x = e-2

Λ σ κψrαy 8 Ν α αποδείξετε ότι:

β γ α log- Jog- log-α γ · β α · γ β = 1 ,α,β,γ > Ο.

2 χ log- Iog-B' Ν α λύσετε την εξίσωση χ 3 • 3 2 = 8 . !Γ' ! 1 In� ιη.Σ

2 Υ . Χ 2 = y-2 · ; Να λύσετε το σύστημα:

Ι · y 2 ln- ln-X 2 · y χ = 4-1

Να λύσετε την ανίσωση f(x) < 1 .

Να λύσετε την εξίσωση f(x) = -1 {f( α) · f(β) = -1 Να λύσετε το σύστημα: (Σ) f(α)+f(β) =Ο

, . Για να ορίζεται η f(x) πρέπει και αρκεί: { χ > 0 , δηλαδή χ > 1 . Άρα η f έχει σύνολο

ln x > Ο ορισμού το Α = (1, +οο) .

Με χ Ε Α έχουμε : f(x)< l <=> ln(lnx) < lne <=> ln Χ < e <::::> Χ < ee <::::> Χ Ε ( 1 , ee ) ,

Α ν Α το πρώτο μέλος, τότε : 1 , Η χ Ε Α έχουμε : f(x)= 1 <=> ln(lnx) = lne- <=>

1�=( lcg/3-l<gy) lcgα+( l<gy-lcgα) lcgβ+( lcgα -lcgβ) l<gy= οπότε Α=1 .

Η προηγούμενη σχέση, αν θεωρήσουμε : (α, β, γ) = (χ,2,3) με χ>Ο γίνεται:

2 3 χ log- log- log-

X 3 · 2 χ • 3 2 = 1 (*), οπότε : 2 χ

log- log- (*) 1 χ 3 . 3 2 = 8 <=> -- = 8 <=> 3 log-

2 χ

3 - log- 3 3 2 χ = 23 <=> log - = -3 <=> - = e-3 χ χ

3 3 <=> χ = ----=3 <=> χ = 3e e

Η αρχική σχέση ισχύει αν αντί logt έχουμε lnt. Θεωρούμε: (α,β,γ) = (2,x,y) με x,y>O

Ιη� 1n2'. Ιη� οπότε έχουμε: 2 Υ · χ 2 · y χ = 1 (** ) Το σύστημα με τη βοήθεια της (**) , γράφεται:

� = y-2 1 2 �- - �- � Υ χ Υ x = y

1 - 4- 1 <=>

- \η� --χ - 2 Υ = Γ2 ln-2 Υ

2 ln - = 2

χ <=>

χ ln - = 2

Υ

' ι Έστω η συνάρτηση f(x) = ln(ln χ) .

Να βρείτε το πεδίο ορισμού της f.

I -1 e-1 , , , e-1 - Ο ln χ = e <=> χ = e δεκτη τιμ η αφου e =ee >e = 1 .

Με α, β Ε Α παρατηρούμε ότι f(α), f(β) στο σύστημα (Σ) είναι ρίζες της εξίσωσης: t2 -1=0, δηλαδή της ( t + l)( t - 1) = 0 . Άρα (Σ) ς::;

<=>{f(α) = - 1 , { f(α) = 1

<=> f(β) = 1

η f(β) = -1 {Ι η ( ln α) = - 1

ή { ln ( ln α) = 1

<=> ln ( ln β ) = 1 ln ( ln β ) = - 1 { ln α = e- Ι

ή { ln α = e

<=> ln β = e ln β = e - Ι {α = ee- 1 , { α = ee

<=> η - 1 β = ee β = ee

Έστω η συνάρτηση f(x) = log(2 · 25x - 5 · 4x )

και η ευθεία ε : y = χ + log 3 .

Να βρείτε το πεδίο ορισμού της f.

::.

Να υπολογίσετε τα κοινά σημεία της

γραφικής παράστασης της f και της ευθείας

ε.

Να αποδείξετε ότι το σημείο

Α(� , f(�)) είναι «πάνω» από το σημείο Β 2 2

θ , , , , 3 της ευ ειας ε, το οποιο εχει τετμημενη 2 .

Για να ορίζεται η f(x) πρέπει και αρκεί 2 · 25χ - 5 · 4χ > Ο . Άλλα:

ΕΥΚΛΕΙΔΗΣ Β ' 83 τ.3/39

Page 42: Ευκλειδης Β 83

------------ Μαθηματικά για την Β ' Λυκείου -----------( i ) <:::> 2 · 25x > 5 · 4χ <::::>

52χ 5 5 5 2 ο 52 χ > 5 ο 22χ <::::> - > - <::::> (-)2Χ > (-)

22χ 2 2 2

2 1 1 , 5 Ά f ' <=> χ > <=> χ > - , αφου - > 1 . ρα η εχει 2 2

σύνολο ορισμού το Α = ( ..!__ , +οο) . 2

Οι τετμημένες των σημείων τομής της γραφικής παράστασης της f και της ευθείας ε, προκύπτουν από τις λύσεις της εξίσωσης: log(2 · 25 χ - 5 · 4χ ) = χ + log 3 ( 1 ) Στο Α έχουμε: ( 1 ) <=> log(2 · 52x - 5 · 22χ ) = log lOx + log3 <=>

log(2 · 52χ - 5 · 22χ ) = log(2x · 5χ · 3) <=> 2 ο 52χ - 5 ο 22χ - 3 ο 2 χ ο sx = ο <=> ( 5 )lx ( 5 )χ

2 · 2 - 3 · 2 - 5 = 0 <::::> ( 5 Jx , ( 5 Jx 5 ( 5 Jx 5 2

= - l η 2 = 2 <=> 2 =

2<=> χ = 1 .

'Εχουμε: f(l) = log(2 · 53 - 5 · 23 ) = log 2 1 0 . 2

Αρκεί λοιπόν να αποδείξουμε ότι:

log 2 1 Ο > l + log 3 , ή log 2 1 Ο - log 3 > l , ή 2 2

3 3 log 70 > l , ή log 70 > log ι ο2 , ή 70 > ι ο2 , ή

2 702 > 1 03 , ή 4900> 1 000, που ισχύει.

Έστω η συνάρτηση f(x) = ln χ + χ - 1 .

Να βρείτε το πεδίο ορισμού της Α.

Αν Ο < χ < 1 , να αποδείξετε ότι: f(x) < Ο.

Αν θ Ε (Ο, π

) , να αποδείξετε ότι: 2

συνθ < el-συνθ .

Η f(x) έχει προφανώς σύνολο ορισμού Α = (Ο, +οο) .

Ο < χ < 1 => ln χ < Ο και χ - 1 < Ο =>

ln χ + χ - 1 < Ο => f(x) < Ο .

π (β) γ) θ ε (0, -) => Ο < συνθ < 1 =>

2

f( συνθ) < Ο => ln( συνθ) + συνθ - 1 < Ο =>

ln( συνθ) < 1 - συνθ => συνθ < e1-συνθ .

Έστω η συνάρτηση f(x) = ln(x + k), k Ε R .

της οποίας η γραφική παράσταση διέρχεται

από το σημείο (e2-1 ,2).

Ν α βρεθεί το k ώστε η γραφική της

Ν α λύσετε την εξίσωση

f4 (x) = 2f2 (x) - 1 .

από το σημείο

, , {f(x) = f(y - 1) να λυσετε το συστημα .

f(f(x)) = ln 2

Η f έχει σύνολο ορισμού το Α = ( -k, +οο ) . Στο Α έχουμε : f( e2 - 1) = 2 <=> ln( e2 - 1 + k) = 2 e2 - 1 + k = e2 <=> k - 1 = Ο <=> k = 1 . Δεκτή τιμή αφού e2 - 1 > - 1 = -k . Άρα A = (- l, +oo ) .

Στο Α έχουμε : f(x) = 1n(x + 1), χ > - 1 Η εξίσωση έχει σύνολο ορισμού το ( - 1, +οο) , οπότε: f\x) = 2f2 (χ ) - 1 <=>

f4 (x) - 2f2 (x) + 1 = 0 <=> (f2 (x) - 1 )2 = 0

<:::> f2 (x ) - 1 = 0 <=> f(x) = - 1 ή f(x ) = l <=> ln(x + 1) = - 1 ή ln(x + 1) = - 1 <::::> x + 1 = e- 1 ή x + 1 = e <=> x = e- 1 - 1 ή

x = e - 1 . Για να ορίζεται το σύστημα πρέπει και

αρκεί x,y- 1 , f(x),y > - 1 , δηλαδή x > e- 1 - 1 , y > O .

Τότε : {f(x) = f(y - 1) { ln(x + 1) = ln y f(f(x)) = 1n 2 <=> 1n(ln(x + 1) + 1) = ln 2

<=> { x + 1 = y { x + 1 = y ln(ln y + 1) = 1n 2 <=> 1n y + 1 = 2

<=> {χ = y - 1 <=>{χ = e - 1

. ln y = 1 y = e

Δεκτές τιμές αφού e - 1 > e - 1 - 1 και e>O.

ΕΥΚΛΕΙΔΗΣ Β' 83 τ.3/40

Page 43: Ευκλειδης Β 83

------------ Μαθηματικά για την Β ' Λυκείου -----------

Β ' ΛΥΚΕΙΟΥ Λογαρ ιθμ ι κές - Εκθετι κές Ανισώσε ι ς

Διονύσης Γιάνναρος

Στο άρθρο αυτό παρουσιάζουμε μια μέθοδο επίλυσης λογαριθμικών και εκθετικών ανισώσεων, που βασίζεται στις παρακάτω δύο προτάσεις:

Η ρ 6ταση l : Αν α,β,γ>Ο και αf: 1 , τότε : • lοgαβ> lοgαγ � (α-1 )(β-γ)>Ο ο lοgαβ<lοgαγ � (α-1 )(β-γ)<Ο

Π ριίταση 2 : Αν α>Ο, αf: 1 και β, γ εR, τότε : • αβ>αγ � (α-1 )(β-γ)>Ο .. αβ>αγ � (α-1 )(β-γ)>Ο

Η απόδειξη και των δύο αυτών προτάσεων όπως θα δούμε βασίζεται στο γεγονός ότι: Αν α>1, τότε οι συναρτήσεις f(x)=logαx και g(χ)=αχ είναι γνησίως αύξουσες ενώ, όταν Ο<α<1 είναι γνησίως φθίνουσες στο R+ * , R αντιστοίχως. Πράγματι, για την πρόταση 1 ,

παρατηρούμε ότι: lοgαβ>lοgαγ� [α > 1 ) ή β > γ [α<1J� [α- 1 > ο ) ή [α-1 <0

J�(α-1 )(β-γ)>Ο. β<γ β - γ > Ο β-γ<Ο

Ομοίως: lοgαβ< lοgαγ � �(α- 1 )(γ-β)>Ο�(α-1 )(β-γ) < Ο Τελικά λοιπόν οι ισοδυναμίες της πρότασης

1 ισχύουν και μάλιστα για ανισοϊσότητες. Ειδικά για γ= 1 γίνονται: lοgαβ�Ο �

�( α-1 )(β-1 )� Ο και lοgαβ::::Ο � ( α-1 )(β-1 ):::::0 . Ανάλογα για την πρόταση 2, παρατηρού-

με ότι: αβ >αγ � [α > 1 ) ή [α < 1 )� [α-1>0 J β > γ β < γ β-γ>Ο

ή [α- 1 < 0 ) � (α-1 )(β-γ) > Ο κ.λ.π. β - γ < Ο

Ισχύουν δε και ως ανισοισότητες. Ειδικά για γ=Ο γίνονται: αβ� 1 � (α- 1 ) ·β�Ο και αβ:::; 1 � (α- 1 ) · β::::Ο . • Είναι σημαντικό να παρατηρήσουμε, ότι οι

ισοδυναμίες της πρότασης 2 ισχύουν προφανώς και για α= 1 , καθόσον τότε και τα δύο μέλη τους αποτελούνται από ψευδείς προτάσεις στις ανισότητες και αληθείς προτάσεις στις ανισοϊσότητες

αντιστοίχως. Ακολουθούν μερικές εφαρμογές (λυμένες) με δύο τρόπους για

να φανεί σε ποιες περιπτώσεις συμφέρει

να χρησιμοποιούνται οι παραπάνω προτάσεις και σε ποιες όχι.

Ε φ α ρ μογi:ς 1 ) Να λυθεί η ανίσωση : log(x2-4)<0 (1)

ΛίJση : Η εξίσωση ( 1 ) έχει σύνολο ορισμού το Α= {χεR/ χ2-4>0} = (---οο,-2)υ(2,+οο)

η:ιιJπο ς : Στο Α έχουμε: ( l )�log(x2--4)< <log 1 � χ2 - 4 < 1 � χ2 - 5 < Ο � (x-J5 )(x+ J5 )<0 � χε (-J5 , J5 ) � χε (-J5 , -2) υ(-2, J5 ). β ' τρ6πος: (Από την πρόταση 1 για γ= 1 ) . Στο Α έχουμε: ( 1 )�( 1 0-1 )(χ2--4-1 )<0 � 9(χ2 - 5) < 0 � xε (-J5 , J5 ) � � χε (-J5 , -2) υ(-2, J5 ). Να λυθεί η ανίσωση : logx(x2-4)<0 (2)

ΛίΗιηι : Η εξίσωση (2) έχει σύνολο ορισμού το Α= {χεR/ χ>Ο, x:;t 1 , χ2--4>0} = (2,+οο) . α c τρόπος: Στο Α έχουμε: χ>2> 1 , οπότε : (2) � logx (χ2--4)< logx 1 � χ2 - 4 < 1 � χ2 - 5 < Ο � χε (-J5 , J5 ) � χε (2, J5 ) . β τρti'υπος : (Από την πρόταση 1 για γ= 1 ) Στο Α έχουμε: (2)�(χ-1 )( χ2 - 4 - 1 )<0

� χ2-5<0�χε (-J5 , J5 ) �χε (2, J5 ).

3 ) Να λυθεί η ανίσωση : Ιοgχ+3 [ l + x: )>o (3) 1 - χ

ΛiJ ση : Η (3) έχει σύνολο ορισμού Α= {χεR/χ+3>0,

2 1 + Χ2

x+3:;t 1 , 1-χ :;tO, --2 >Ο} = (-1 ,0) υ(Ο, 1 ) . 1 - χ

α ' τρι'ιτως: Στο Α έχουμε: χ+3>2> 1 ,

ΕΥΚΛΕΙΔΗΣ Β ' 83 τ.3/41

Page 44: Ευκλειδης Β 83

------------ Μαθηματικά για την Β ' Λυκείου ------------οπότε : (3) <=> logx+3 [ 1 + χ: ) > logx+3 1 <=>

1 - χ

1 + χ2 > 1 <=> � >Ο � 1 -χ2 >Ο <=> 1 - χ 2 1 - χ2

χε (-1 ,O)u(O, 1 ) β ' τρόπος : (Από την πρόταση 1 για γ= 1 )

Στο Α έχουμε: (3 )<=>(χ+3-1 )( 1 + χ: -1 )>0 1 - χ

2χ 2 χ +2> 1 2χ 2 χ ;< Q <=> (χ+2)( -- )>Ο <=> -- >0 <:::> 1-χ2>0

1 - χ2 1 - χ2

<=> χε (- 1 ,0)υ(Ο, 1 ) .

4) Να λυθεί η ανίσωση : (x-S)logx (2χ+1)<0 (4) Λύση : Η (4) έχει σύνολο ορισμού Α= {χεR/χ>Ο, x:f 1 , 2χ+ 1>0} = (0, 1 ) υ( l ,+οο) .

α ' τρόπος : Στο Α έχουμε: i) Αν χ>5, τότε 2χ+ 1> 1 1 οπότε

logx(2x+ 1 )>logx 1 1 >0. Άρα δεν ισχύει η ( 4) . ii) Αν 1< χ<5 , τότε (4)<=> logx (2χ+ 1 )> Ο <=>

2χ+ 1> 1 <=> χ>Ο <=> χε ( Ι , 5 ) . i i i ) Αν Ο<χ< 1 , τότε 1 <2χ+ 1 <3 , οπότε

logx(2x+ 1 )> logx3>0. Άρα δεν ισχύει η ( 4) . β' τρόπος: (Με βάση την πρόταση 1 για γ= 1 )

' {χ > 5 ' Στο Α εχουμε: (4)<=> η logx (2χ + 1) < Ο

{χ < 5 {χ > 5

logx (2x + 1) > 0<=>

(χ - 1)(2χ + 1 - 1) < 0 ή

{χ < 5 {χ > 5 (χ - 1)(2χ + 1 - 1) > 0<=>

χ � (Ο, Ι) ή αδυνατο {χ < 5

<=>χε ( 1 ,5) . χ ε ( -οο, Ο) u ( 1 , +οο)

Σχόλιο : Θα μπορούσαμε (ως συνήθως), πιο σύντομα, να βρούμε πρώτα το πρόσημο κάθε παράγοντα χ-5, logx(2x+ I ) και μετά του γινομένου τους, σε πίνακα.

5) Να λυθεί η ανίσωση :

log2x (2χ2 -4χ+6)� log2x (χ2+χ) (5)

ΛίJση : Η εξίσωση (5) έχει σύνολο ορισμού το Α= {χεR/ 2χ>Ο, 2x:f 1 , 2χ2-4χ+6>0,

2 1 1 χ +χ>Ο} = (0, - ) υ( - ,+οο) . 2 2

α ' τρόπος : Στο Α έχουμε:

i) Αν χ> _!_ , τότε (5) <=> 2χ2-4χ+6< χ2+χ 2

-

<=> Χ2-5χ+6 �0 <=> ΧΕ [2,3 ] .

i i) Αν χε (Ο, � ) , τότε (5)<=> 2χ2-4χ+6�

2 1 χ +χ<=>χε (-οο,2]υ[3 ,+οο)<=>χε (Ο, - ) . 2

Τελικά: (5)<=> χε (Ο, _!_ )u[2,3] . 2

β ' τρόπος: (Με βάση την πρόταση 1 για α= 2χ, β=2χ2-4χ+6 και γ=χ2+χ). Στο Α έχουμε: (5)<=> (2χ-1 ) [2χ2-4χ+6-(χ2+χ)] ::;ο <=> (2χ-1 )(χ2 -5χ+6)::; ο <=> (2χ-1 )(χ-2)(χ-3)::;0 Θέτουμε τις ρίζες στον άξονα χχ ' , οπότε :

1 1 (5)<=>χε (-οο,2

)υ[2,3]<=>χε (Ο, 2

)u[2,3 ] . 1

.οο - 2 too 2

+ I . I +

• Η πρόταση 2 όμως μπορεί να εφαρμοσθεί μόνο σε

εκθετικές αvισώσεις με θετική βάση, όπως θα δούμε

στην συνέχεια.

6) Να λυθεί η xx�x3x-t (6) στο Α= (0, +οο).

Λί>ση : α ' τ ρόπος : Στο Α έχουμε:

i) Αν χ>1 , τότε (6)<=>χ::Ξ;3χ-1<=>χ�_!_ <=>χε( l ,+οο). 2

ii) Αν Ο<χ< 1 , τότε (6)<=>χ>3χ-1<:::::>χ<_!_ <=>χε(Ο, _!_ ] . -- 2 2

iii) Αν x=l , η (6) ισχύει. Άρα (6)<=>χε(Ο, _!_ ]u[ l ,+oo). 2

β ' τρ όπος : (Με βάση την πρόταση 2 για α= χ, β=χ και γ=3χ-1 ) . Στο Α έχουμε: (6)<=>(χ-1 ) [χ-(3χ-1 )]::Ξ;Ο<:::::>(χ-1 )( 1-2χ)::Ξ;0<=>

1 (χ- 1 )(2χ-1 )�0<=> χε (Ο, - ]u[ 1 ,+oo) . 2

l:ημι:ίωση : Η εξίσωση (6) όμως επαληθεύεται και για

αρνητικές τιμές του χ, π. χ. για χ=-3 γίνεται ( -3Γ3 ;"; ( -3)-10 • Αφήνουμε λοιπόν ως ανοιχτό πρόβλημα για τους αναγνώστες την πλήρη λύση της (6) καθώς και των:

χ 1 χ -2 χ χ 1 +3 χ � \ , ι: � ) > ( X: J )

ΕΥΚΛΕΙΔΗΣ Β ' 83 τ.3/42

Page 45: Ευκλειδης Β 83

Β ' ΛΥΚΕΙΟΥ

Στο διπλανό σχήμα οι χορδές ΑΒ = Γ Δ = λ6 τέμνονται υπό γωνία 30° στο σημείο Μ. Υπολογίστε την περίμετρο και το εμβαδόν του καμπυλόγραμμου τριγώνου ΜΓΒ.

Φέρουμε τις ΟΓ και ΟΒ και παρατηρούμε ότι αφού ΑΒ=ΓΔ=λ6, τα τρίγωνα ΑΟΒ και ΓΟΔ είναι ισόπλευρα. Άρα, f = Β = 60° και Α = Δ = 60° . Αφού όμως Μι = Μ2 = 30° έχουμε ότι τα τρίγωνα ΜΚΓ και ΜΛΒ είναι ορθογώνια. Συνεπώς,

RJ3 ΟΚ = ΟΛ = α =--6 2 RJ3 R (2 -J3)

και ΚΓ = ΛΒ = R ---= .

Από το ορθογώνιο τρίγωνο ΜΚΓ

ΜΓ = 2 · ΚΓ = ( 2 -J3)R και ομοίως

2 2 αφού Μι = 30° ,

MB = (2 -J3)R . Εξ

� Μέτρηση Κύκλου Αθανάσιος Κοιλάκος

άλλου εύκολα βλέπουμε ότι, Ρ = πR · 30 = πR 'Ετσι

ΑΟΓ = ΓΟΒ = ΒΟΔ = 30° . Άρα το μήκος f του τόξου ΓΒ είναι

� 1 80 6

. το καμπυλόγραμμο τρίγωνο ΜΓΒ έχει περίμετρο

ΜΓ + ΜΒ + ι = ( 4 - 2J3 + � J · R . Αν, τώρα, Ε είναι το εμβαδόν του θα ισχύει ότι

Ε = (οm) - (ΒΟΚ ) - (ΜΚΓ) = πR2 30 _ _!_ΚΒ · ΟΚ -_!_ΓΚ · ΓΜ · ημ60ο ή ( . . . ) E = ('i +.!!:_ -J3)R2 τ.μ.

360 2 2 2 1 2

Δίνεται κύκλος (0, R) και δύο μικρότεροι κύκλοι (Κ1, r), (Κ2, r) με Ο < r < R , οι οποίοι εφάπτονται 2

θ Rη μ - -. και μεταξύ τους και εσωτερικά με τον (0, R). α) Να αποδείξετε ότι r = θ 2 όπου θ = Κ1 ΟΚ2 •

η μ - + 1 2 β) 'Εστω r = ( 2J3 - 3) R . Τότε (i) Ν α δείξετε ότι Ο < r < � και να υπολογίσετε τη γωνία θ. (ii) Α ν Α, Β, Γ είναι τα σημεία επαφής των τριών κύκλων, να υπολογίσετε την περίμετρο και το εμβαδόν (ΑΒΓ) του καμπυλόγραμμου τριγώνου ΑΒΓ.

α) Παρατηρούμε αρχικά ότι αφού Ο < r < R , ο κύκλος (Κ1 , r) τέμνει 2

την ακτίνα ΟΒ στο Β και σ' ένα ακόμη εσωτερικό της σημείο . Ομοίως και ο (Κ2, r) την ΟΓ. [Το σχήμα έχει σχεδιαστεί με τις υποθέσεις του ερωτήματος (β) ώστε να διευκολύνει στην επίλυση του θέματος. Στην γενική περίπτωση τώρα, εφαρμόζουμε το νόμο των συνημιτόνων στο τρίγωνο Κ10Κ2. ] Είναι ΟΚ1 = ΟΚ2 = R - r και ΚιΚ2=2r. Άρα Κι Κ� = ΟΚ� + ΟΚ� - 20Κ ι · ΟΚρυνθ ή

θ Rημ-(2r)2 = 2(R - r)2 ( 1 -συvθ) ή (2r) = 2 (R - r) η μ� ή r = θ

2 . Η 2 ημ2 + 1

ΕΥΚΛΕΙΔΗΣ Β ' 83 τ.3/43

Page 46: Ευκλειδης Β 83

------------- Μαθηματικά για την Β ' Λυκείου -------------

, θ Ο R ξ ' ' ' ξ' Π ' θ r Α ' ' ' θ υπο εση < r < - ε υπακουεται απο αυτη την ε ισωση . ραγματι η μ- = -- . φου ομως η γωνια 2 2 R - r ' ' θ ' Ο θ 1 ' 0 r 1 ' 0 R ανηκει σε τριγωνο α πρεπει < ημ-2 < η < -- < η < r < -2 . R - r

β) (i) Είναι r = ( 2.J3-3) R . Τότε, προφανώς, r > Ο . Η ανισότητα r < � ισοδυναμεί με 7 > 4.J3 ή με

49 48 λ θ , Τ , θ r 2.fi - 3 .J3 Ά θ .J3 , Ο θ π > που α η ευει. ωρα η μ- = -- = r:; = - . ρα η μ- =- και αφου < - < -2 R - r 4 - 2ν3 2 2 2 2 2

θ θ � � � � συμπεραίνουμε ότι - = 60° ή - = 1 20° . Συνεπώς, ΟΚ1Α = ΟΚ2Α = 30° και ΒΚ 1 Α = ΓΚ2Α = 1 50° . 2 2 β) ( . . ) 'Ε , ΒΓ , , πR · 1 20 _ 2πR Α ,

ΑΒ� ΑΓ , 'δ , , ιι χουμε οτι το εχει μηκος - . κομη, τα και εχουν το ι ιο μηκος ισο 1 80 3 πr · 1 50 5πr ( ι ο.J3 - 1 5 ) ( 2 ι ο.J3 - 1 5 ) ι o.fi - 1 3 με = - = πR . Έτσι το ΑΒΓ έχει περίμετρο πR - + = πR . 1 80 6 6 3 3 3

( ___..,_ ) ( Δ J ( � ) ( ___..,_ ) πR2 120 1 πr2 1 50 Τώρα, (ΑΒΓ) = ΟΒΓ - ΟΚΙ Κ2 - KI BA - κ2�Α = 360 -20ΚΙ · ΟΚ2 · ημ1 20° - 2360 = ( . . . )

= ( 1 2 + I Oπ.fi - 7.J3- Ι Ο:π )R 2 τ.μ . . Π ρ6βλημα 3 Γ 3 δ 'ξ , Ε: -Εν αν + R Π , θ , 'ζ λ' , , ια ν � να απο ει ετε οτι ,

= . οιον αρι μο προσεγγι ει το πη ικο αυτο οταν το ν Ρν -Ρν 2

αυξάνεται απεριόριστα; (Συμβολικά ν � +οο ). Λί)ση

Ε' Ε I Ρ' I I Ρ λ' I λ λ' I λ 'Ε ου ε ν - ν = 2 v αv - 2 ν αν = ν να ν - ν v αv = v αv - v αv . Α ού ό ω λ' = R λ /α χ μ Ρ' - Ρ (λ' - λ ) 2 (λ' - λ ) 2(λ' - λ ) φ ' μ ς, v v v

v ν v v ν ν ν ν ν ν και

R'λv - λ α R2 - α2 α� = R ο παραπάνω λόγος ισούται με αν , v ν ν

2( Rα> - λν ) 2 (R - αν ) 2 Καθώς το ν αυξάνεται χωρίς

, 'ζ , R , αν + R 'ζ θ , R + R R περιορισμο το αν προσεγγι ει την ακτινα οποτε το 2 προσεγγι ει τον αρι μο -2- = .

Π ρ{ιβλημα 4 Δίνεται κανονικό εξάγωνο Α1Α2Α3Α4Α5Α6 εγγεγραμμένο σε κύκλο (O,R). υ) Να υπολογίσετε τα μήκη των διαγωνίων του. β) Να υπολογίσετε το εμβαδόν του τετραπλεύρου ΑιΑ3Α4Αs και τα εμβαδά των τριγώνων ΑιΑ2Α3, ΑιΑsΑ6. γ) ΝΑ υπολογίσετε το συνολικό εμβαδόν Ε των 6 κυκλικών � Α τμημάτων που σχηματίζονται από τις πλευρές του ') :::. εξαγώνου. δ) Να αποδείξετε ότι (ΑιΑ3Α4Αs)>(ΑιΑ2Α3) +(ΑιΑsΑ6)+Ε ι\ί)ση α) Έχουμε δύο ειδών διαγωνίους. Αυτές που αφήνουν στα δύο ημιεπίπεδα 1 και 3 κορυφές αντίστοιχα π.χ. Α1Α3, και αυτές που αφήνουν στα δύο ημιεπίπεδα 2 και 2 κορυφές αντίστοιχα π.χ. ΑΙ�· Παρατηρούμε ότι Α Α λ Α Α, Ο\ =Α, Ο\ +1\0\ +1\0\ = 60° + 60° + 60° = 1 80° δηλαδή τα σημεία Α1 , Ο, � είναι συνευθειακά. Επομένως, Α1Α4 = 2R . Τώρα Α1 Α3Α4 = 90° αφού η εγγεγραμμένη αυτή γωνία βαίνει σε ημικύκλιο. Έτσι

ΕΥΚΛΕΙΔΗΣ Β ' 83 τ.3/44

Page 47: Ευκλειδης Β 83

------------- Μαθηματικά για την Β ' Λυκείου ------------­από το Π. Θ. στο τρίγωνο Α1Α3Α4 έχουμε Α1Α� = Α1Α� - Α3Α� = = (2R)2 - R2 = 3R2 ή Α1Α3 = R.J3 . β) Τα τρίγωνα Α1Α3Α4 και Α1Α4Α5 είναι ίσα οπότε (A1A3A4As ) = 2(A1A3A4) = 2 · .!.A1A3 · A3A4 =R2.J3 . 2 Ακόμη, τα τρίγωνα Α1Α6Α5 και Α1Α2Α3 είναι ίσα και Az = Α6 = 1 20° . Άρα, (ΑΙΑ2Α3 ) = (Α1Α6Α5 ) = _!_Α2Α1 · Α2Α3 · ημ1 20ο = _!_R2ημ60ο = R2.J3 . 2 2 4 γ) Το κυκλικό τμήμα που σχηματίζει η χορδή Α1Α2 έχει εμβαδόν ε = (οu ) - (οΑ Α ) = πR2 60 .!.οΑ · ΟΑ · ημ60ο = = 2:_R2 -� · .J3 = R2 [2:_ - .J3 J τ .μ. I 2 I 2 360 2 I 2 6 2 2 6 4

δ ) Η ζητούμενη ανισότητα ισοδυναμεί με την R' J3 > R ΊJ3 + R' [π - 3� J ή με την 2 J3 > π . Όμως,

.J3 :::: 1 , 732 οπότε 2.J3 > 2 · 1 , 7 = 3, 4 > π . Π ρ{ψl..η μυ 5 Δίνεται ισοσκελές τρίγωνο ΑΒΓ με ΑΒ = ΑΓ = β = � J3 + 2 και ΒΓ = α = 1, το οποίο είναι εγγεγραμμένο σε κύκλο (0, R). Στο σημείο Β φέρουμε την εφαπτομένη του κύκλου που τέμνει την προέκταση της ΑΓ στο σημείο Δ. Θέτουμε χ = Γ Δ και y = ΒΔ. ω Να αποδείξετε ότι τα τρίγωνα ΑΒΔ και ΒΓΔ είναι όμοια και να υπολογίσετε τα χ, y. !) ) Ν α αποδείξετε ότι ο κύκλος είναι μοναδιαίος και να υπολογίσετε το εμβαδόν του κυκλικού ..--.. τμήματος που ορίζεται από την χορδή ΒΓ. γ ) Να υπολογίσετε τα μήκη των τόξων ΑΒ και ΑΓ . δ ) Να υπολογίσετε το εμβαδόν του μεικτόγραμμου τριγώνου ΔΒΓ . . \ ί1ση

Α

β = �.J3 + 2 έχουμε

Δ Δ Λ υ) Εύκολα βλέπουμε ότι ΑΒΔ � ΒΓ Δ αφού έχουν τη γωνία Δ κοινή

Λ Λ Λ και Α = Γ Β Δ αφού η γωνία Γ Β Δ σχηματίζεται από τη χορδή ΒΓ και την εφαπτομένη ΒΔ στο σημείο Β. Άρα ΑΒ - ΑΔ - ΒΔ η' ' ΒΓ - ΒΔ - ΓΔ

Ι = χ + β = r Από εδώ λαμβάνουμε το σύστημα α Υ χ y = Ix }

y2 = χ(� + β) οπότε με αντικατάσταση έχουμε β: χ2 = χ( χ + β) ή αφού χ > Ο,

α β2 , α2β , αβz -2 χ = (χ + β) η χ = 2 2 . Άρα, Υ = 2 2 • Βέβαια, για να έχουν α β - α β - α νόημα τα χ, y πρέπει β > α το οποίο εδώ ισχύει. Θέτοντας α= 1 ,

.J3 + 2 .J3 + 1 �.J3 + 2 J2 y = -- = -- και χ = =-.J3 + 1 2 .J3 + 1 2

Λ Λ Λ β) Αρκεί να βρούμε τη γωνία Α αφού τότε Ο = 2Α . Εφαρμόζοντας το νόμο των συνημιτόνων στο ΒΑΓ ' 2 2β2 2β2 Α ' Α 1 α 2 ' Α 1 1 .J3 ' Α .J3 ' εχουμε α = - συν η συν = -- οποτε συν =

( ,-;:;

) =- η συν =- η 2β2 2 ν3 +2 2 2

Λ Λ Λ Λ Α = 30° . Συνεπώς, Ο = 60° . Άρα, ΟΒΓ = Ο Γ Β = 60° οπότε το τρίγωνο ΒΟΓ είναι ισόπλευρο . Άρα, R = α = 1 δηλαδή ο κύκλος είναι μοναδιαίος. Αν ε είναι το εμβαδόν του κυκλικού τμήματος τότε ε = (οΒΓ) - (ΒΟΓ) = πR 2 • 60 .J3 = 2: - .J3 τ.μ. 360 4 6 4

Λ Λ Λ γ) Από τη συμμετρία του σχήματος, η ΑΟ διχοτομεί τη γωνία Α , οπότε ΟΑΒ = 1 5° . Έτσι ΑΟΒ = 1 50°

ΕΥΚΛΕΙΔΗΣ Β' 83 τ.3/45

Page 48: Ευκλειδης Β 83

-------------- Μαθηματικά για την Β ' Λυκείου -------------

Λ � � πR - 1 50 5π και όμοια ΑΟΓ = 1 50° . Άρα, μήκος(ΑΒ) = μήκος(ΑΓ) = =- . 1 80 6

;; \ 1 ο 1 1 J3 + 1 [ π J3 J 3J3 1 π ο , Ε = (ΔΒΓ) - ε = 2yαημ30 - ε = "2 " 2 ·-

2-- 6 - 4 =-

8-+ 8 -6 τ.μ . .

π 6

ο Α Β

Σε ένα τεταρτοκύκλιο (0, R) εγγράφουμε κύκλο ακτίνας r. Να βρείτε την α�τίνα r και το εμβαδόν του κύκλου αυτού.

\� Λ Λ Στο ορθογώνιο τρίγωνο ΚΟΑ είναι ΑΚ = r και Α Ο Κ = 45° . Άρα, Ο Κ Α = 45°

και ΑΟ = r. Με το πυθαγόρειο θεώρημα βρίσκουμε ΟΚ = rJ2 . Εξ άλλου έχουμε

ΟΒ = ΟΚ + ΚΒ ή R = rJ2 + r ή r = _R_ ή r = R (Ji - 1 ) . Άρα J2 + 1 '

Ε = πr2 = πR2 ( 3 - 2J2) τ.μ.

Σε ένα τεταρτοκύκλιο ΒΟΑ ακτίνας R γράφουμε κύκλο με κέντρο Β και ακτίνα R, ο οποίος τέμνει το τεταρτοκύκλιο στο Γ. Ν α υπολογιστεί το εμβαδόν

Α του καμπυλόγραμμου τριγώνου ΟΓ Α. Ο ""-�-.,....-------4 . \ίJση

Φέρνουμε τις ΒΓ, ΟΓ και παρατηρούμε ότι ΟΓ=R ως ακτίνα του τεταρτοκύκλιου Λ και ότι ΒΓ=R ως ακτίνα του κύκλου (Β, R) . Συνεπώς, το τρίγωνο ΒΟΓ είναι ισόπλευρο δηλ. Β = 60° και

Λ ΓΟΑ=30σ . Έτσι το κυκλικό τμήμα που ορίζει η χορδή ΟΓ έχει εμβαδόν

ε = (Βοr

) - (ΒΟΓ) = πR2 60

-R2 J3 = R2 [!!_-JjJ τ.μ. Τώρα το ζητούμενο εμβαδόν υπολογίζεται από τη 360 4 6 4

διαφορά (οΑΓ) - ε = R' [ � - 1� J τ .μ . . Π ρίιβλη μα 7

Μια κυκλική πλατεία έχει ακτίνα R = 10,35 m ενώ το εσωτερικό της καλύπτει ομόκεντρος κύκλος ακτίνας ρ = 9,65 m. Ο κυκλικός δακτύλιος που περισσεύει πρόκειται να καλυφθεί από τετράγωνα πλακάκια με τρόπο ώστε δύο απέναντι κορυφές του τετραγώνου να βρίσκονται με συμμετρικό τρόπο πάνω στις δύο περιφέρειες.

·Υπολογίστε το εμβαδόν του κυκλικού δακτυλίου. τί μήκος πλευράς πρέπει να έχει κάθε τετράγωνο πλακάκι; "πόσα πλακάκια πρέπει να γίνουν παραγγελία; (εφ2° = 0,035)

Το εμβαδόν του κυκλικού δακτυλίου είναι προφανώς πR2 - πρ2 = π (R - ρ) (R + ρ) = π · Ο, 7 · 20 = 43, 982m2 Αν δ είναι η διαγώνιος των τετραγώνων τότε από το σχήμα

βλέπουμε ότι ρ + δ = R ή δ = R - ρ . Αν α είναι η πλευρά τότε (R - ρ)Ji

α = και με αντικατάσταση, α = 0, 495m . 2

·r ) Θ · θ ΑΟΛ Β , , ΑΟΒ · φe ΑΒ % δ R - ρ , φe R - ρ · • ετουμε = οποτε απο το εχουμε ε =-=--=--=-- η ε =--. ΟΒ ρ + % δ + 2ρ R + ρ R + ρ

Οπότε εφθ = 0,035 ή εφθ = εψ2° και αφού η γωνία θ είναι οξεία, θ = 2°. Όμως, κάθε τετράγωνο

«απασχολεί» γωνία 2θ = 4°. 'Ετσι θα χρειαστούν 360 = 90 πλακάκια. 4

ΕΥΚΛΕΙΔΗΣ Β ' 83 τ.3/46

Page 49: Ευκλειδης Β 83

-------------- Μαθηματικά για την Β ' Λυκείου

<<Οδοιπορικό>> πάνω στις Κωνι κές Τομές

Λ . Ε Ι � . \ ΓΩ Π ! : Θεωρούμε αναγκαίο, πριν εισέλθουμε στην

μεθοδική παρουσίαση θεμάτων, να αναφερθούμε στα εξής γενικότερα πράγματα:

I . ' Ενιις Ο ρ ισ μ (κ: της κωνιιο'j� γμ ιφμής απι'ι τ ο ν Λ πολλι!η• ιο ( 262 - ! lJO π .Χ . )

Κωνική γραμμή είναι ο γεωμετρικός τόπος των σημείων ενός επιπέδου, τα οποία έχουν την ιδιότητα: Ο λόγος των αποστάσεων των σημείων αυτών από ένα σημείο Τ και από μια ευθεία δ είναι σταθερός θετικός αριθμός, έστω ε (εκκεντρότητα). Το σημείο Τ ονομάζεται Εστία, η ευθεία δ ονομάζεται Διευθετούσα και ο αριθμός ε ονομάζεται Εκκεντρότητα. Εάν ε<l έχουμε Έλλειψη, εάν ε>l έχουμε Υπερβολή, εάν ε=l έχουμε Παραβολή.

2 . Γι:ν ικ 1] εξίσωση κ ι•η' Ί κψ; γμιφμt1 ς κω της εφωrτο μi:νης της στο :•π ι J π : rΙ ο .

Κάθε εξίσωση της μορφής: αχ2 + βy2 + γχ + δy + ε = 0 (*) , είναι εν γένει εξίσωση

κωνικής γραμμής (που αντιμετωπίζεται χωρίς στροφή των αξόνων), ή ευθείας ή σημείου ή αδύνατη. Όμως, όταν α * Ο ή β * Ο μία βασικότατη μέθοδος επεξεργασίας είναι η γνωστή μέθοδος συμπλήρωσης τετραγώνου, που γνωρίζουμε από την θεωρία του δευτεροβάθμιου τριωνύμου. Απλά και κάνοντας μία υπέρβαση, θα αναφέρουμε ότι η εξίσωση της εφαπτομένης της σε σημείο της Μ ( χ 0 , y 0 ) , δίνεται και από τον τύπο:

2αχοΧ +2βyοy+γ( χ +xo ) +δ(y+yo ) +2ε = 0 ( * *) .

Αν υποθέσουμε ότι έχει δοθεί η εξίσωση μιας επίπεδης γραμμής, έστω f(x, y)=O, χ, y Ε � και σημείο του επιπέδου αυτής Α( α, β), το πρώτο που ελέγχουμε είναι το ενδεχόμενο το σημείο Α να είναι σημείο της γραμμής αυτής, δηλαδή αν f(α,β)=Ο. Επίσης όταν μας δοθεί η εξίσωση μιας επίπεδης γραμμής, έστω f (χ, y) = Ο, χ, y Ε � και σημείο του επιπέδου αυτής Α( α,

β),και μας ζητηθεί να ασχοληθούμε με την εξίσωση της εφαπτομένης ευθείας από το σημείο Α( α, β) στην γραμμή με εξίσωση f (χ , y) = Ο, χ, y Ε �. ξεκινάμε από το

ενδεχόμενο η εφαπτομένη ευθεία να είναι εκείνη που έχει εξίσωση χ=α και συνεχίζουμε με το ενδεχόμενο να έχει εξίσωση της μορφής: y = λχ + κ.

Β. Θ Ε ί\/i Λ.ΤΛ : I . Έστω η παραβολή y2 = 2px, p > Ο και

ευθεία ε : αχ + βy + γ = Ο, με (α, β) -::1- (0, 0). Βρείτε τις ικανές και αναγκαίες συνθήκες που εξασφαλίζουν τις αντίστοιχες σχετικές θέσεις της παραβολής με την ευθεία.

ΛίJση :

χ =L ( 1 ) { 2

Θεωρούμε το σύστημα: 2p αχ + βy + γ = Ο ( 2)

Σωτήρης Ε. Λουρίδας Αν α = Ο, οπότε β -::1- Ο, η παραβολή με την

ευθεία έχουν μοναδικό κοινό σημείο το

Μ [_i_ _ r) που όμως δεν είναι σημείο 2pβ2 ' β '

επαφής. Α ν α -::1- Ο τότε ( ! ) 2

(2 )<=>αL+ βy + γ = Ο <=> αy2 + 2pβy + 2pγ = 0 ( 3 ) . 2p

Η ( 3 ) είναι 2ου βαθμού ως προς y και έχει διακρίνουσα Δ = 4p (pβ2 - 2αγ ) , άρα παίρνουμε: i) Δ > Ο <=> pβ2 > 2αγ <=> «Η ευθεία τέμνει την παραβολή», i i ) Δ = Ο <=> pβ2 = 2αγ <=> «Η ευθεία εφάπτεται της παραβολής» i i i ) Δ < Ο <=> pβ2 < 2αγ <=> «Η ευθεία δεν έχει κοινά σημεία με την παραβολή» . Όταν β * Ο, τότε

έχουμε : αχ +βy+γ = Ο<=>y=-�χ _ _r <=>y=λχ +κ, β β

· λ α Ο γ οπου =-- -::/- και κ = --β β '

οπότε η (i)

ισοδύναμα γράφεται p > 2λκ, η (ii) ισοδύναμα γράφεται p = 2λκ και η (iii) ισοδύναμα γράφεται p < 2λκ. Παρατηρούμε ότι οι σχέσεις ii, iii, εξασφαλίζουν από μόνες τους ότι λ -::1- Ο , ενώ στην i είναι αναγκαίο να δίνεται.

2 .Θεωρούμε την παραβολή χ2 = 2py, p > Ο. Βρείτε την κλίση της εφαπτομένης της παραβολής αυτής, όταν η εστία της παραβολής απέχει από την εφαπτομένη αυτή απόσταση 5p.

Λί>ση : Προφανώς η ευθεία χ = Ο δεν μπορεί να είναι

μία τέτοια εφαπτομένη . Έστω Α( x0 , y0 ) σημείο της παραβολής. Η εξίσωση της εφαπτομένης της στο σημείο αυτό είναι: XXa =p(y+y0) <=>XXa -py-py0 =0,

με κλίση λ= Χο ( *) . Το σημείο Α είναι σημείο της Ρ

παραβολής οπότε έχουμε : χ� = 2py0 ( * *) . Με βάση τις σχέσεις ( *) , ( * *) η εξίσωση της εφαπτομένης γίνεται: 2λχ - 2y - λ2p = Ο. Όμως η

εστία της παραβολής είναι Ε (Ο,� J , που σημαίνει ότι η απόσταση της εστίας από την εφαπτομένη

ΕΥΚΛΕΙΔΗΣ Β ' 83 τ.3/47

Page 50: Ευκλειδης Β 83

------------- Μαθηματικά για την Β ' Λυκείου -------------

2 1p + λ2p l Ρ� είναι d = r:---::1 = . Άρα παίρνουμε: 4ν 1 + λ2 2

Ρ� 1 + λ2 r:--::; � 5p =>ν 1 + λ2 = 1 0 =>ι! = 99 => λ = ±3ν 1 1 .

2 3 " α) Αποδείξτε ότι κάθε εφαπτομένη της

παραβολής y 2 = 15χ, έχει την μορφή χ = Ο ή 15

y = μχ + -, με μ :;t: Ο. 4μ

β) Χρησιμοποιώντας το αποτέλεσμα αυτό ή κάποιο άλλο, βρείτε τις κοινές εφαπτόμενες της παραβολής αυτής και του κύκλου: r +f = 16( *) .

Λ.ίJση : Προφανώς η χ = Ο, είναι εφαπτομένη της

παραβολής. Στην άσκηση ( 1 ) αποδείξαμε ότι για να εφάπτεται η ευθεία ε : y = λχ + κ της παραβολής C : y = 2px πρέπει και αρκεί p = 2λκ. Στην

1 5 περίπτωση μας - = 2λκ, δηλ. μπορούμε να 2

θ . λ ο 1 5 τ λ · · εσουμε: = μ :;t: και κ =-. ε ικα παιρνουμε:

1 4 ε : y = μχ +-. 4μ

β} Η χ=Ο, δεν είναι εφαπτομένη του κύκλου 1 5 (*) . Για να είναι η ευθεία y = μχ +-, με μ :;t: Ο 4μ

εφαπτομένη του κύκλου (*) , πρέπει και αρκεί η απόσταση του κέντρου του από την ευθεία μας να ισούται με την ακτίνα του κύκλου. Δηλαδή πρέπει

και αρκεί: � �4 ( 1) <c> 1_!2_ 12 = 1 6 (μ2 + 1 ) � \ιμ2 + 1 4μ

4 2 ( 3 ) · ( 3 ) � 256μ + 256μ - 225 = ο� μ = -4 η μ = 4 ο Τελικά οι ζητούμενες κοινές εφαπτόμενες

. θ ' 3 5 3 5 ειναι οι ευ ειες: y = -- x - , y = -x + . 4 4 .ι Θεωρούμε την οικογένεια των

παραβολών W = {CP / CP : y2 = 4px,p ε �· } .

α) Αποδείξτε ότι σε κάθε μία από τις παραβολές αυτές, υπάρχουν δύο σημεία Μ, τέτοια ώστε το τρίγωνο ΜΣΕ να είναι ισόπλευρο, όταν Ε η εστία της παραβολής και Σ η προβολή του Μ στην διευθετούσα της. �) ) Βρείτε τον γεωμετρικό τόπο των σημείων Μ.

Θεωρούμε Μ ( χ0 , Υ ο ) το ζητούμενο σημείο της παραβολής. Αφού ΜΣ = l\1E θα έχουμε: ΔΜΣΕ ισόπλευρο � ΜΣ=ΣΕ�(χ0 +p)

2 = 4p2 + y� � �χ� - 2px0 - 3p2 = Ο� ( χ0 = -p ή χ0 = 3p) , τελικά χ0 = 3p, αφού x0p ;:::: Ο . Παρατηρούμε λοιπόν ότι: χ0 = 3p, οπότε y� = 1 2p, δηλαδή Υ ο = ±2pJ3 .

Ζητάμε την ικανή και αναγκαία συνθήκη, μεταξύ των χ0 , y 0 ώστε να υπάρχει p ε �· : Χ 0 = 3p, Υ ο = ±2pJ3 . Για αυτό πρέπει και

X a r::; αρκεί y0 = ±2-ν3 , που σημαίνει ότι ο 3

ζητούμενος γεωμετρικός τόπος είναι το ζεύγος των . 2J3 2J3 ευθειων y =--χ Υ = ---χ.

3 ' 3 5. Θεωρούμε χορδή ΑΒ της παραβολής

y2 = 4px, p > Ο, που διέρχεται από την εστία της Ε. Αν Μ είναι το σημείο της παραβολής που η

εφαπτομένη της σε αυτό να είναι παράλληλη στην χορδή ΑΒ, aποδείξτε ότι: ΑΒ=4ΕΜ.

: δ ψ I -----� ' �� . _//r::-

Προφανώς είναι E(p,O) . Αν ΑΒ _i χ 'χ , έχουμε : ΑΒ = 4p, EM = p, οπότε ΑΒ = 4ΕΜ.

Θεωρούμε ότι AB,ix 'x , Α ( Χ ι , Υ ι ) , Β ( X z , Υ2 ) , • • 2 4 2 4 οποτε παιρνουμε : Υ ι = pχ ι , y 2 = px2 => {ΕΑ2 = (χ ι - p)2 + y� = (χ ι + p)2 } ΕΒ2 = (χ2 - p)

2 + y� = (χ2 + p)2 =>

ΕΑ = χι + p και ΕΒ = χ2 + p, αφού p · χ ;:::: Ο, p > Ο Άρα ΑΒ = Χ ι + χ2 + 2p. Από την άλλη μεριά

έχουμε: Μ ( χ0 , Υ ο ) => ΕΜ2 = ( χ0 - p )2 + y� =

= ( Xa - Ρ )2 + 4pχσ =

( Χ α + Ρ )2 => ΕΜ = Χ α + p.

Η εξίσωση της εφαπτομένης είναι y0y = 2p ( x + x0 ) δηλ. 2px - y0y + 2px0 = 0 με

συντελεστή διεύθυνσης λ = 2 Ρ = λ ΑΒ . Η εξίσωση Υ ο

2p y =-(x - p) . Άρα οι Υ ο

α) Έχουμε: E (O, p) , οπότε Σ (-p, y0 ) . της ΑΒ είναι:

ΕΥΚΛΕΙΔΗΣ Β' 83 τ.3/48

Page 51: Ευκλειδης Β 83

------------- Μαθηματικά για την Β ' Λυκείου -------------

συντεταγμένες των Α, Β επαληθεύουν το σύστημα: {'/ =4px,p > O 2p ( ) που σημαίνει ότι τα χ 1 , χ2 επαλη-y=- x-p Υ ο

θεύουν την εξίσωση : χ2 - (2p +4x0 ) x +p2 = 0. Επομένως χ 1 + χ2 = p + 2χ0 , οπότε παίρνουμε:

ΑΒ = 2p + 4χ0 + 2p = 4 ( χ0 + p) = 4ΕΜ. 2 2

6. 'Εστω η έλλειψη .;- + � = 1 και ευθεία α β

ε : α1χ + β1y + γ1 = 0, β1 :;t: Ο <::> y = λχ + κ, . αι γι οταν λ = --, κ = -- .

βι βι Βρείτε τις ικανές και αναγκαίες συνθήκες

που εξασφαλίζουν τις αντίστοιχες σχετικές θέσεις της έλλειψης με τη ευθεία ε.

ΛίJση :

-+ - = 1 ( 1 ) Θεωρούμε το σύστημα: α2 β2 { xz yz

y = λχ + κ (2 ) ,

( ) ( ι ) _:C ( λχ + κ)z -2 <::::> 2 + 2 - 1 <::::> α β

<::::> (β2 + λα2 ) χ2 + 2κλα2χ + α2 ( κ2 - β2 ) = Ο, με διακρίνουσα Δ = 4α2β2 (β2 + α2λ2 - κ2 ) . Τελικά παίρνουμε: i ) «Η ευθεία τέμνει την έλλειψη σε δύο σημεία» <::::> Δ > Ο <::::> α2λ2 + β2 > κ2 . i i ) «Η ευθεία εφάπτεται της έλλειψης» <::::> Δ = Ο <::::> α2λ2 + β2 = κ2 • i i i) «Η ευθεία δεν έχει κοινά σημεία με την έλλειψη» <::::> Δ < Ο <::::> α2λ2 + β2 < κ2 .

7. Αποδείξτε ότι οι ευθείες με κλίση λ που

εφάπτονται στην έλλειψη � + Υ: = 1, είναι της α β

μορφής y = λx+Jα2J!- + β2 ή y = λχ-Jα2λ2 + β2 .

Υπόδειξη : Άμεση εφαρμογή του προηγού­μενου θέματος, περίπτωση (ii) .

Εφαρμογή : Βρείτε τις εφαπτόμενες της έλλειψης:

χ2 y2 - + - = 1, από το σημείο Μ(1 ,3 ) . 9 4

Λ ίJση : Είναι φανερό ότι από το σημείο Μ δεν διέρχεται κατακόρυφη εφαπτομένη της έλλειψης. Για να διέρχεται η εφαπτόμενη y = λχ + ../9λ2 + 4 , από το σημείο Μ (1, 3) πρέπει και αρκεί, 3 = λ +../9λ2 + 4 (1) . Αλλά: ( 1) <::::> 3 - λ = ../9λ2 + 4 <::::>

<=> { 2 λ � 3

<::::> { {

λ �: 1 } 8λ + 6λ - 5 = 0 λ Ε -4 '2

Για να διέρχεται η εφαπτόμενη y = λχ - ../9λ2 + 4 , από το σημείο Μ (1 , 3) πρέπει και αρκεί, 3 = λ - .J 9λ2 + 4 ( 2) .

Αλλά ( 2 ) <::::> λ- 3 = ../9λ2 + 4 <::::>

<::::> { 2 λ

� 3

<::::> { {

λ �/ 1 } 8λ + 6λ - 5 = 0 λ ε -4 ,.2 ,

τιμές που είναι, προφανώς απορριπτέες. Τελικά οι ζητούμενες εφαπτόμενες είναι:

1 5 5 1 7 ε1 : y = -x + - , ε2 : y = --x +-. 2 2 4 4 Π αρατήρηση : Εργαζόμενοι ομοίως για τις

σχετικές θέσεις υπερβολής χ: -� = 1 και ευθείας α β

ε : α,χ+f3ιy+γ1 =0, 13ι :;eΟ<::>y=λχ+κ, όταν λ=�, κ=_ll , Ι3ι Ι3ι

παίρνουμε: i) «Η ευθεία τέμνει την υπερβολή σε δύο σημεία» <::::> κ2 > α2λ2 - β2 :;t: Ο και σε ένα σημείο όταν

κz > α2λz - β2 = Ο. ii) «Η ευθεία εφάπτεται της υπερβολής» <::::> κz = αzλz - β2 * Ο. iii) «Η ευθεία δεν έχει κοινά σημεία με την υπερβολή» <::::> κ2 < α2λ2 - β2 •

8. Αποδείξτε ότι ο γεωμετρικός τόπος των σημείων Μ από τα οποία άγονται κάθετες

χ2 y2 εφαπτόμενες προς την έλλειψη -2 + -2 = 1,

α β είναι κύκλος. Ο κύκλος αυτός ονομάζεται διευθύνων κύκλος της έλλειψης (dierector circle) ή κύκλος του Monge.

Λύση : ΕυθίJ : Τα σημεία Κ1 (α, β ) , Κ2 ( α, -β ) , Κ3 (-α, β ) , Κ4 ( -α, -β) ,

ανήκουν στον κύκλο ( ο)α2 + β2 ) . 'Εστω τώρα μια από τις εφαπτόμενες της έλλειψης y=λχ±Jα2'Κ +β2 , και μια από τις κάθετες σ' αυτές

1 � εφαπτόμενές της y = -λχ ± να )! + β , λ :;e Ο, που τέμνονται σε σημείο Μ ( χ0 , y0 ) • Τότε έχουμε: { Υο =λχσ ±Jα?Κ +β2 { Υο -λχ0 =±Jα2Κ +β2

1 �1 2 � 1 2 1 2 � Υο =--χο ± α 2+β Υο +-Χο =+ α 2+β λ λ λ λ ΕΥΚΛΕΙΔΗΣ Β ' 83 τ.3/49

Page 52: Ευκλειδης Β 83

------------- Μαθηματικά για την Β ' Λυκείου ------------­(Υο - λχο )2 = (±�αzλz + βz )z

(λy,;χ, )' = (�α' λ� + β' )' �

{io +Κ� -2λχσΥσ =<iΚ +Jf ;; (Κ + ι) ,ς +(Κ + ι)iο = Κiο +,ς +2λχσΥο =<i +Κif

= (λz + l)αz + (λz + l)βz => χ� + Υ� = αz + βz =>

=>,ς +iο =�<i +β2 =>Me(C), όπου c: (ο.�α2 +β2 ) .

ο Θα αποδείξουμε τώρα ότι κάθε σημείο Μ ( χ0 , Υ ο ) του κόκλου C, διαφορετικό των σημείων

Κ1 (α, β) , Κ2 ( α, -β) , Κ3 (-α, β) , Κ4 ( -α,-β) , έχει την ιδιότητα: οι εφαπτόμενες προς την έλλειψη από αυτό

είναι κάθετες. 'Εστω ε : y = λχ + �α2λ 2 + β2 , λ :#: Ο. μία

εφαπτόμενη της έλλει ς διερχόμενη από το Μ.

:

/ I ,(0.-β)

. - - L - -Τότε θα ισχ6ει: Υο = λχ0 + �α2λ2 + β2 , λ :#: Ο (l) . Θα

αποδείξουμε ότι ευθεία (δ): y = _.!_χ + κ κάθετη της (ε) λ στο Μ είναι εφαπτόμενη στην έλλειψη. Αρκεί λοιπόν

κ2 = α2 �+ β2 . Προφανώς ισχ6ει: Υ ο = _.!_χ0 + κ (2) . λ λ (1) => Υ� + λ2χ� - 2λχ0y0 = α2λ2 + β2 , ( 2) => λ 2y� + χ� + 2λχ0y0 = λ 2κ2 , οπότε με πρόσθεση

κατά μέλη παίρνουμε: 1 ( ι +λz ) ( αz +βz ) =αzλz +βz +λzκz => .. . => κz = α2 λ2 +β2 .

χ2 ΥΖ Δίνεται η έλλειψη C : - + - = 1 και οι

αz βz

αz ευθείες χ = ± -, με γz = αz - βz (που καλούνται

γ

διευθετούσες της). Αποδείξτε ότι ο λόγος των αποστάσεων του τυχόντος σημείου Μ ( χ0 , y 0 ) της έλλειψης από την εστία Ε(γ,Ο) και από την

αz ευθεία χ = -, είναι ίσος με την εκκεντρότητα

γ της έλλειψης και αντιστρόφως (το ίδιο

συμβαίνει και για τον λόγο των αποστάσεων του σημείου Μ από την εστία Ε '(-γ,Ο) και την

αz ευθεία χ = --).

γ Αν Ε'(-γ,Ο), έχουμε: ' 2 ( )2 2 2 ( )2 2 ΕΜ = Χ0 + γ + y0 , ΕΜ = Χ0 - γ + y0 =>

=> Ε'Μ2 -ΕΜ2 = 4γχ =>Ε' Μ-ΕΜ = 4ΎΧο => 0 Ε'Μ + ΕΜ ΕΜ-(2α-ΕΜ} = 2ΎΧο =>ΕΜ=α+ ΎΧο ,ΕΜ=α-ΎΧο => α α α

α2 α2 => -- < χ0 <- και

γ γ

ΜΚ = � - χ0 => : = (α-:0 ) { � - χ0 ) =� = ε.

Έστω τυχόν σημείο Μ ( χ0 , Υ ο ) του

επιπέδου που απέχει από την εστία Ε(γ, Ο) και την

ri ΕΜ γ ευθεία χ=- , αποστάσεις ΜΕ, ΜΚ, ώστε: -- = - . γ ΜΚ α

, ΕΜ γ /( )2 2 γ ( α2 ) Τοτε: -- = -=> ν χ - γ + y =- -- χ => ΜΚ α α γ

=> (χ -γ)2 +I = (α-Ιχ)2 => χ: +� = l=>Με( c) . ε α β

ΧΖ y2 Δίνεται η υπερβολή -- - = 1 και οι

αz βz

αz ευθείες χ=±-, με γ2 = α2 + β2 (που καλούνται

γ διευθετούσες της). Αποδείξτε ότι ο λόγος των αποστάσεων του τυχόντος σημείου M(x,y) της υπερβολής από την εστία Ε(γ,Ο) και από την

θ ,

<i , ,

, ευ εια χ=-, ειναι ισος με την εκκεντροτητα γ

της υπερβολής και αντιστρόφως (το ίδιο συμβαίνει και για τον λόγο των αποστάσεων του

2 σημείου Μ από την Ε'(-γ,Ο) και την χ=-!_).

γ Εργαζόμαστε ακριβώς όπως στο

προηγούμενο θέμα χρησιμοποιώντας τους αντίστοιχους από την θεωρία τύπους.

Να βρεθούν οι εφαπτόμενες της

υπερβολής: C: i _Υ =1, από το σημείο Μ{1,3) . 25 9 Θεωρούμε Μ { γ,Ο) , Ν (Ο,δ) , με γ, δ ε Ζ,

τα δυο σημεία τομής της εφαπτομένης της γραμμής β2Χ2 - α2y2 = α2β2 ' α, β Ε z• ( * ) , με

τους άξονες. Να αποδειχθεί ότι ο αριθμός

ΕΥΚΛΕΙΔΗΣ Β' 83 τ.3/50

Page 53: Ευκλειδης Β 83

------------- Μαθηματικά για την Β' Λυκείου -------------

( α4 + γ4 ) { β4 + δ4 ) αναλύεται σε άθροισμα ενός

τετραγώνου ακεραίου αριθμού και μίας τέταρτης δύναμης ακεραίου αριθμού.

Ν α βρεθεί μία ακέραια λύση της Διοφαντικής εξίσωσης: χ2 + y4 = 337 · 881 (ι) .

ΛΊ!Η:rη : α) Οι ζητούμενες εφαπτόμενες είναι της

μορφής y=λχ±.J25λ2 - 9 . Για να διέρχεται κάποια από αυτές από το σημείο Μ(1, 3) πρέπει και αρκεί

3 = λ ± .J 25λ2 - 9 . (που ισοδυναμεί με την εξίσωση 4λ2 +λ - 3=0, με λ � 3 ή λ s 3 . . . ). Δεκτές

3 τιμές του λ είναι οι λ1 = - 1 και λ2 = 4" , οπότε οι

ζητούμενες εφαπτόμενες είναι: 3 3J2Ϊ ε1 :y= - χ+4, ε2 :y=-χ+--. 4 2

β)Κατ' αρχήν παρατηρούμε ότι οι αριθμοί γ, δ είναι διάφοροι του Ο (μηδενός). Η γραμμή (*) είναι

2 2 υπερβολή, αφού ισοδύναμα γράφεται ..;.- -� = 1 .

α β Έστω ότι η εφαπτομένη της υπερβολής που

διέρχεται από τα σημεία Μ(γ, Ο) , Ν(Ο, δ) , εφάπτεται σε αυτή σε σημείο της Σ (χ0 , y0 ) . Η

εξίσωση της εφαπτομένης αυτής στο Σ( χσ,y0) είναι ε : β2χοχ-α2yοy= α2β2 . Μ ε ε� β2χογ = α2β2 �

α2 Ν 2 δ 2β2 β2 χ0 =-, ε ε � -α Υο = α � Υο = --. γ δ

Από την άλλη μεριά η εφαπτομένη αυτή ευθεία (ε) ως διερχόμενη από τα σημεία Μ,Ν θα

έχει εξίσωση �+χ_ = 1 . γ δ

Σε( ε)� Χο + Υο = 1� α2 _ β2 = 1�α2δ2 -βγ =fδ2 (i) . γ δ i δ2

Γνωρίζουμε ότι (ταυτότητα του Lagrange ) :

( α4 + γ4 ) (β4 + δ4 ) _ ( α2β2 + γ2δ2 )2 =

2 ( i ) (α2δ2 - β 2γ2 ) =γ4δ4 => (α4 + γ4 ) (β 4 + δ4 )=

=(α2β 2 + γ 2δ 2 )2 + γ4δ4 =(α2β 2 + γ 2δ 2 )2 +(γδ )4 . γ) Με αναφορά στην υπερβολή (C) και την

εφαπτομένη ε1 (ερώτημα (Α) που την βλέπουμε ως

ε1 : � +χ_ = 1 ), έχουμε: α=5 , β=3, γ=δ=4, οπότε: 4 4 ( Ι) <::> χ2 + y4 =(54 +44 )( 34 +44 ) επομένως λόγω του

(Β) παίρνουμε: x2+y4=(52 · 32 +42 - 42 )2 + ( 4 · 4)4 ,

που μας οδηγεί στην διαπίστωση ότι μία λύση είναι η (χ1 ,y1 ) , με χ1 = 25 · 9 + 1 6 · 1 6 = 48 1, y1 = 16.

� 2 , Δίνονται τα σημεία Β, Γ {Β :f!: Γ) .

Θεωρούμε ότι σημείο Α κινείται, ώστε να απέχει από την ευθεία ΒΓ σταθερή απόσταση d>O. Βρείτε τον γεωμετρικό τόπο του ορθοκέντρου Η του τριγώνου ΑΒΓ.

Ι: κ{:ψΙi ' Σε τέτοιες περιπcώσεις επιλέγουμε κατάλληλο σύστημα αξόνων, φροντίζοντας κατά το δυνατόν, τα περισσότερα από τα δεδομένα σημεία να ευρίσκονται πάνω σε αυτούς τους άξονες και αυτό για να aπλοποιήσουμε το υπολογιστικό μέρος.

Ε�ουμε ορθοκανονικό σύστημα αξόνων Ο.χ y, όπου Ο το μέσο της ΒΓ, το σημείο Γ επί του ημιάξονα Οχ και το σημείο Α στο ημιεπίπεδο {x'x,Oy) . Αν θεωρήσουμε ΒΓ=2α, θα έχουμε: Β( --α, Ο), Γ( α, Ο). Θεωρούμε επίσης ότι Α(λ,d). Οι εξισώσεις των υψών ΑΔ, ΒΕ είναι χ=λ και ( α - λ) χ - d · y + α( α - λ) = Ο, αντίστοιχως. Έτσι το ορθόκεντρο Η θα έχει ως συντεταγμένες τις λύσεις του συστήματος των εξισώσεων των υψών ΑΔ, ΒΕ. Συνεπώς για να βρούμε τον γεωμετρικό τόπο του ορθόκεντρου Η ( χ0 , y 0 ) , θα βρούμε την ικανή και αναγκαία συνθήκη μεταξύ των χ0 , y 0 ώστε το

σύστημα: {(α - λ) χ0 _ :;0=+λα(α - λ) = Ο Να είναι συμβιβαστό ως προς λ. Για αυτό

πρέπει και αρκεί (α- χ0 ) χ0 -dy0 + α(α- χ0 ) = 0,

δηλαδή y0 =_!.� +ι( (1) . Προφανώς (1) <=>Η ε ( C) , d d

όπου (C) η παραβολή με εξίσωση y = _.!_χ2 +�, d d που είναι ο ζητούμενος γεωμετρικός τόπος.

ψ iO.•J) (λ.d)

/

� � ιι.μυηΊ ιημπ];fο πρσηyσύμενο θέμα 12 δεν μπορεί να

ανnμετωπιστεί με μεθόδους Ευκλεiδεαις Γεωμετρiας καθότι ο γεωμετρuώς τόπος είναι παραβοΑή και &1). ευ&iα (ή τμήμα της) είtε κύκλος (ή τμήμα του) 1WU συνήθως μας οδηγεί η Μd!οδολοyία επίΛοοης με βάση την Ευκλείδεια Γεωμεφία.

Γη·ικi: ϊrιφυηΊι•ηση :Επισημάνθηκε κmά την διαδοαισία επίΛοοης των αντίσtοwον θεμάτων, η αναyκωότητα rου Ανnσrρόφου στην περiπτωση της «wrόφασηφ Ύtα το αν μiα γριηψή είναι Γεωμετρuώς τόπος σημείων με συyκειφψένη ιδώτητα. Αυτό έγινε με την εmσήμανση της Μαθηματικής διαδuαισίας Ευθύ - Αντίσφοφο όπω; έγινε σrο θέμα 8 ή μέσω της Μαθηματικής διαδuαισίας της εξασφάλισης ικανής και αναyκαίσς σιΜ}ήιcης όπω; έyινε και σrο θέμα 12.

ΕΥΚΛΕΙΔΗΣ Β' 83 τ.3/51

Page 54: Ευκλειδης Β 83

------------- Μαθηματικά για την Β' Λυκείου -------------

Η σημασία της Διοφαντικής εξ ίσωσης αχ+βψ=γ γ ια την επίλυση τρ ιγωνομετρ ι κών εξ ισώσεων που

, , , , *

περ ι εχουν εφαπτομενες η συνεφαπτομενες Γεώργιος Τασσόπουλος Σχολικός Σύμβουλος Μαθηματικών

Αν λάβουμε υπ' όψη μας την τριγωνομετρική ταυτότητα εφθ = σφ (% - θ J για κάθε θ :;t:κπ+Ξ , κ Ε Ζ

τότε αντιλαμβανόμαστε ότι οι εξισώσεις τις οποίες τελικά αρκεί να μελετήσουμε είναι της μορφής εφf (χ ) = εφg (χ ) (I) ή σφf (χ ) = σφg (χ ) .

Κατά την επίλυση της εξίσωσης ( 1 ), υπάρχει κίνδυνος να οδηγηθούμε σε λάθος, αν τη θεωρήσουμε, όπως συνήθως γίνεται (κακώς), ισοδύναμη με την εξίσωση : f( χ) =κπ+g( χ) , ΚΕΖ (Π). Συμβολικά : (Ι) <::::> (Π) . • Είναι λοιπόν απαραίτητο να aποσαφηνίσουμε αρχικά την έννοια της ισοδυναμίας δυο εξισώσεων.

Για παράδειγμα επιλύοντας τις εξισώσεις (ε, ) : ημχ = συνχ και ( ε2 ) : εφχ = 1 βρίσκουμε ότι έχουν το ίδιο σύνολο λύσεων, δηλαδή L1 = L2 = {χ Ε JR I χ = κπ + � , κ Ε Ζ} = L (τον τρόπο λύσης τους παραθέτουμε λίγο πιο κάτω για να αναφερθούμε εκτενώς σε μερικά επίμαχα σημεία) . Οι εξισώσεις εξάλλου ( ε2 ) : εφχ = 1 και ( ε3 ) : ημχσυνχ = συν2χ έχουν διαφορετικά σύνολα λύσεων, αφού : L3 = {χ Ε JR I χ = κπ +% ή χ = κπ +� , κ Ε Ζ} :;t: L2 .Θα συμπεράνει λοιπόν κάποιος (κακώς), ότι οι (ε ι ) , (ε2) είναι ισοδύναμες, ενώ οι (ε2), (ε3) δεν είναι ισοδύναμες. Η άποψη όμως αυτή είναι επιπόλαιη διότι δεν αποσαφηνίζει σε ποιο σύνολο θεωρούμε τις ισοδυναμίες: (ε ι )<=:>(ε2), (ε2)<=:>(ε3) αφού οι εξισώσεις (ε ι ), (ε3) έχουν σύνολο ορισμού το Α = JR ενώ η (ε2) το Β = JR - {χ Ε JR I χ = κπ + %, κ Ε Ζ} . Εξετάζοντας λοιπόν προσεκτικότερα το θέμα θα διαπιστώσουμε ότι ούτε για τις (ε ι ), (ε2) αλλά ούτε και για τις (ε2), (ε3) μπορούμε να aποφανθούμε για το αν είναι ή όχι ισοδύναμες αν πρώτα δεν ξεκαθαρίσουμε σε ποιο σύνολο θεωρούμε ορισμένες τις εξισώσεις και τις ισοδυναμίες, αφού οι εξισώσεις δεν έχουν το ίδιο σύνολο ορισμού, όπως προαναφέραμε. Το σωστό λοιπόν είναι να πούμε ότι και οι δυο ισοδυναμίες έχουν νόημα στο D = Α n Β = Β :;t: 0 και όχι γενικά στο JR , αφού στο JR δεν ορίζονται όλες αυτές οι εξισώσεις. Στο D λοιπόν μπορούμε να πούμε ότι: η πρώτη ισοδυναμία (ει )<=:>(ε2) είναι αληθής, ενώ ή δεύτερη (ε2)<=:>(ε3) είναι ψευδής. Γενικά λοιπόν, αν δύο εξισώσεις (ε ι ), (ε2) έχουν σύνολα ορισμού D, , D2 με D, n D2 = D :;t: 0 , τότε το D είναι το ευρύτερο υποσύνολο του JR στο οποίο μπορούμε να ορίσουμε την έννοια της ισοδυναμίας μεταξύ των (ε ι ), (ε2) . Θα λέμε λοιπόν τότε ότι η εξίσωση (ε1) είναι ισοδύναμη της (ε2) (Συμβολικά: (ε1)<=:>(ε2)), όταν και μόνον έχουν το ίδιο σύνολο λύσεων στο D = D1 n D2 • (εκτός αν από κάποιο περιορισμό προκύπτει ότι: χ Ε D ' c D ).

• Για παράδειγμα: Η εξίσωση (ε, ) : (χ - 1 ) (χ - 2 ) (-2- - 1J = Ο είναι ισοδύναμη της χ - 3 ( ε2 ) : (χ - 1 ) (χ - 2) (-2- - 1] = Ο , στο σύνολο D = D, n D2 = JR - {3, 5} με κοινό σύνολο λύσεων το 5 - χ L = { 1 , 2} , παρότι η (ει ) με σύνολο ορισμού το D1 = JR - {3} έχει σύνολο λύσεων το L1 = {1 , 2, 5} και η (ε2) με σύνολο ορισμού το D2 = JR - { 5} έχει σύνολο λύσεων L2 = {1 , 2 ,3} , δηλαδή L, :;t: L2

* Αφορμή για το άρθρο αυτό στάθηκαν κάποιες ερωτήσεις συναδέλφων από το Πειραματικό Λύκειο του Πανεπιστημίου Πατρών στο Καστρίτσι, για τις οποίες και τους ευχαριστώ.

ΕΥΚΛΕΙΔΗΣ Β ' 83 τ.3/52

Page 55: Ευκλειδης Β 83

-------------- Μαθηματικά για την Β ' Λυκείου -------------.. Εξάλλου η εξίσωση ( ε3 ) : ( χ - 2 ) ( χ - 3) (-1-- 1J = Ο έχει στο σύνολο ορισμού της D3 = JR - { 4} ως χ - 4

σύνολο λύσεων το L3 = {2, 3, 5} και η ( ε4 ) : ( χ - 2) ( χ - 3) (-1-- 1J = ο έχει στο σύνολο ορισμού 6 - χ της D4 = 1R - {6} ως σύνολο λύσεων το L4 = {2, 3 ,5 } = L3 , όπως ακριβώς οι αρχικές εξισώσεις : ημχ = συνχ και εφχ = 1 . Είναι όμως απαράδεκτο να πούμε ότι η (ε3) στο D3 είναι ισοδύναμη της (ε4) στο D4 -:ι:. D3 , αφού μία

σχέση μεταξύ των ( ε3), ( ε4) πρέπει να θεωρείται αληθής ή ψευδής σε κάποιο σύνολο στο οποίο θα έχουν νόημα και οι δύο εξισώσεις. Εν προκειμένω οι ( ε3), ( ε4) είναι ισοδύναμες στο D = D1 n D2 = JR - { 4, 6} ή σε κάποιο υποσύνολο του D γενικότερα.

Μόνον όταν οι εξισώσεις: (ε1), (ε2), • • . , (εv) θεωρήσουμε ότι έχουν σύνολο ορισμού το D = D1 Γ\ D2 Γ\ . . . Γ\ Dv -:ι:. 0 , μπορούμε να πούμε ότι ικανοποιούν ή όχι τη σχέση

(ε i ) <=> (ε j ) , i , j ε { 1 , 2, . . . , ν} , η οποία πλέον αποδεικνύεται ότι είναι σιέση ισοδυναμίας (Ανακλαστική-

Συμμετρική-Μεταβατική), στο D και όχι θεωρώντας κάθε μία ορισμένη στο αντίστοιχο σύνολο Di ,Dj Εντελώς ανω..σ,α είναι λάθος να γράψουμε κάτι αντίστοιχο για ανισώσεις με διαφορετικά σύνολα ορισμού.

()) Για παράδειγμα γράφεται (κακώς) : χ - 1 > Ο <:::::> ( χ - 1 ) ( χ - 2) > Ο , με την εντύπωση ότι, στην χ - 2

περίπτωση αυτή, δεν ενδιαφέρει σε ποιο σύνολο ορίζεται η ισοδυναμία, επειδή η πρώτη στο σύνολο ορισμού της D1 = JR - { 2} έχει ως σύνολο λύσεων το L = ( -οο, 1 ) υ ( 2, +οο) και η δεύτερη στο σύνολο ορισμού της D2 = JR έχει επίσης ως σύνολο λύσεων το L. Όμως μια τέτοια ισοδυναμία, όπως ορίσαμε προηγουμένως, έχει νόημα μόνο στο D = D1 n D2 = JR - { 2} Εξάλλου γράφεται (καλώς), ότι: Στο D = D1 n D2 = JR - {2} ισχύει: χ - 1 � 0 <=> (χ - 1) ( χ - 2) � 0 , χ - 2 αφού οι ανισώσεις αυτές με κοινό σύνολο ορισμού το D έχουν κοινό σύνολο λύσεων το L = ( -οο, 1] u ( 2, +oo) . Ανάλογη παρατiΗJηση μπορεί να γίνει και για τα ισοδύναμα κλάσματα

χ - 1 1 1 χ - 2 Για παράδειγμα έχουμε: ( ) rv - στο D1 = Ζ - {0, 1} και - rv ( ) στο D2 = Ζ - {0, 2} . χ χ - 1 χ χ χ χ - 2 Για να ισχύει όμως η μεταβατική ιδιότητα, δηλαδή να είναι και χ - 1

χ ( χ - 1 ) χ - 2 ( )

(πράγμα χ χ - 2 απαραίτητο προκειμένου να είναι η σχέση «�», σχέση ισοδυναμίας )πρέπει και αρκεί η σχέση «�» να ορίζεται στο D = D1 n D2 = Ζ - {Ο, 1 , 2} . Διαφορετικά είναι παράλογο να μιλάμε για ισοδυναμία κλασμάτων όταν π.χ θεωρούμε ότι τα προηγούμενα κλάσματα είναι ισοδύναμα ανα δύο αλλά δεν είναι

χ - 1 και τα τρία ισοδύναμα. Πολύ μάλλον είναι αδιανόητο να πούμε ότι το κλάσμα ( ) ορισμένο στο χ χ - 1 Α = Ζ - {Ο, 1} είναι ισοδύναμο του _!_ ορισμένου στο Β = Ζ - {Ο} . χ

• Ας ασχοληθούμε τώρα με την επίλυση των εξισώσεων ( ε1 ) : ημχ = συνχ και ( ε2 ) : εφχ = 1 ως πρώτο βήμα για να επισημάνουμε τους κινδύνους που κρύβει η γενική εξίσωση του τίτλου : εφf ( χ ) = εφg (χ ) . Η επίλυση της ( ε 1 ) μπορεί να γίνει τουλάχιστον με τρεις τρόπους:

1 "c ψίιπος : Στο σύνολο ορισμού της Α= R έχουμε:

Ώς γνωστόν, αν α, β, γ, δ ε Ζ και β · δ -:ι:. Ο , τότε : � rv l <::::> α · δ = β · γ β δ ΕΥΚΛΕΙΔΗΣ Β ' 83 τ.3/53

Page 56: Ευκλειδης Β 83

-------------- Μαθηματικά για την Β' Λυκείου ------------­(ε1 ) <=> ημχ = συνχ <::::> συν( π - χ) =συνχ <=> π - χ = 2κπ + χ ή �- χ = 2κπ - χ 2 2 2

π ' π π π <=> 2χ = 2 - 2κπ η Ο · χ = 2κπ -2<=> χ = -κπ +4<=> χ = κπ +4 , κ ε Ζ .

Παρατηρήστε ότι η εξίσωση Ο· χ=2κπ-� είναι αδύνατη αφού 2κπ-� :;t:Ο , δηλαδή κ:;t:_!_ για κάθε κ�. 2 2 4

Επίσης στο σύνολο ορισμού της A=R έχουμε:

( ε1 ) <::::> ημχ -εφ�συνχ=Ο<::::>ημχσυν�-συνχημ� = Ο<::::> ημ( χ -�) = Ο<::::> χ -� =ΚΠ<=>χ =ΚΠ+� , κ ε Ζ : (ο mo συνηθισμένος)

Στο σύνολο ορισμού της A=R εξετάζουμε δύο περιπτώσεις:

ιι) Αν συνχ=Ο , δηλαδή χ =κπ+�, κ εΖ , τότε ημχ =±l , οπότε η (ε1) δεν ισχύει. 2

Αν συνχ:;t:Ο, δηλαδή χ:;t:κπ+�, κ εΖ , τότε: (� )<=> ημχ = l<::::>εφχ= 1<::::>εφχ=εφ�<=>χ=κπ+�, κεΖ 2 συνχ 4 4

Κάποιος εmπόλαια σκεπτόμενος θα θεωρήσει προφανές το ότι όλες οι τιμές κπ + � είναι δεκτές, 4

αφού θα σκεφτεί ότι: κπ+� :;t:κπ+� για κάθε κ εΖ . Όμως έτσι θεωρεί το κ ίδιο και στα δύο μέλη, δηλαδή 4 2

συγκρίνει ένα στοιχείο του συνόλου L'1 ={χεJR/χ=κπ+� , κεz} με το αντίστοιχο στοιχείο του συνόλου

LΊ ={χ εJR/ χ =κπ+�, κεz} π.χ το }t� με το }t� και όχι δύο τυχαία στοιχεία των συνόλων αυτών. 2 2 4

Κανονικά λοιπόν έπρεπε να συγκρίνουμε δύο τυχαία στοιχεία τους, δηλαδή να εξετάσουμε αν υπάρχουν '71 , π π ( ") Ό ( " ) 1 1 4 4 l 1 , κ1 , Κ2 Ε ΙL..ι ωστε κ1π+-=Κ2Π+- 1 . μως 1 <::::> κ1 +- = κ2 +-<=> κ1 - κ2 = <::::> κ1 -κ2 =- , πραγμα 4 2 4 2 4

αδύνατο στο Ζ , αφού ( κ1 -Κ2) εΖ και _!_ � Ζ . Άρα όλες οι τιμές χ=κπ+�, κ ε Ζ αποτελούν λύσεις της 4 4

(ε1 ) . Τελικά και με τους τρεις τρόπους βρήκαμε ως σύνολο λύσεων της (ε1 ) το L={χεJR!χ=κπ�, κεz} . Για την επίλυση της ( ε2) παρατηρούμε ότι έχει σύνολο ορισμού το

B=JR-{x εJR/ x =κπ+�, κ εΖ} :;t:Α και στο Β έχουμε: (ε2 ) <::::> εφχ = εφ�<=>χ =κπ+�, κ εΖ . 2 4 4

Με τον ίδιο τρόπο που εργαστήκαμε προηγουμένως, διαπιστώνουμε ότι όλες οι τιμές αυτές είναι δεκτές. Άρα το σύνολο των λύσεων της (ε2) είναι το L. Δεν έχει όμως νόημα να πούμε ότι η (ει ) με σύνολο ορισμού το 01 = Α είναι ισοδύναμη της (ε2) με σύνολο ορισμού το 02 = Β, αλλά ότι είναι ισοδύναμες στο 01 n 02 = Α n Β = Β :;t: 0 , όπως προαναφέραμε. Είδαμε προηγουμένως ότι ο έλεγχος της αλήθειας ή όχι της σχέσης (i) μας οδήγησε στον έλεγχο του αν έχει ή όχι η Διοφαντική εξίσωση 4χ -4y= 1 λύση (χ, y) = ( κ1 , κ2 ) • Θα ασχοληθούμε λοιπόν στη συνέχεια με μια πολυπλοκότερη εξίσωση της μορφής εφf (χ) = εφg (χ) για να φανεί η χρησιμότητα της Διοφαντική ς εξίσωσης αχ +β ψ =γ στη διερεύνησή της.

π π

Ν λ θ , ξ' 3χ - 5π χ (I) α :υ ει η ε ισωση: εφ 4 = εφ6 .

'Ε ' f ( ) 3χ - Sπ f ( ) χ ' λα ' χουμε εν προκειμενω: 1 χ = εφ 4 , και 2 χ = εφ6 με συνο ορισμου:

ΕΥΚΛΕΙΔΗΣ Β ' 83 τ.3/54

Page 57: Ευκλειδης Β 83

------------- Μαθηματικά για την Β' Λυκείου -----------­Α1 = {χ ε!R / 3χ�5π * κπ+� { 1) , κ εz} και Α2 ={ χ ε!R/� :;t:κπ+� (2) , κ ε z} .

Προφανώς: ( 1 ) <=> 3χ - 5π :;t: 4κπ + 2π <::::> χ :;t: 4κπ + 7π και (2) <=> χ :;t: 6κπ + 3π , οπότε η (I) έχει 3 3

σύνολο

έχουμε :

ορισμού το D = A1 n A2 = {χ ε !R / χ ::;t:-4�-π + 7

3π και χ :;t: 6κπ + 3π, κ ε z} . Στο

3χ -5π χ 12κπ 15π -- = κπ+-<::::> 9χ - 15π = 12κπ+2χ <=> 7χ = 12κπ+ 15π <::::> χ = -- +-, κ ε Ζ . 4 6 7 7

D λοιπόν

Από τις τιμές αυτές δεκτές είναι όσες ανήκουν στο D. Εξετάζουμε λοιπόν αν υπάρχουν τιμές πού ξ , δ λ δ ' , '71 , 12Κ1Π 1 5π 4Κ2Π 7π , 12ΚιΠ 15π 6 3 ε αιρουνται, η α η αν υπαρχουν κι , κ2 ε ιu ωστε: -- +- = -- +- η -- +- = κ2π + π ,

7 7 3 3 7 7 , , , , λ ' Δ , ξ , 12χ 15 4y 7

(")

12χ 15 6 3 ( " ") ουσιαστικα αν εχουν η οχι υσεις οι ιοφαντικες ε ισωσεις: - +- = -+- ι , - +- = y+ 11

7 7 3 3 7 7

Παρατηρούμε ότι: ( i) <=> 36χ + 45 = 28y + 49 <=> 36χ -28y = 4 <=> 9χ -7y = 1 <=> y = 9χ - 1 . 7

Ως γνωστόν μία και μόνο λύση της (i) προκύπτει για χ ε { 0, 1,2, . . . , 6} . Πράγματι για χ=4 βρίσκουμε y=5 , οπότε όλες οι λύσεις της (i) βρίσκονται, όπως ξέρουμε, από τους τύπους: χ =4+ 7ρ, y=5+9ρ, ρ ε Ζ ,

1 2 ( 4 + 7ρ ) π 1 5π δηλαδή κ1 = 4 + 7 ρ, κ2 = 5 + 9ρ . Οι τιμές λοιπόν Κι = +- = 12ρπ + 9π , απορρίπτονται. 7 7

Εντελώς ανάλογα: ( ii) <=> 12χ + 15 =42y+21 <=> 12χ -42y = 6 <=> 2x - 7y = l <=> χ = 7Υ + 1 . 2

Όμοια για y ε {Ο, 1} έχουμε μία μόνο ακέραια λύση της (ii). Πράγματι για ψ= 1 βρίσκουμε χ=4, οπότε:

( ii) <=>χ = 4 + 7 ρ, y = 1 + 2ρ, ρ εΖ . Βρήκαμε λοιπόν ότι εξαιρούνται οι ίδιες πάλι τιμές Κι = 12ρπ + 9π ενώ περιμέναμε να εξαιρεθούν και κάποιες άλλες mθανόν. Είναι άραγε αυτό συμπτωματικό ή όχι; Φοοικά όχι

διότι: Αν θ 1 =κπ+θ2 , τότε: θι = κ1π+� => θ2 = (κι - κ)π+�=>θ2 = κ2π +� με Κι - κ = κ2 , δηλαδή 2 2 2

Τελικά λοιπόν: Αν θ1 = κπ + θ2 , τότε: θ1 = κ1π + π <=> θ2 = κ2π + π με Κι - κ2 = κ. Κατά συνέπεια, αν 2 2 υπάρχει κ Ε Ζ ώστε f (χ ) = κπ + g (χ ) για κάποιες τιμές του χ Ε IR , τότε γι' αυτές τις τιμές του χ ισχύουν

οι παρακάτω συνεπαγωγές: (r(χ) =λπ+��λΕΑςΖ)=>(g(χ) =μπ+��μ=λ-κ) και

(g(χ) =μπ+��μΕΒςΖ)=>(f(χ) =λπ+��λ=μ+κ} Επομένως, οι απαγορευμένες τιμές του κ που f{ χ) = κπ+g( χ))

προκύπτουν από το σύστημα π (Σ1 ) είναι ίδιες με αυτές που προκύπτουν από το σύστημα f(χ) = λπ

+2

f(χ) =κπ+g(χ)) π (Σ2 ) με λ-μ=κ. Στην προκείμενη περiπτωση βρήκαμε: κ=4+7ρ, λ=5+9ρ, μ=1+2ρ, όπου

g(χ) = μπ+

2 πράγματι: λ-μ=(5+9ρ}-(1+2ρ)=4+7ρ. Βέβαια η μορφή : κ=4+7ρ δεν είναι μόνοσήμαντη. Πιθανόν να έχουμε κ=7-3+7ρ=7(ρ+ 1 )-3=7ρ '-3 , ή κ=14-1 0+7ρ=7(ρ+2}-10=7ρ"-10 κ.λ.π.

Τελικά λοιπόν: ( Ι) <=> χ = 12κπ + Ι Sπ με κ ε Ζ και κ :;t: 4 + 7ρ, ρ ε Ζ . 9 2

Αν λ Ε Α ' = Ζ -Α και μ=λ-κ, τότε f(χ) :;ιι: λπ+� και g(χ) :;ιι: μπ+� 2 2 ΕΥΚΛΕΙΔΗΣ Β' 83 τ.3/55

Page 58: Ευκλειδης Β 83

-------------- Μαθηματικά για την Β ' Λυκείου -------------Επίσης: Αν μ Ε Β ' = Ζ - Β και λ=μ+κ, τότε g ( χ ) ;t. μπ + 2: και f (χ ) ;t. λπ + 2: .

2 2

Τελικά λοιπόν: f ( x ) * λπ +2: και g ( χ ) * μπ + 2: <=> (λ Ε Α' και μ = λ - κ) ή (μ Ε Β ' και λ = μ + κ) <:::} 2 2

(λ Ε Α'και λ - μ = κ) ή (μ Ε Β ' και λ - μ = κ) <:::} (λ Ε Α' ή μ Ε Β ') και λ - μ = κ •

Χρήσιμη εφαρμογή : Στην εξίσωση π.χ. (ε1 ) : εφ χ - π = εφ2: αν παρατηρήσουμε αρχικά ότι: 2: ;t. λπ + 2: 4 3 3 2

' θ λ '71 ' λ 1 ' θ ' ' ' ' λ λ ' ( ') χ - π π '71 ' για κα ε Ε u..- , αφου * -- , τοτε α ειμαστε σιγουροι οτι ο ες οι υσεις της ε1 : -- = κπ + - , κ Ε u..- , ειναι 6 4 3

και λύσεις της ( ε1 ), δηλαδή ανήκουν στο σύνολο ορισμού αυτής D = {χ Ε JR I χ �π * κπ + � , κ Ε Ζ} . Πράγματι αν

' χ - π π ' '71 ' θ ' π ( ) π λ π λ ( ) '71 ' ηταν -- = μπ + - , για καποιο μ Ε u..- , τοτε α ηταν και - = μ - κ π + - = π + - , για = μ - κ Ε u..- , πραγμα 4 2 3 2 2

άτοπο, Οι εξισώσεις λοιπόν ( ε1 ) , ( ε1 ') είναι ισοδύναμες στο D. Έτσι θα μπορούσαν να λυθούν απλούστερα οι εξισώσεις της μορφής εφχ = 1, εφχ = .J3 κ.λπ. Το κακό όμως είναι ότι συνήθως αυτές θεωρούνται ισοδύναμες των

χ = κπ + 2: , χ = κπ + 2: κ.λ.π. χωρίς να έχει γίνει καμία αναφορά στα προηγούμενα συμπεράσματα. 4 3

• Αντιλαμβανόμαστε πλέον ότι η πλήρης λύση των εξισώσεων αυτών οδηγεί στη Διοφαντική εξίσωση αχ + βy = γ . Ως γνωστόν, αν σ' αυτή ισχύει Μ.Κ.Δ (α,β)= 1 και βρούμε μία λύση της

( χ0 , y0 ) , τότε οι άπειρες λύσεις της δίνονται από τους τύπους χ = χ0 - β ρ, y = y 0 + αρ, ρ Ε Ζ και η δυσκολία της είναι ανάλογη του μεγέθους των συντελεστών α,β. Καλό είναι λοιπόν να φροντίσουμε ώστε να μην οδηγήσουμε τους μαθητές σε τέτοιες Διοφαντικές εξισώσεις, αλλά σε μορφές αχ+βψ=γ με Μ.Κ.Δ ( α,β)=ρ> 1 , όπου ο αριθμός l να μην είναι ακέραιος ώστε η εξίσωση να είναι αδύνατη και ρ έτσι να μην απορρίπτεται κάποια από τις πιθανές ρίζες της εξίσωσης που βρήκαμε αρχικά. π.χ 1 2χ - 8y = 6 <=> 3χ - 2y = � <=> 3χ - 2y = � , που είναι αδύνατη, αφού : ( 3χ - 2y) Ε Ζ και � � Ζ . 4 2 2

Η ξ' ' δ ( χ - π ) ( 3π - χ ) ' λ ' δ ' ' ε ισωση για παρα ειγμα εφ -3- = εφ -2- , ευκο α μπορειτε να ιαπιστωσετε οτι μας

οδηγεί στις αδύνατες Διοφαντικές εξισώσεις: 4κ - 1 Ο λ = 1, 6κ + 1 Ο λ = - 1

τ�1 � ' ' ' ' ' λ ' 9χ - 1 ' ' UΙ.Gιωνοντας εmσημαινουμε οτι η ευρεση των ακεραιων υσεων της y = -- μπορει να γινει και 7 στοιχειωδώς, αν λάβουμε υπόψη μας ότι η Ευκλείδεια Διαίρεση χ:7 δίνει χ = 7λ + υ με λ Ε Ζ ,

9 ( 7λ + υ) - 1 9υ - 1 υ Ε { 0, 1 , 2, . . . , 6} , οπότε y = = 9λ +-- και μόνο για υ=4 βρίσκουμε ακέραιες τιμές του 7 7 y τις, y = 9λ + 5, λ Ε Ζ , οπότε χ = 7λ + 4, λ Ε Ζ . Το ότι για μια μόνο τιμή του υ Ε {0, 1 , 2, . . . , 6} , ο

θ , 9υ - 1 · · ' · ' ' δ ' (9 1 ) 7 δ' ' 7 αρι μος -- ειναι ακεραιος, προκυπτει απο το γεγονος οτι οι ιαιρεσεις υ - : ινουν σιγουρα 7 διαφορετικά υπόλοιπα ( φυσικούς αριθμούς μικρότερους του 7), οπότε ένα μόνο απ' αυτά θα είναι μηδέν. Πράγματι, αν με υ1 * υ2 είχαμε τις ταυτότητες διαίρεσης: 9υ1 - 1 = 7 π1 + ρ, 9υ2 - 1 = 7 π2 + ρ , τότε 9 ( υ1 - υ2 ) = 7 ( π1 - π2 ) και αφού Μ.Κ.Δ(7 ,9)= 1 , ο αριθμός 7 θα διαιρεί τον ( υ1 - υ2 ) πράγμα άτοπο, αφού Ο < l υ1 - υ2 1 < 7 . Έτσι οδηγούμαστε με έναν άλλο τρόπο (κατά τη γνώμη μου, πιο φυσιολογικό) στην εύρεση των τύπων χ = χ0 - βρ, y = y0 + αρ, ρ Ε Ζ που δίνουν τις λύσεις της Διοφαντικής εξίσωσης αχ+βψ=γ, όταν φυσικά Μ. Κ. Δ( α, β)= 1 . Η γενίκευση είναι εντελώς ίδια.

• Θυμηθείτε ότι: (p1 Λ q) ν (p2 Λ q) <:::} (p1 ν p2 ) Λ q

ΕΥΚΛΕΙΔΗΣ Β ' 83 τ.3/56

Page 59: Ευκλειδης Β 83

------------- Μαθηματικά για την Γ Λυκείου ------------

Γ ' λΥΚΕΙΟΥ Θέματα Πιθανοτήτων Κονόμης Άρτι - Χαραλαμπάκης Στάθης

Στο άρθρο αυτο εχουμε επιλέξει, μια σειρά από βασικές ασκήσεις από το κεφάλαιο των πιθανοτήτων, για τη Γ τάξη ενιαίου λυκείου . Η πληρότητα και η κάλυψη όλης της ύλης του κεφαλαίου ήταν για μας το ζητούμενο. Έγινε προσπάθεια οι ασκήσεις να είναι όσο γίνεται συνδυαστικές και πρωτότυπες.

Ι ) Έστω Α, Β δύο ενδεχόμενα του δειγματικού χώρου Ω για τα οποία ισχύει:

α) Η πιθανότητα να πραγματοποιηθεί ένα

λ , ' Α Β ' 5 του αχιστον απο τα , ειναι "6

β) Ρ( Α) -:ι:. Ρ( Α-Β). γ) οι αριθμοί Ρ(Α) , Ρ(Α-Β) είναι στοιχεία

του συνόλου {-.!. � � 2} . 3 ' 3 ' 9 ' 6

Ι ) Ν α αποδείξετε ότι τα ενδεχόμενα Α, Β δεν είναι ασυμβίβαστα. 2) Να βρείτε τις πιθανότητες των ενδεχομένων Α και Α-Β. 3) Να βρείτε τις πιθανότητες:

Λύση

ί) Να πραγματοποιηθεί το Β, ίί) Να πραγματοποιηθεί μόνο ένα από τα ενδεχόμενα Α, Β, ίίί) Να μην πραγματοποιηθεί το Α και να μην πραγματοποιηθεί το Β iv) Να μην πραγματοποιηθεί ένα τουλάχιστον από τα ενδεχόμενα Α , Β .

Ι ) α ' τρόπος (Με απαγωγή σε άτοπο) : Αν τα Α,Β ήταν ασυμβίβαστα, δηλαδή AnB=0, τότε P(AnB)=O. Όμως: Ρ(Α)::ι:.Ρ(Α-Β)=> Ρ(Α)-::ι:.Ρ(Α)-Ρ(ΑnΒ)=>Ρ(ΑnΒ)-::ι:-0, άτοπο. β ' τρόπος (Συνθετικά) : Γνωρίζουμε ότι: Χ=0=>Ρ(Χ)=Ο, οπότε με αντίθετοαντιστροφή θα έχουμε: Ρ(Χ)::ι:.Ο=:>Χ::ι:.0.Στην περίπτωση μας έχουμε: P(A)'f':P(A-B)=>P(A)'f':P(A)­P(AnB)=>P(AnB)'f':0=>AnB::ι:.0=:>Tα Α,Β δεν είναι ασυμβίβαστα.

2) Έχουμε: Α-Βς;Α, οπότε Ρ(Α-Β).�Ρ(Α). Άρα , , ' λ { Ι 2 2 7 } απο τις τιμες του συνο ου -- ,- ,- ,-

3 9 3 6

έχουμε: Ρ(Α)=� και Ρ(Α-Β)=� διότι οι τιμές 3 9

Ι 7 Ι -- και - απορρίπτονται αφού το -- < Ο 3 6 3

7 και - > Ι . 6

3) ί) Βρίσκουμε πρώτα την mθανότητα του

ενδεχομένου AnB. Από την υπόθεση έχουμε: Ρ( AuB) =� , άρα: P(B)=P(AuB)-P(A)+P(AnB)

6 Έχουμε: P(A-B)=P(A)-P(AnB)

2 2 4 => - =- - P(AnB) => P(AnB)= - . 9 3 9 Βρίσκουμε την πιθανότητα του Β :

Ρ(ΑυΒ)=Ρ(Α)+Ρ(Β)-Ρ(ΑnΒ) =>Ρ(Β)= 5 2 4 1 Ι Ρ(ΑυΒ)-Ρ(Α)+Ρ(ΑnΒ)=--- + - =-6 3 9 Ι 8

ίί) Το ενδεχόμενο να πραγματοποιηθεί μόνο ένα από τα ενδεχόμενα Α, Β είναι το (A­B)u(B-A). Από την άλλη μεριά ξέρουμε ότι τα ενδεχόμενα Α-Β και Β-Α είναι ασυμβίβαστα. Έτσι έχουμε: Ρ((Α-Β)υ(Β-Α))=Ρ(Α-Β)+Ρ(Β-Α)= =P(A)-P(AnB)+P(B)-P(BnA)=

2 Ι 1 4 7 =P(A)+P(B)-2P(AnB) =- +--2 ·- =- . 3 Ι 8 9 1 8

ί ί ί) Το ζητούμενο ενδεχόμενο είναι A'nB '=(AuB) ' (Κανόνας de' Morgan),

οπότε: Ρ( Α ' nΒ ')= Ι-Ρ(ΑυΒ)= Ι- -2_ = .!_ . 6 6

iv) Το ζητούμενο ενδεχόμενο είναι Α 'υΒ '=(ΑnΒ) ' , (Κανόνας de' Morgan),

οπότε: Ρ(Α 'υΒ ')= I-P(AnB)= Ι -� = 2_ . 9 9

Σχόλιο της Σ. Ε . : Οι τύποι Ρ (Α' nB ') = Ρ ((ΑυΒ) ')

και Ρ (Α' υ Β ') = Ρ ((Α nΒ) ') μπορούν να προκύψουν

και χωρίς τους τύπους του de ' Morgan, φυσικά με δυσκολότερη διαδικασία. Γι ' αυτό αρχικά πρέπει να κατανοήσουν οι μαθητές ότι: Για κάθε ζεύγος συνόλων Χ, Ψ ισχύει: Χ - Ψ = Χ nΨ ' , δηλαδή η πράξη Χ-Ψ

πλεονάζει και θα μπορούσαμε να περιοριστούμε μόνο σε δύο πράξεις, την ένωση και την τομή. Έτσι λοιπόν:

Ρ (Χ -Ψ) = Ρ (Χ nΨ') = Ρ(Χ) - Ρ (Χ nΨ) (1). Από τη σχέση (1) με Χ=Α · και Ψ=Β παίρνουμε:

Ρ ( Α' n B ') = Ρ (Α') - Ρ (Α' n B) =

= 1 - Ρ (Α) - [Ρ (Β) - P (A n B)] =

ΕΥΚΛΕΙΔΗΣ Β ' 83 τ.3/57

Page 60: Ευκλειδης Β 83

------------ Μαθηματικά για την Γ Λυκείου ------------

= 1 - [P (A ) + P ( B) - P ( A n B)] =

= 1 - P (A u B) = P ((A u B) ') (2 ) Στη συνέχεια έχουμε: P ( A' u B ') = Ρ (Α') + Ρ ( Β ') - Ρ (Α' n Β') = (2) =1-Ρ(Α) + 1-Ρ(Β) -[1-Ρ( AuB)J =

=1-[Ρ(Α) + Ρ(Β) -Ρ( AuB)] = 1 -Ρ( AnB) = P((AnB) ') 2 ) Για τα ενδεχόμενα Α ,Β ενός δειγματικού

χώρου Ω ισχύει ότι: P(A)·P(B)=P(AnB). Να αποδείξετε τις παρακάτω σχέσεις:

α) Ρ(Α-Β) = Ρ(Α)·Ρ(Β ') β) Ρ((Α-Β)υ(Β-Α))=Ρ(Α)·Ρ(Β ') +Ρ(Β)·Ρ(Α ') γ) Ρ(Α) + Ρ(Β)� 2Ρ(Α)·Ρ(Β) δ) Ρ((ΑυΒ) ') = Ρ(Α ')·Ρ(Β ')

Λ{Jση α) Έχουμε: Ρ(Α-Β) = Ρ(Α) - Ρ(ΑΓΙΒ) = =Ρ(Α) - Ρ(Α)·Ρ(Β) = Ρ(Α) - ( 1- Ρ(Β)) = Ρ(Α)· Ρ(Β ') β) Τα ενδεχόμενα Α - Β και Β - Α ασυμβίβαστα. Έτσι έχουμε: Ρ[(Α-Β)υ(Β-Α)]=Ρ(Α-Β)+Ρ(Β-Α) = Ρ(Α)-Ρ(Β ') + Ρ(Β)·Ρ(Α ') . , Α ΓΙ Β ς Α} Ρ (Α ΓΙ Β) :ς Ρ (Α)}

γ) Εχουμε: => => Α ΓΙ Β ς Β Ρ (Α ΓΙ Β) :ς Ρ (Β)

=> 2P (A n Β) :ς Ρ (Α) + Ρ (Β) δ) Έχουμε: Ρ[(ΑυΒ) '] = 1 - Ρ(ΑυΒ) = =1 - Ρ(Α) - Ρ(Β) + P(AnB) = =1 - Ρ(Α) - Ρ(Β) + Ρ(Α)·Ρ(Β) = = 1 - Ρ(Α)- Ρ(Β) ·( 1 - Ρ(Α)) = · =( 1 - Ρ(Α)) ( 1 - Ρ(Β)) = Ρ(Α}Ρ(Β ') .

3) Ένας υπάλληλος μιας εταιρείας πηγαίνει στο ταχυδρομείο για να ταχυδρομήσει τρία γράμματα για τρεις διαφορετικές χώρες. Για κάθε γράμμα απαιτείται γραμματόσημο διαφορετικής αξίας. Ο υπάλληλος κόλλησε στην τύχη τα γραμματόσημα. Να βρείτε την πιθανότητα του ενδεχομένου:

α) Κάθε γράμμα να έχει το σωστό γραμματόσημο,

β) Ένα μόνο γράμμα να έχει το σωστό γραμματόσημο,

γ) Ένα τουλάχιστον γράμμα να έχει το σωστό γραμματόσημο,

δ) Κανένα γράμμα να μην έχει το σωστό γραμματόσημο

Λi1ση Αν σημειώσουμε με 1 , 2, 3 τα γράμματα και

Χι , Χ2 , Χ3 αντίστοιχα τις χώρες για τις οποίες προορίζονται, τότε ο συμβολισμός ΙΧι σημαίνει ότι στο γράμμα Ι έχει επικολληθεί γραμματόσημο που αντιστοιχεί στη χώρα Χι :Ετσι, όλες οι δυνατές επικολλήσεις, οι οποίες αποτελούν το

δειγματικό χώρο Ω είναι οι tx;�3Jς,JX;3�2Jς,��3Jς,��IJς,3)\�2Jς,3)\�IJς (Ο συμβολισμός π. χ Ι Χι 2Χ2 3Χ3 δείχνει ότι το γραμματόσημο 1 έχει επικολληθεί για τη χώρα Χ1 , το γραμματόσημο 2 έχει επικολληθεί για τη χώρα Χ2 και το γραμματόσημο 3 έχει επικολληθεί για τη χώρα Χ3}. Έχουμε Ν(Ω)=6. Θεωρούμε τα ενδεχόμενα : Α= {κάθε γράμμα έχει το σωστό γραμματόσημο} Β= {ένα μόνο γράμμα έχει το σωστό γραμματόσημο} Γ= {ένα τουλάχιστον γράμμα έχει το σωστό γραμματόσημο} Δ= {κανένα γράμμα δεν έχει το σωστό γραμματόσημο} .

_ Ν (Α ) Ι α) Α - { ΙΧ1 2Χ2 3ΧΛ και Ρ (Α ) = -- = -Ν (Ω) 6

β) Β = { ΙΧ1 3Χ2 2Χ3 , ΙΧ2 2Χ1 3Χ3 , 3Χ1 2Χ2 ΙΧ3 } Ν (Β) 3 Ι και Ρ ( Β) =--=- =-Ν (Ω) 6 2

γ) Γ={ Ι�2)ζ3Χ3 , Ι�3)ζ2Χ3 ,2�])ζ3Χ3 ,3�2)ζDζ} και Ρ (Γ ) = Ν (Γ ) = � = 3_

Ν (Ω ) 6 3

δ) Δ= {3Χ1 ΙΧ2 2Χ3 , 2Χ1 3Χ2 ΙΧ3 } και Ρ(Δ) = Ν (Δ) = _!_ Ν(Ω) 3

4) Έστω Ω={l , 2, 3 , . . . , 10} δειγματικός χώρος με ισοπίθανα απλά ενδεχόμενα. Θεωρούμε τη συνάρτηση f με τύπο:

f {χ) = _!_χ3 -2χ2 + 3χ+λ\ όπου λ Ε Ω, x E JR . 3

Έστω τα ενδεχόμενα: Α={ λ Ε Ω I το τοπικό

' f ' λ ' ' ' 52 } μεγιστο της ειναι μεγα υτερο η ισον του "3

Β={ λ Ε Ω I το τοπικό ελάχιστο της f είναι μικρό­τερο ή ίσο του 4 } . α) Να μελετήσετε την f ως προς τη μονοτονία και τα ακρότατα. β) Να βρείτε της πιθανότητες των ενδεχομένων Α, Α n Β και Α υ Β. γ) Αν S2 είναι η διακύμανση των τιμών λ, 2λ, 4λ, Sλ όπου λ Ε Ω να βρείτε την πιθανότητα του

ενδεχομένου : Γ = {λ Ε Ω I S2 > 1 0} .

Λ1Jση α) Το πεδίο ορισμού της f είναι όλο το JR . Έχου-με: f ' (x) =x2 -4x +3, οπότε : f '( x) < O<=:>x E ( 1,3) , και f ' ( χ ) > Ο <:::::> χ Ε ( -ω, Ι ) υ (3, +οο) και αφού η f είναι συνεχής στα σημεία Ι , 3 θα έχουμε ft( --α:sΙ] ,

Λ

fl [ Ι, 5) και f t [3, +oo) . Επομένως το f( Ι) =�+λ2 Λ 3

ένα τοπικό μέγιστο και το f ( 3) = λ 2 είναι τοπικό ΕΥΚΛΕΙΔΗΣ Β ' 83 τ.3/58

Page 61: Ευκλειδης Β 83

------------ Μαθηματικά για την Γ Λυκείου ------------ελάχιστο της f. β ) Βρίσκουμε τα στοιχεία του Α από τη σχέση : � + λ2 � 52 ( 1 ) � λ2 � 1 6 � lλ l � 4 � λ � -4 ή 3 3 λ � 4 , οπότε Α = {4, 5 , 6, 7, 8 , 9, 1 0 } . Βρίσκουμε τα στοιχεία του συνόλου Β από τη σχέση : f (3 ) � 4 � λ2 � 4 (2). � λ2 � 4 � 1λ l � 2 � -2 � λ � 2 , οπότε Β= { 1 ,2 } .

'Εχουμε Ρ(Α) =2 και Ρ(Β) =2 =� και AnB=0, 10 10 5 οπότε P(AnB) = Ο. Επίσης έχουμε: ΑυΒ= { 1 ,2,4,5,6,7,8,9, 1 0 } , οπότε: Ρ(ΑυΒ) =_2_ . 1 0 γ ) Βρίσκουμε πρώτα τη μέση τιμή των παρατηρή­σεων λ, 2λ, 4λ, 5λ. Έχουμε : ;z = λ + 2λ + 4λ + 5λ = 1 2λ = 3λ . Οπότε η 4 4 � (λ-3λ)2 +(2λ-3λ)2 +(4λ-3λ)2 +(5λ-3λ)2 10λ2

4 4 . Βρίσκουμε τα στοιχεία του συνόλου Γ από τη σχέση : S2 > 1 0 (3 ) . 'Εχουμε: ( 3 ) � 1 Ολ 2 > 1 0 � 4 �λ2 > 4� 1λl > 2 � λ > 2 ή λ < -2 . Άρα:

8 4 Γ = { 3 , 4, 5 , 6, 7, 8 , 9, 1 0 } και Ρ (Γ) = - = - . 1 0 5 .,; , Έστω Α και Β δύο ενδεχόμενα ενός

δειγματικού χώρου Ω και p ε (0, 1 ). Δίνεται οτι οι Ρ(Α), Ρ(ΑυΒ), P(AnB) είναι ανα δύο διαφορετικές μεταξύ τους και αποτελούν στοιχεία του συνόλου {p-1 , p, p+l , p2, p3}

Να δείξετε οτι Ρ(Α) = p2, Ρ(ΑυΒ) = p και P(AnB)= p3

/') ) Να δείξετε οτι Ρ(Β) = p3 - p2 + p γ) Να δείξετε οτι Ρ(Β-Α) > Ρ(Α-Β)

1' . ' Επειδή : p< l �p- 1 <0 και p>O�p+ 1> 1 , οι τιμές p-1 και p+ 1 δεν αποτελούν τιμή κάποιας πιθανότητας. Επίσης γενικά έχουμε: Ας(ΑυΒ) και (ΑnΒ)ςΑ. Άρα Ρ(Α)�Ρ(ΑυΒ) και P(AnB)�P(A).

Άρα Ρ(ΑυΒ)� Ρ( Α) � P(AnB) και επειδή Ο< p < 1 � p3 < p2 < p, θα είναι: P(AnB)= p3 ,

Ρ(Α) = p2, Ρ(ΑυΒ) = p. ; J i Έχουμε: Ρ(ΑυΒ) = Ρ( Α) + Ρ( Β) - P(AnB), οπότε: Ρ(Β)=Ρ(ΑnΒ)-Ρ(Α)+Ρ(ΑυΒ)= p3 - p2 + p. γ, Είναι Ρ(Β-Α) - Ρ (Α-Β) = =Ρ(Β) - P(AnB) - [Ρ( Α) - P(AnB) ] = =P(B)-P(AnB)-P(A)+P(AnB)=P(B) -Ρ(Α)= =pJ -p2+p-p2=p3 -2p2+p=p(p2 -2p+ 1 )=p(p-1 ?>Ο. Άρα: Ρ(Β-Α) > Ρ (Α-Β). Έστω δειγματικός χώρος Ω που αποτελείται

από 2005 στοιχεία , τα οποία είναι ισοπίθανα . Θεωρούμε τα συμπληρωματικά ενδεχόμενα Α και Α' του

Ω με Ο < Ρ(Α) < 1 . Αν ισχύει:

Ρ (Α) 4 , 9 ( ) + -( ) � 16 , τοτε : Ρ Α' Ρ Α

Να βρείτε το πλήθος των στοιχείων του Α. Αν κάποιο ενδεχόμενο Β του

Ω έχει 1204

στοιχεία, να αποδείξετε οτι τα Α, Β δεν είναι ασυμβίβαστα. .\iHϊiJ

α) Αν Ρ (Α) = χ ε (0, 1 ) τότε: χ>Ο, 1-χ>Ο, οπότε: Ρ(Α) 4 χ 4 , 9-( ') +-(-) = 16 � 9-+-= 16 , εχουμε Ρ Α Ρ Α 1 - χ χ

9 χ2 +4( 1-x)= 1 6[x( l-x)]�x=� �Ρ(Α)=� . Όμως 5 5 Ρ(Α) = Ν(Α) �� = Ν(Α) �Ν(Α) = 802

Ν(Ω) 5 2005 Ν(Β) 1 204 ji ) Ν(Β) = 1 204, οπότε Ρ(Β) = -( -) = --. Ν Ω 2005

Έστω ότι τα ενδεχόμενα Α, Β είναι ασυμβίβαστα. Θα ισχύει τότε: Ρ(ΑυΒ)=Ρ(Α)+Ρ(Β)=3�= :m>>l, 5 � � άτοπο. Άρα τα Α, Β δεν είναι ασυμβίβαστα. :: ; Ι 'ΛI;:;: υ"''- Βρίσκουμε: P(AnB)=P(A)+P(B)-P(AuB)= =� + 1 204 - P (A u B)= 2006 - P (A u B) > O ,

5 2005 2005 ' αφου

2006 Ρ (Α υ Β) � 1 < -- . Επομένως P(AnB),c(), οπότε 2005 AnB;e0, δηλαδή τα Α, Β δεν είναι ασυμβίβαστα.

-- 'Ε Ω Ρ(Α) ι· .Jx + 3 - 2 Ί ) στω Α,Βς με: = ιm

2 και

χ-+Ρ(Ω) Χ - Χ

Ρ(Β) = lim 4y - l

y--+P(A) 8y2 + 2y - 1

ιι) Να βρείτε τις πιθανότητες Ρ(Α) και Ρ(Β) �� ) Να βρείτε τη μέγιστη και την ελάχιστη τιμή των πιθανοτήτων των ενδεχομένων : il ) A n B i O Α υ Β

Γ:Δεν πραγματοποιείται κανένα από τα Α,Β i" } Δ: Πραγματοποιείται ακριβώς ένα από τα Α,Β

Λ ι·;;τη , rx+3-2 1

α ) Εχουμε: 2 = ( � ) και χ -χ χ νχ + 3 + 2 4Υ-1 1 , οπότε: P(A) =lim 1

8)7 +2y-1 2y+1 χ...ι χ{ JX+3+2) 4 '

P(B) = Iim-1- =� y__J_ 2y+ 1 3

4

ΕΥΚΛΕΙΔΗΣ Β ' 83 τ.3/59

Page 62: Ευκλειδης Β 83

------------ Μαθηματικά για την Γ Λυκείου -----------β) i) Ισχύει: (AnB) ς Α και (AnB) ς Β. Από την πρώτη σχέση έχουμε: O�P(AnB)�P(A) = ..!. και

4

από τη δεύτερη O�P(AnB)�P(B) = � . Τελικά 3

λοιπόν: Ο � Ρ (Α n Β ) � ..!. . Όταν τα ενδεχόμενα Α, 4

Β είναι ξένα μεταξύ τους, τότε P(AnB ) =Ο και αυτή είναι η ελάχιστη τιμή για την πιθανότητα P(AnB). Αν AnB = Α, δηλαδή Α ς Β τότε η μέγιστη τιμή της πιθανότητας του ενδεχομένου AnB είναι Ρ( Α) = ..!. .

4

i i) Από το ερώτημα i) έχουμε Ο � P(AnB) � ..!. . 4

Για το ενδεχόμενο Α υ Β έχουμε: 1 2 1 1

Ρ(ΑυΒ)= = - + - - Ρ (Α n Β) = =--χ με 4 3 12

P (A n B ) = χ ε [ 0,�] . Αλλά [ 1 ] 1 1 1 1 1 1 1 2 1 1

χ ε 0'4 � 12 -4 �

12 - χ �

12 �3 � Ρ(ΑυΒ) �

12

, , Ρ (Α ) 1 1 , με μεγιστη τιμη της υ Β την - οταν

1 2

Α n Β = 0 , οπότε χ=Ο και ελάχιστη τιμή την � , 3

όταν Α ς Β , οπότε χ = � , δηλαδή Α υ Β = Β .

ί ί ί)Έχουμε: Γ = A 'nB ' = (Α υ Β) ' , οπότε Ρ (Γ) = 1 - Ρ (Α υ Β) = 1 -y και y ε [% , :�]

2 1 1 1 1 � 1 - - � 1 - y � 1 - - � - � Ρ (Γ ) � -

3 1 2 3 1 2 Όπως και στο ερώτημα i ) αποδεικνύεται ότι

Ρ (Α'nΒ ')=Ρ(Γ) γίνεται ελάχιστη με τιμή _.!_ αν 1 2

AnB=0 και γίνεται μέγιστη με τιμή ..!. αν AnB=A. 3

ίv)Έχουμε: Δ = (Α - Β) υ (Β - Α), με (Α-Β) και (Β-Α) ασυμβίβαστα. Άρα: Ρ(Δ)=Ρ((Α-Β)υ(Β-Α))=Ρ(Α-Β)+Ρ(Β-Α)= =Ρ (Α) - Ρ (Α n Β) + Ρ(Β) - Ρ (Α n Β) =

= 1 � - 2Ρ (Α n Β ) = : � - 2χ και χ ε [Ο,�]

5 1 1 1 1 5 1 1 ' � - � - - 2χ � - �- � Ρ(Δ) � - . Επομενως 1 2 1 2 1 2 12 12

η Ρ(Δ) παίρνει μέγιστη τιμή την .!_!_ για AnB=0 1 2

και ελάχιστη τιμή την _2._ για Α n Β =Α. 1 2

8) Για τα ενδεχόμενα του δειγματικού χώρου Ω ισχύουν: (Ρ(Α))2+2 (Ρ(Α) ')2=3 Ρ(Β) και

Ρ(Β)=-3χ2+2χ-.!. , χ ε IR . Να βρεθούν: 9

α) i) Η μέγιστη τιμή του Ρ(Β) i i) Οι πιθανότητες Ρ(Α) και Ρ(Β) i i i) Οι τιμές του χ

β) Αν επιπλέον ισχύει Ρ(Α-Β) = � , τότε να 9

βρεθεί η πιθανότητα: i) Να πραγματοποιηθεί μόνο το Β i i) Να πραγματοποιηθεί ένα μόνο από τα Α, Β iii) Να μην πραγματοποιηθεί κανένα από τα Α, Β Λύση

α) i ) Αν f (x ) = -3x2 + 2χ -� , τότε

f (χ ) = -6χ + 2 και f (χ ) > Ο � χ < _!_ , ενώ 3

f (χ ) < Ο � χ > _!_ . Λόγω της συνέχειας της f 3

θα έχουμε: f!( -oo,l] και f J [l , +oo} οπότε μέγιστη τιμή της Ρ(Β) είναι η

r(l) = -3 ·� +% -� = -� +% =% · i i) Αν Ρ ( Α ) = y τότε: (Ρ(Α))2 +2 (Ρ(Α) ')2 = 3 Ρ(Β)

� γ +2( 1 -y)2 � 3 ·%� (y �)2 �O� y =%� P(A) =% ·

Αλλά 3 Ρ(Β) = (Ρ(Α))2+2(Ρ(Α) ')2= ( 2 )2 ( 2 )2 2 2 3 +2 1 - 3 =3 . Άρα Ρ(Β) = 9 .

i i i) Ρ(Β)=3 �f( χ) =3 <:::>-3χ2+2χ-_!_ =3 � χ = _!_ . 9 9 9 9 3

β) Εχουμε: Ρ( Β-Α) = P(B) - P(AnB) =� -P(AnB) .

Αλλά: Ρ( Α - Β) = Ρ (Α) - P(AnB) 5 2 2 5 6 5 1

�9 =3 -Ρ( AnB) �Ρ( AnB) =3 -9 =9 -9 =9 ' 2 1 1 7 Άρα: Ρ (Β - Α) = 9 - 9 = 9 και Ρ (Α υ Β ) = 9 .

ίί)Έστω Γ= (Α - Β) υ (Β -Α) .

Οπότε, Ρ (Γ) = Ρ (Α - Β) + Ρ (Β - Α) =( . . . )=� 3

i i i) Ρ (A 'nB ')=P ((Α υ Β) ')= 1 - Ρ (ΑυΒ) = � 9

ΕΥΚΛΕΙΔΗΣ Β' 83 τ.3/60

Page 63: Ευκλειδης Β 83

------------- Μαθηματικά για την Γ Λυκείου -------------

Η ανισότf/τα οτα προβΛήματα Τf/ς ανάΛUσf/ς ως δεδομένο ή ως (f/τούμενο

Θανάσης Π. Χριστόπουλος, Παναγιώτης Π. Χριστόπουλος

Α. Η ανισότητα ως δεδομένο • Αν f(x):Sg(x) κοντά στο χ0, και lim f(x) =+οο, Όταν σε μια άσκηση, ως δεδομένο, είναι μια σχέση

ανισότητας ή ανισοϊσότητας τότε μπορούμε να ακο­λουθήσουμε μια συγκεκριμένη πορεία-μέθοδο για τη λύση του προβλήματος. Με τα παρακάτω παραδείγματα θα δούμε μερικές διαφορετικές περιπτώσεις αξιοποίη­σης μιας δοσμένης ανισότητας. 1 . Ανισότητα σε συναρτη σιακή σχέση

Συνήθως ενεργούμε ως εξής: • Αντικαθιστούμε με διάφορες κατάλληλες τιμές • Αντικαθιστούμε με διάφορες κατάλληλες μεταβλη­

τές • Διαμορφώνουμε σχέση για το ζητούμενο • Χρησιμοποιούμε την ιδιότητα: αν ιQβ και α::;β, τό­

τε α=β ΑΣΚΗΣ Η ι 11 Να βρεθεί ο τύπος της συνάρτησης f, αν f(x)-x ::; χ2 :5 f(x-1)+x ,για κάθε χεR (1) Λύση Από την ( 1 ) έχουμε f(x):Sx2+x, για κάθε χεR

(2). Επίσης από την ( 1 ) έχουμε f(x-1 ) Ξ:: χ2 -χ, για κάθε χεR (3). Θέτουμε u=x-1 , δηλαδή x=u+ 1 , οπότε : (3) =>f(u)Ξ::(u+ 1 )2-(u+ 1 )=>f(u)Ξ::u2+u:::::> f (χ)Ξ::χ2+χ, για κάθε χεR (4) . Από (2), (4) έχουμε f(x)= χ2+χ, για κάθε χεR.

ΑΣΚΗΣ Η 2η Να βρεθεί ο τύπος της συνάρτησης f, αν f

περιττή και (x2+1)f(x):S3x , για κάθε χεR. Λύση Έχουμε χ2+ 1 >0. Άρα από (χ2+ 1 )f(x) ::; 3χ προ-

κύπτει f(x):S� ( 1 ), οπότε f(-x):S 3( -2χ) για χ + 1 (-χ) + 1

κάθε χεR (2) και αφού η fπεριττή, f(-x)=-f(x) . Άρα από τη (2) προκύπτει -f(x):S �3χ , δη­χ + 1

λαδή f(χ)Ξ::� για κάθε χεR (3). Από ( 1 ),(3) χ + 1 έχουμε f(x)=� για κάθε χεR. χ + 1 2. Αξιοποίη ση με ό ρια

• Αν f(x)<g(x) κοντά στο χ0, τότε lim f(x):S lim g(x), εφόσον υπάρχουν τα όρια.

Χ ----+Χο

Χ --+ Χ ο

τότε και lim g(x)=+oo (ή αν lim g(x)=-oo, τό-χ --+χ0 Χ --+ Χ ο

τε και lim f(x) =-οο Χ -+ Χ ο

ΑΣΚΗΣΗ 3η Μια συνάρτηση f είναι συνεχής στο χ0=0 και

έχει την ιδιότητα ιf(χ) -η�3Χ Ι � χ2ημ2 � για

κάθε χεR* (1) Να υπολογίσετε την τιμή της f στο χ0=0.

Λύση : Η ιδιότητα γράφεται 1 ημ3χ 1 ημ3χ -χzημz - + -- � f(χ) � χ2ημ2 - + -- και χ χ χ χ

l.m( 2 2 1 ημ3χ)- 1;...,( 2 2 ι ημ3χ)-ο 3-3 <*) l -Χ ημ -+-- - un.,_X ημ -+-- - + -χ--.ο χ χ χ--.ο χ χ οπότε από το κριτήριο παρεμβολής θα έχουμε lim f(x) = 3 και αφού η f είναι συνεχής στο χ0=0 χ �Ο

θα είναι f(O) = lim f(x) = 3 χ �ο

ΑΣΚΗΣΗ 4η Αν συν2χ+συν2αχΞ::2(1+αχ2) για κάθε χεR*

(1 ), να βρεθεί η τιμή του α Λύση

Αφού α:;t:Ο, ( 1 ) :::::> συν2χ+συν2αχΞ::2( 1 +αχ2)=> :::::> ι-ημ2χ+ ι-ημ2αχ 2: 2+ 2αχ2:::::> 2 2 2 2 2 ημ χ 2 ημ αχ :::::>ημ χ+ημ αχ:S-2αχ :::::>-2-+α -2-2-�-2α(2) . χ α χ 2 2 Αλλά lin(ημ

2x +α2��) = ι + α2 και lim(-2α) = -2α

χ--.ο χ α χ χ--.ο

Άρα: (2)=> 1 2+α2:5-2α=>( α+ ι )2:50:::::> :::::>α+ l=Ο:::::>α=-1 (για α=Ο δεν ισχύει)

ΑΣΚΗΣΗ 5η Α ν f συνεχής στο R και ισχύει

xf(x) � f(x) + χ2 + χ - 2 (1) για κάθε χεR-{1}, να βρεθεί το f(1)

Λύση

• Αν g(x):Sf(x):Sh(x) κοντά στο Χο και lim g(x) από κριτήριο παρεμβολής θα είναι 1ίmχ2ημ2 _!_ = Ο . Επίσης: Χ --+ Χ σ Χ 40 Χ = lim h(x)=L, τότε και lim f(x) =L χ -+χσ χ --+χσ Iim(ημ3x) = lim ημ3χ · 3 = lim ημu · 3 = 1 · 3 = 3

χ->0 Χ

χ--.ο

3χ u--.o U

ΕΥΚΛΕΙΔΗΣ Β ' 83 τ.3/61

Page 64: Ευκλειδης Β 83

------------- Μαθηματικά για την Γ Λυκείου -------------

Αφού η f είναι συνεχής στο R θα είναι και στο χ0= 1 , δηλαδή limf(x) = limf(x) = f(l) , οπότε αντί

χ--+Γ χ--+1+

να βρούμε το f( l ) αρκεί να βρούμε το limf(x) . Η χ -> 1

σχέση ( 1 ) γίνεται (x - l)f(x) :<ς x2 + χ - 2 (2) "' Αν χ> 1 , τότε χ-1 >0 και

(2) � f(x) :<ς xz + x - 2 � f(x) :<ς (x - l)(x + 2) χ - 1 χ - Ι

� f(x) :<ς χ + 2 � lim f(x) :.ς lim (x + 2) � χ --+ 1 + χ --+ 1 +

lim f(x) :<ς 3� limf(x) :<ς 3 . (i) χ -+1 + χ -+ 1

αι Αν χ< 1 , τότε χ-1<0 και

(2) � f(x) � xz + χ - 2 � f(x) � (χ - l)(x + 2) χ - 1 χ - 1

� f(x) � x + 2 � lim f(x) � lim(x + 2) � χ-+Γ χ -+ 1 -

lim f(x) � 3� lim f(x) � 3 . (ii) Χ ---JοΓ χ -+ Γ

Από (i), (ii) προκύπτει lim f(x) = 3 . Χ -> 1

3. Αξιοπο ίηση μ�; παρ ογισγους n f'(x) ---+μονοτονία -ακρότατα r. f"(χ)---+κοίλα-κυρτά ---+ανισότητα Jensen Προσοχή! ! ! Αν f(x)<g(x) για κάθε x ε AfnAg δεν

ισχύει κατ ' ανάγκην f' (χ )<g ' (χ) για κάθε χ ε Af 'n Ag ' .

,\Σ K if-[U-Ji 6' � Αν f'(x) + f(x) > O για κάθε x E JR (ι) και

f(ι) = ι να δειχτεί ότι f(O) < e . \ {)ση Από ( 1 )� eXf'(x) + exf(x) > ο � (exf(x)) ' > ο

� g'(x) > ο ' όπου g(x) = exf(x) άρα gtJR οπότε: Λ

g(O) < g(l) � e0f(O) < e1 f(l) � f(O) < e 4. ,\ξιωτο iηση με Θι;{iφημα FΕRΜΑΤ

Όταν δοθεί ανισοϊσότητα Α(Χ)2::Β(Χ) τότε πιθανό­τατα πρέπει να χρησιμοποιήσουμε Θ.Fermat, οπότε ερ­γαζόμαστε ως εξής: Α(Χ}-Β(Χ)2::0, θεωρούμε συνάρτη­ση h(x)= Α(Χ}-Β(Χ) για την οποία προφανώς ισχύει h(x)2::0 ( Ι ) . Σε αυτό το σημείο είναι σημαντικό να βρούμε κάποια τιμή χ0 για την οποία h(x0)=0, οπότε η ( 1 ) γίνεται: h(x)2::h(x0) (2) (mθανόν το χ0 να μην είναι μοναδικό). Η σχέση (2) σημαίνει πώς η h παρουσιάζει ελάχιστο στο χ0. Ελέγχουμε τώρα αν το χ0 είναι εσωτε­ρικό σημείο και στο οποίο η h παραγωγίζεται, οπότε με

βάση το θεώρημα Θ.Fermat θα είναι h'(x0 ) = Ο και

έτσι από αυτή τη σχέση θα προκύπτει το ζητούμενο. Α Σ Κ Η Σ Η Ί' � Αν f παραγωγίσιμη στο χ0=ι με f(ι) = ι και

f(x) � χ2 για κάθε χ >Ο δείξτε ότι f'(ι) = 2

Λi)ση Αφού f(x) � χ2 � f(x) - χ2 � Ο . Θεωρούμε τη

συνάρτηση g(x) = f(x) - χ 2 • Είναι g(x) � Ο ( 1 ) . Αλλά g(l) = f(I) - 12 = 1 - 1 = Ο επομένως η ( 1 )

γίνεται g ( χ) � g(l) αυτό σημαίνει ότι η g παρουσιά­

ζει ελάχιστο στο χ0= I και στο εσωτερικό αυτό ση­μείο του (0, +οο) η g είναι παραγωγίσιμη, αφού g'(x) = f'(x) - 2x για κάθε χΕ(Ο, +οο), οπότε από Θ. Fermat έχουμε g'(l) = Ο � f'(l) - 2 = Ο � f'(l) = 2

Α 2---: t� !-1 2.: Η 8" Α ν f (χ) = 2 χ + 3 χ + m χ - 2 - συν χ όπου m>O

να βρεθεί η τιμή του m ώστε f(x) � Ο για κάθε Χ Ε JR (I)

Παρατηρούμε ότι: f( Ο) = 2° + 3° + m0 - 2 -συνΟ = I + I + 1 - 2 - 1 = 0 ,

οπότε: (I)=> f(x) � Ο => f(x) � f(O) =>Η συνάρτηση

f παρουσιάζει ελάχιστο στο χ0=0 το οποίο είναι εσωτερικό σημείο του R. Στο R η f παραγωγίζεται

με f'(x) = 2x Ιη 2 + 3χ lη 3 + mx Ιη m + ημχ . Άρα

από Θ. Fermat παίρνουμε :

f'(O) = Ο => 2° Ιη 2 + 3° Ιη 3 + m0 Ιη m + ημΟ = Ο => I ln2+ ln 3+ lnm= O=> In(2 · 3 · m) = 0=>6m=l=> m = -6

ΑΣ Κ Η Π-! 9' � Έστω f συνεχής συνάρτηση f: R ---+ R τέτοια ώστε f(ι )=2. Α ν για κάθε χ Ε JR , ισχύει

h(x)= J.'� 1 +z2 1 f(t)dt-lz 1 - z 2 Ι (χ2 - 1) � Ο όπου

Ζ1 , z 2 Ε C* , τότε: ω Να αποδείξετε ότι η συνάρτηση h είναι πα­

ραγωγίσιμη στο JR και να βρείτε τη h '.

j$ ) Να αποδείξετε ότι Jz 1 + z 2 J =I Ζ 1 -Ζ 2 I γ) Με δεδομένη τη σχέση του ερωτήματος β να

αποδείξετε ότι � Ε Ι z .

Λ-ί)ση α . Προφανώς: Ιz ι+zz l Ε JR και για κάθε χ Ε JR η f είναι συνεχής στο κλειστό διάστημα με άκρα Ι , χ. Άρα η συνάρτηση J' ιz , +z2 1 f(t)dt είναι παραγωγί­

σιμη για κάθε χ Ε JR . Επίσης η συνάρτηση Ιz ι­z2 Ι (χ2- 1 ) είναι παραγωγίσιμη, οπότε η h είναι πα­ραγωγίσιμη στο JR με

h'(x)= Ιz , +z2 1 f(x) - 2lz 1 - z2 I χ β. Είναι h(x)� και αφού h( I )=Ο θα είναι h(x)2:h( I ).

Επίσης η h είναι παραγωγίσιμη στο εσωτερικό σημείο χ0= 1 του JR οπότε από το θεώρημα Fer­mat παίρνουμε: h'( l )=Ο =>

Ιz , +z2 1 f( l ) - 2 lz1 - z2 1= 0=> Jz1 + z2 J =I z 1 -z2 1

γ . Jz , + zz J =I z, -z2 I => Jz , + z2 Jz =I z, -Zz 12 =>

ΕΥΚΛΕΙΔΗΣ Β ' 83 τ.3/62

Page 65: Ευκλειδης Β 83

------------- Μαθηματικά για την Γ Λυκείου -------------(zl + z2 )(zl + z2 ) = (z l -z2 )(zl -z2 ) =>

zl zl +zl z2 +z2 zl +z2 z2 =zl zl -zl z2 -z2 zl +z2 z2 =>

2zl z2 + 2z2 zl = ο => z2 zl = -z2 zl => z2 zl Ε Ι ο Αλλά - -� = z2 · � = � · � ε Ι αφού z z ε

Ι και l z 1 2 ε JR. - I 1 2 ' 2 I I

ΖΙ z� · � ΖΙ ΑΣ ΚΗΣΗ 1 0'� Αν 7' � αχ + 1 για κάθε χ ε JR. να βρεθεί η

τιμή του πραγματικού αριθμού α. Λύση

Θεωρούμε τη συνάρτηση f(x) = γ - αχ - 1 . Είναι f(x) � Ο για κάθε χ ε JR. . Αλλά Ο= f(O) , ο­

πότε f(x) ::; f(O) για κάθε χ ε JR. , δηλαδή η συ­

νάρτηση παρουσιάζει μέγιστο στο εσωτερικό ση­μείο, χ0 =Ο του JR. . Αλλά η f είναι παραγωγίσιμη

στο R με f'(x) = γ ln 7 - α , με Θ. Fermat παίρ-

νουμε f'(O) = Ο => 7° ln 7 - α = Ο => α =ln7.

ΑΣΚ Η Σ Η l l '� Έστω f συνάρτηση παραγωγίσιμη στο JR.

τέτοια ώστε: ( f(t)dt � χ2 - χ για κάθε χ ε R Να αποδείξετε ότι υπάρχει ένα τουλάχιστον

χ0 ε ( 0, 1) τέτοιο ώστε f '(x0 ) = Ο

ΛίJση Θεωρούμε τη συνάρτηση

h(x) = ( f(t)dt - χ2 + χ που είναι παραγωγίσιμη

στο R και είναι h '(x) = C ( f(t)dt - χ2 + χ) ' = f(x2 )2x - f(x) - 2χ + 1 , για κάθε χ ε JR. .

Είναι h(x) � Ο και αφού Ο = h(O) = h(1) , θα

είναι h(x) � h(O) και h(x) � h(l) . Οπότε ισχύουν

οι προϋποθέσεις του θεωρήματος Feπnat για χ0=0 και για χ0= 1 . Άρα h '(Ο) = Ο και h '(1) = Ο από τις

οποίες προκύπτουν f(O) = 1 και f(l) = 1 , δηλαδή

f(O) = f(l) . Επίσης η f είναι συνεχής στο [0, 1 ] ,

παραγωγίσιμη στο (0, 1 ) . Συνεπώς ισχύει το Θ. ROLLE, δηλαδή υπάρχει τουλάχιστον ένα

χ0 ε (Ο, 1 ) τέτοιο ώστε f'(x0 ) = 0 .

5. Αξιοπο ίηση με ΟΛΟΚΛ Ή ΡΩΜΑ ΤΑ

• Αν f,g συνεχείς στο [α,β] και f(x)�g(x), γ ια

κ ά θ ε Χ Ε [α , β ] τότε r f(x)dx ::; r g(x)dx ΑΣΚΗΣΗ 1 2'� Έστω f συνάρτηση, συνεχής στο [1 ,3 ) τέ-

τοια ώστε: !2 f(x)dx > Ο και J: f(x)dx < Ο

Να αποδείξετε ότι υπάρχει ένα τουλάχιστον χ0 ε (1 ,3) τέτοιο ώστε f(x0 ) = Ο

Λύση Αν ήταν f(x) ::; O για κάθε χ στο [ 1 ,2] τότε

j2 f(x)dx ::; Ο άτοπο, άρα η συνάρτηση παίρνει και

θετικές τιμές στο [ 1 ,2] δηλαδή υπάρχει ξ1 ε [ 1 ,2] ώστε f(ξ ) > 0 . Ομοίως αν ήταν f(x) � Ο για κάθε

χ στο [2,3] τότε r f(x)dx � ο άτοπο, άρα η συ­

νάρτηση παίρνει και αρνητικές τιμές στο [2,3] δηλ. υπάρχει ξ2ε[2 ,3] ώστε f(ξ2 ) < Ο . Για την f , ισχύ­

ει το Θ. Bolzano στο [ξ 1 ,ξ2] ς;:: [ 1 , 3 ] , άρα υπάρχει

ένα τουλάχιστον χ0 ε ( 1 ,3) τέτοιο ώστε f(x0) = 0 ΑΣ Κ Η Σ Η 1 3 ' � Έστω f συνάρτηση, κοίλη στο [0,2] με

2 f'(l) = f(1) = 1 . Να δειχτεί ότι Jr(x)dx � 2

ο

Λύση Η εφαπτόμενη στη γραφική παράσταση της f

στο σημείο της με τετμημένη 1 είναι: y - f(l ) = f '(l)(x - 1) οπότε είναι y = χ και αφού

f κοίλη θα είναι για κάθε χ ε [Ο, 2] , f(x) ::; x 2 2 2

.Άρα Jr(x)dx ::; Jxdx και από αυτό Jf(x)dx ::; 2 ο ο ο

Β. Η ανισότητα ως ζητούμενο Με τα παρακάτω παραδείγματα θα δούμε πολλές

διαφορετικές περιπτώσεις απόδειξης μιας ζητούμενης ανισότητας ή ανισοϊσότητας. i . Απόδειξη με απλές ιδιότητες διάταξης ι ι . Απόδειξη με μονοτονία-ακρότατα-σύνολο τιμών

ΑΣΚΗ Σ Η 1 4 '� Ν α δειχτεί ότι για χ�Ο ισχύει e' + συνχ � χ + 2 ΛίJση

Θεωρώ τη συνάρτηση f(x) = ex +συνχ-χ- 2 Η παράγωγος της είναι f'(x) = ex - ημχ - 1 και

η δεύτερη παράγωγος της Γ (χ) = ex - συνχ , οπό­

τε για χ>Ο είναι e' > 1 και αφού -συνχ � -1 , θα

ισχύει e' - συνχ > Ο , δηλαδή Γ'( χ) > Ο . Επομένως

η f' είναι γνησίως αύξουσα στο [0, +οο ) . Άρα: χ>Ο => f '(x) > f '(O) =0=> f '(x) > Ο => f γνησίως αύ-

ξουσα στο [0, +οο). Άρα: χ�Ο => f(x) � f(O) = Ο => ex + συνχ - χ - 2 � ο => ex + συνχ � χ + 2

• Προφανώς για χ>Ο θα ισχύει f(x) > f(O) = O ,

δηλαδή ex + συνχ > χ + 2 ί ί ί . Απόδειξη με Θεώρη μα Μέσης Τ ιμής

ΑΣΚΗΣΗ 1 5 '� Δ ίνεται η συνάρτη ση f, παραγωγίσι­

μη στο R που ικανοποιε ί τις σχέσε ις :

f(x) - e-r(x) = χ - 1 , χ ε R

i ) Ν α εκφ ρ α στε ί η f ' ως συνάρτηση της

ΕΥΚΛΕΙΔΗΣ Β ' 8 3 τ.3/63

Page 66: Ευκλειδης Β 83

------------- Μαθηματικά για την Γ Λυκείου ------------f και να δ ε ιχτε ί ότι η γ ραφ ική παρά­σταση της f β ρ ίσκεται πάνω από την ευθ εία y=x-1 .

i i ) Να β ρ εθε ί το f(O ) , η μονοτονία της f κα ι τη ς f ' .

i i i ) Να δ είξετε ότ ι χ -< f(x) < χ · f ' (x) για 2

κάθε χ>Ο . Λύση

i ) Παραγωγίζου με και τα δύο μέλη τη ς δοσμένη ς σχέσης , οπότε παίρνουμε :

f'(x) + e -ΨJ. f'(x) = 1 � f'(x)(1 + e -f(x )) = 1 �

f'( ) - 1 ' 1 -f(x ) ο � χ -1 -r(x )

, αφου + e > . + e

Από τη δοσμένη σχέση έχουμε

f(x)- x+1= e-r(x) > O , οπότε: f'(x)= 1 f(x)- x + 2

και f(x)> χ - 1 δηλαδή η γραφική παράσταση της f βρίσκεται πάνω από την ευθεία y=x-1 i i) Για χ=Ο έχουμε f(O) -e-f{0) = - 1 , η οποία αληθεύει μόνο αν f(O) = Ο (Η συνάρτηση h(x)= x-e-x+ 1 έχει ρίζα την χ0=0. Αλλά h' (x)= 1+e-x>O, οπότε h t IR. . Άρα η χ0=0 είναι μοναδική ρίζα της)

Λ

Είναι f'(x) = 1

1-f(x ) + e

f(x ) e Ο

,

1 + ef(x ) > , οποτε

ftiR. . Το δεύτερο μέλος παραγωγίζεται επομένως Λ

υπάρχει δεύτερη παράγωγος της f και είναι f(x ) . f '( )(1 + f(x ) ) - f(x ) f( x ) . f '( ) f "(χ) = e χ e e e χ =

(1 + ef(x ) )2

- er(x ) . f '(x) ' - r( ) > Ο . Αρα f t IR. . (1 + e χ )2 Λ

ί ί ί) Αφού χ>Ο αρκεί να δείξουμε ότι 1 f(x) f '( ) , 1 f(x) -f(O) f'( ) δ λ δ , - < -- < χ ' η - < < χ , η α η 2 χ 2 χ -0 f '(O) < f '(ξ) < f '(x) ( Ι) για κάποιο ξ Ε {Ο, χ ) , αφού ισχύουν οι προϋποθέσεις Θ.Μ.Τ. για την f στο

[Ο,χ] οπότε υπάρχει ξΕ (Ο,χ) ώστε f'(ξ) f(x) -f(O) χ-0

Η (I) ισχύει αφού f t [Ο, χ] και Ο<ξ<χ Λ ΑΣΚΗΣΗ 1 61] Δίνεται συνάρτηση f με f'(x)>1 για κάθε

χ Ε 1R . Να δειχτεί ότι η εξίσωση f(x)=O έχει μια τουλάχιστον πραγματική ρίζα.

Λύση Για χ>Ο , ισχύουν προϋποθέσεις Θ.Μ.Τ. για

την f στο διάστημα [Ο,χ] οπότε υπάρχει ξΕ (Ο,χ)

τέτοιο ώστε: f'(ξ) = f(x) - f(O) . Αλλά f'(ξ) > 1 χ - 0

και χ>Ο, οπότε f(x) > χ + f(O) και lim (χ + f(O)) = +οο . Άρα lim f(x) = +οο

χ --++οο χ --++co

Επίσης για χ<Ο, ισχύουν προϋποθέσεις Θ.Μ.Τ. για την f στο διάστημα [ χ,Ο] οπότε υπάρχει

ξΕ (χ,Ο) τέτοιο ώστε: f'(ξ) = f(O) - f(x) . Αλλά 0 - χ

f'(ξ) > 1 και χ<Ο, οπότε f(x) < χ + f(O) και lim (χ + f(O)) = -οο . Άρα lim f(x) = -οο . Η f εί-

χ ----)--α) χ --+-ο:ι

ναι παραγωγίσιμη στο IR. οπότε και συνεχής σε αυτό. Από τα παραπάνω προκύπτει ότι f(A) = IR. και αφού ΟΕ f(A) επομένως υπάρχει χ0 Ε IR. τέ­τοιο ώστε f(x0 ) =Ο (β 'τρόπος: με χρήση Bolzano) iv . Κυρτότητα - σχετι κή θέση εφαπτόμενης

ΑΣΚΗΣΗ 1 7η Α ν f κυρτή και δύο φορές παραγωγίσιμη

στο 1R με f(2)=0 και f'(2)=1 και g(x) = er<•> + χ2 (1) για κάθε χ Ε 1R , να δειχτεί ότι g(x):?:Sx-5 για κάθε χ Ε 1R .

Λύση

Παρατηρούμε ότι:( l )� g'(x) = er<x>f(x)+2x � g"(x) = ef( x ) [f'(x)]2 + ef< x >f '(x) + 2 . Αφού η f είναι κυρτή θα είναι f '(χ) ;:::: Ο ,οπότε

g " (χ) > Ο , δηλαδή η g είναι κυρτή. Η εφαπτόμενη ευθεία στη γραφική παράσταση της g στο σημείο της A(2,g(2)) είναι y-5=5(x-2) και επειδή g κυρτή, η γραφική της παράσταση θα βρίσκεται «πάνω» από την εφαπτόμενη δηλαδή g(x):?:Sx-5 για κάθε xER.

ΑΣΚΗΣΗ 1 81] Δίνεται συνάρτηση f με f(x) = ln(ex + 1)

α) Εξετάστε την κυρτότητα της f β) Να βρεθεί η εξίσωση της εφαπτόμενης στη

γραφική παράσταση της f στοΑ(Ο,f(Ο)) και

να δειχτεί ότι ln( e' + 1) - � � ln 2 2

Λί>ση

e' � α) Έχουμε f '(x) = -- , οπότε f"(x)=--2 >0 e' + 1 (e' +1) Άρα η f είναι κυρτή

β) Είναι f(O)=ln2 και Γ(Ο)=� , άρα

ε : y-ln2=_!_(x-O) , δηλαδή ε: y =_!_x + ln 2 . Αφού 2 2

η f είναι κυρτή, η γραφική της παράσταση θα βρί­σκεται πάνω από την εφαπτόμενη δηλ.

f(x);:c:lx+ln2, άρα ln(e' +1) -� 2=: ln2 για κάθε xER. 2 2

ΕΥΚΛΕΙΔΗΣ Β' 83 τ.3/64

Page 67: Ευκλειδης Β 83

Ερευνητικές Εργασίες . . . για την Α ' Λυκείου Α. Μητρογιαννοπούλου

Σχολική Σύμβουλος Μαθηματικών - Επιμορφώτρια για τις ερευνητικές εργασίες

Από το σχολικό έτος 201 1-12 στο Αναλυτικό Πρόγραμμα του Λυκείου είναι οι ερευνητικές ερ­γασίες, 3 ώρες για το Γενικό Λύκειο και 2 για το Επαγγελματικό. Σε άλλες χώρες και ιδιαίτερα στη Γαλλία ' έχουν εισαχθεί εδώ και αρκετά χρόνια. Συμμετέχουν όλοι οι μαθητές με ομαδοσυνεργα­τική διαδικασία, η οποία βαθμολογείται, ενώ υ­πάρχει ομαδικός και ατομικός βαθμός. Ειδικά για το α' τετράμηνο της πιλοτικής εφαρμογής ο βαθ­μός είναι μόνο ομαδικός.

Ας δούμε όμως στην αρχή τι είναι και πως θα 'τρέξει' η ερευνητική εργασία στη σχολική διδα­κτική πραγματικότητα και κατόπιν θα δοθούν συ­γκεκριμένα παραδείγματα καλής πρακτικής.

Η βασική φιλοσοφία των εν λόγω εργασιών εί­ναι η διερευνητική προσέγγιση της μάθησης, δηλαδή να γίνει έρευνα σε φυσικό και κοινωνικό πεδίο με εργαλεία τόσο ποσοτικά όσο και ποιοτι­κά. (συζήτηση , ερωτηματολόγια, παιχνίδια, δρώ­μενα . . . ) . Οι σκοποί 2 των ερευνητικών εργασιών είναι οι παρακάτω χωριστά ή και συνδυαστικά: • Κατανόηση φυσικού και κοινωνικού κόσμου • Επίλυση προβλημάτων και λήψη αποφάσεων σε προβλη­

ματικές καταστάσεις • Κατασκευή ποικίλης φύσης αντικειμένων και συστημά­

των • Καλλιτεχνική έκφραση

Η ταξινόμηση των θεμάτων3 από επιστημονική άποψη έχει ως εξής: 1 . Ανθρωπιστικές και κοινωνικές επιστήμες 2. Φυσικές Επιστήμες, Μαθηματικά, Τεχνολογία 3 . τέχνες και πολιτισμός 4. Περιβάλλον και Αειφορία

Βέβαια δεν αποκλείεται κάποιο θέμα να μην εντάσσεται αποκλειστικά σε έναν μόνο από τους παραπάνω τομείς, αλλά είναι δυνατό να υπάρξει και κάποιος συνδυασμός τους.

Οι μαθητές οργανώνονται σε νέα τμήματα τα λε­γόμενα 'Τμήματα Ενδιαφέροντος' ανάλογα με την ερευνητική εργασία και τον τομέα που θα εντάσσε­ται κάθε μία από αυτές, ενώ σε κάθε ένα από τα ανωτέρω τμήματα μπορούν να συμμετέχουν μέχρι δύο εκπαιδευτικοί (στα ΕΠΑΛ συγχρόνως δύο).

Η επιλογή θέματος και δραστηριοτήτων έχουν

1 Direction de ι ' enseignement scoιaire - Bureau du contenu des enseignements - Themes ΤΡΕ- Classe de premiere, p. 2 & Direction generaιe de ι ' enseignement scoιaire- Themes ΤΡΕ- Serie L - Classe de premiere, p . 1 , στο www .edusco l. education. fr/00050/themes.htm

2 Η. Ματσαγγούρα, «Θεωρητικές Αρχές και Διδακτικές επι­

λογές», στο Ή καινοτομία των ερευνητικών εργασιών στο Λύκειο ' , σ. 1 7

3 Η . Ματσαγγούρα, ο.π. , σ. 1 8

αναφορές στα γνωστικά αντικείμενα της Α' Τάξης και κυρίως σε όσα σχετίζονται αμεσότερα με την καθημερινή ζωή, ώστε να υπάρχει όσο το δυνατόν περισσότερη σχέση με τις προσωπικές εμπειρίες των μαθητών, έτσι ώστε να μπορούν οι μαθητές να αυτενεργούν, προκειμένου να κατακτήσουν τη νέα γνώση και στη συνέχεια να γίνει αξιοποίησή της.

Οι εργασίες αξιολογούνται και τελικά βαθμολο­γούνται, ενώ η έμφαση δίνεται στα παρακάτω4: α) Διερευνητική διαδικασία και βαθμολογία έρευνας

β) Περιεχόμενο ερευνητικού αποτελέσματος γ) Γλώσσα και δόμηση της ερευνητικής έκθεσης της ομά­

δας δ) Τρόπο παρουσίασης της ερευνητικής εργασίας στην εκ­

παιδευτική κοινότητα

Η τελική βαθμολογία προκύπτει με βάση τα α­ποτελέσματα της έρευνας, τη συμμετοχή κάθε μα­θητή στην όλη διαδικασία και την παρουσίαση της έκθεσης. Υπάρχουν ατομικές εκθέσεις (ατομικός φάκελος κάθε μαθητή) και ομαδική από τα μέλη της ομάδας σχετικά με το τι έχουν κάνει.

Μέθοδος υλοποίησης ερευνητικών εργασιών Το θέμα κάθε τμήματος ενδιαφέροντος αποτε­

λείται από δύο υποθέματα, ένα από το φυσικό και ένα από το κοινωνικό περιβάλλον και πρέπει να ολοκληρώνεται στο τετράμηνο. Δημιουργούνται υποομάδες που αναλαμβάνουν επιμέρους τμήματα του θέματος. Σε ορισμένες περιπτώσεις θα μπο­ρούσε να υπάρχει επέκτασή του και στο επόμενο τετράμηνο. Οι φάσεις της ομαδοσυνεργατικής έ­ρευνας, όπως αναφέρονται στον οδηγό5 είναι οι εξής: Α. Γενικός Προγραμματισμός σε Ολομέλεια Τμήματος

Β. Προγραμματισμός και Προετοιμασία της Ερευνητικής Ομάδας

Γ. Υλοποίηση Δράσεων από τις Υποομάδες για Συλλογή Δεδομένων

Δ. Επεξεργασία Δεδομένων από Ομάδα εντός Τάξης Ε. Επιλογή Τρόπων Αναπαράστασης Νέων Γνώσεων ΣΤ. Προκαταρτική Παρουσίαση Εργασιών σε Ολομέλεια Ζ. Διαμόρφωση Φακέλου της Ερευνητικής Εργασίας Η. Συνοπτική Παρουσίαση Εργασίας στα Ελληνικά ή I και

Αγγλικά Θ. Αξιολόγηση Ομαδικής Εργασίας και Ατομικής Συμβολής

Μελών

Συγκεκριμένα η όλη διαδικασία βασίζεται στα παρακάτω: Επιλογή θέματος I Καθορισμός διαστάσεων θέ­ματος, απ' όπου θα προκύψουν τα υποθέματα των ομάδων και τα ερευνητικά ερωτήματα I Συλλογή

4 Τα ποσοστά της τελικής βαθμολογίας είναι για τους 2 πρώ­τους τομείς από 30% και για τους άλλους δύο από 20%.

5 «Η καινοτομία των ερευνητικών εργασιών», σσ. 54-59

ΕΥΚΛΕΙΔΗΣ Β' 83 τ. 3/65

Page 68: Ευκλειδης Β 83

------------ Ερευνητικές Εργασίες . . . για την Α ' Λυκείου ------------

δεδομένων - αξιοποίηση πηγών και βιβλιογραφίας Επεξεργασία δεδομένων I Εξαγωγή συμπερα­σμάτων - γενικεύσεις

Η πορεία εργασιών, με βάση την εμπειρία από άλλες ενέργειες που υλοποιούνταν σε διάφορα προγράμματα (περιβαλλοντικά, υγείας . . . ) , θα μπο­ρούσε ενδεικτικά να είναι η ακόλουθη με κάθε βή­μα να αντιστοιχεί και σε μία συνάντηση : I . Συζήτηση με μαθητές για την εν λόγω δράση, ανίχνευση

θεμάτων που θα τους ενδιέφεραν, αλλά και προτάσεις από εκπαιδευτικούς

2 . Δημιουργία ομάδων εργασίας μαθητών για ενασχόλησή τους με τις ενότητες - υποθέματα που προκύπτουν λόγω των διαφόρων διαστάσεων του θέματος χωρίς να απο­κλείεται και η ατομική συμμετοχή.

3. Συλλογή πρωταρχικών πληροφοριών 4. Επιλογή υλικού και προσαρμογή του στις ανάγκες του

θέματος 5 . Προσφορά επιπλέον υλικού και ανάλογη επεξεργασία

του 6 . Δημιουργία γραπτού κειμένου6, έργου (αφίσας, ιστοσελί­

δας, τεχνικής κατασκευής, μακέτας . . . . ) ή καλλιτεχνικού δρώμενου (θεατρικού, μουσικού . . . )

7 . Παρουσίαση της δουλειάς κάποιων ομάδων στην ολομέ-λεια - παρατηρήσεις I επισημάνσεις - προτάσεις

8 . Παρουσίαση του έργου και των υπολοίπων ομάδων

9. Σύνθεση των επί μέρους έργων σε ένα τελικό προϊόν 1 0. Παρουσίαση του τελικού πονήματος στην ολομέλεια του

τμήματος - τελικές επεμβάσεις διορθώσεις. 1 1 . Παρουσίαση στο σχολείο, στην τοπική κοινωνία ή σε

διάφορες πολιτιστικές εκδηλώσεις

Στα όσα αναφέρθηκαν θα πρέπει να ληφθούν υπόψη και ορισμένες άλλες ενέργειες ανάλογα με τη φύση του θέματος, όπως είναι η σύνταξη κά­ποιου ερωτηματολογίου7 για τη διεξαγωγή ορισμέ­νων συνεντεύξεων, η επίσκεψη στο πεδίο έρευνας με σχετική φωτογράφηση, ή μία συνεργασία με κάποιο άλλο σχολείο στο εξωτερικό και μάλιστα στην αγγλική γλώσσα ή και σε κάποια άλλη, όπως συμβαίνει στην περίπτωση της δράσης 'ηλεκτρο­νική αδελφοποίηση σχολείων' (etwίnning)

Στη συνέχεια αναφέρονται παραδείγματα για τα γνωστικά αντικείμενα στο νέο Αναλυτικό Πρό­γραμμα της Α' Λυκείου που μπορούν να δώσουν ιδέες για την πραγματοποίηση διεπιστημονικών ερευνητικών εργασιών:

Στην Άλγεβρα υπάρχει το κεφάλαιο των Πιθα­νοτήτων από τη Γ Λυκείου και το κεφάλαιο των προόδων από τη Β ' Τάξη . Στη Φυσική έχουν συ­μπεριληφθεί ορισμένα θέματα από τον Ηλεκτρι­σμό (Β ' Τάξη) . Αλλαγές έχουν γίνει και στην ύλη της Χημείας, ενώ ένα νέο αντικείμενο για την Α ' Λυκείου, αυτό της Βιολογίας, συμπεριλαμβάνεται

6 Πιο εξυπηρετικό είναι, πριν από το τελικό προϊόν να έχουν συνταχθεί επί μέρους κείμενα

7 Καλό είναι αρχικά το ερωτηματολόγιο να εφαρμοστεί σε μικρό πλήθος όπως για παράδειγμα στους μαθητές του ίδι­ου του τμήματος ενδιαφέροντος.

στο αναλυτικό πρόγραμμα. Με βάση τα ανωτέρω ορισμένα θέματα για ερευνητικές εργασίες θα μπορούσαν να είναι τα παρακάτω:

το οποίο προσφέρεται για συνεργασία με τον καθηγητή της Φυσικής και μπορεί να διερευνηθεί η εν λόγω έννοια από μαθηματική άποψη, σύμφωνα με το πρόγραμμα της Άλγεβρας, αλλά και από τη Φυσι­κή τόσο για τα θέματα βολής όσο και για αντικείμενα, κατασκευές και ενέργειες στο φυσικό και κοινωνικό περιβάλλον που σχετίζονται με τη μορφή της παρα­βολής (τούνελ, ποδόσφαιρο . . . )

,, " -1 , όπου θα μπορούσε να γίνει μελέτη α-

πλών κανονικών πολυγώνων, βασικά τετραγώνων και ορθογωνίων, που κυρίως χρησιμοποιούνται σε πλα­κοστρώσεις απλών δαπέδων, καθώς και άλλων ειδών που κυρίως αξιοποιούνται στην Ανατολή, αλλά και στην τέχνη.

\. ' f.J. Eι: 2ΆΤΟ U' με διεπιστημονική συνεργασία φυσικού­

χημικού ή φυσικού και μαθηματικού, εφόσον στα α­ντικείμενα Φυσικής και Χημείας υπάρχουν οι μονάδες μάζας και όγκου. Αν τώρα το θέμα επικεντρωνόταν στις μονάδες μήκους θα μπορούσε να γίνει μελέτη ει­δικά για όλες αυτές τις μονάδες που δε χρησιμοποιού­νται πλέον, αλλά έχουν μείνει στις καθημερινές μας εκφράσεις, π.χ. 'απέχει δυο βήματα' , μη κουνηθείς ρούπι · , ' . . . ένα δακτυλάκι κρασί' κ.ά.

Κ . \ Λ Σ \ Ι Λ.-. Στο εγχειρίδιο της Γραμματικής

της Αρχαίας Ελληνικής γίνεται λόγος για αριθμητικά επιρρήματα και κλασματικούς αριθμούς (σ. 1 32), κα­θώς και για γραφική παράσταση αριθμών (σ. 1 33) . Ένα θέμα το οποίο μπορεί να προσεγγιστεί από διά­φορα γνωστικά αντικείμενα είναι το Ή> Α κω � -

δεδομένου ότι υπάρχει στο βιβλίο ·Έκφραση:Εκθεση (σ. 1 1 9), στο Α.Π. της Χημείας (ισορροπημένη διατροφή - 5° κεφάλαιο) και στη Βιο­λογία (7" κεφάλαιο ) . Επίσης μπορεί να προσεγγιστεί και από το αντικείμενο των Θρησκευτικών μέσω της παράδοσης και των εθίμων για τα Χριστούγεννα και τη Σαρακοστή . Τέλος από τη μεριά των Μαθηματι­κών μπορεί να γίνει επεξεργασία ενός ερωτηματολο­γίου σχετικά με τις διατροφικές συνήθειες των μαθη­τών που μετέχουν στο project, ή και για άλλους μα­θητές της Τάξης ή και όλου του σχολείου.

Βιβλιογραφία: Άλγεβρα, Βιολογία, Γραμματική της Αρχαίας Ελληνι­κής, Έκφραση _:Εκθεση, Φυσική, Χημεία Α ' Λυκείου Ματσαγγούρα Η . , «Θεωρητικές Αρχές και Διδακτικές επιλογές», στο Ή καινοτομία των ερευνητικών εργα­σιών στο Λύκειο ' , σσ. 9- 1 23 . Γκότση Χρ. , Μεταλληνός π . Γ . & Φίλιας Γ. , «Ορθόδοξη πίστη και λατρεία», Α' Ενιαίου Λυκείου, ΟΕΔΒ, ε' έκδοση 2004 . www.oepek.gr www .eduscol .education. fr/00050/themes.htm

ΕΥΚΛΕΙΔΗΣ Β ' 83 τ. 3/66

Page 69: Ευκλειδης Β 83

y2 - X J Α Ι Ι . Αν Ι <χ<2 ( ! ), 3<y<4 (2) και Α =-- , τότε να

xy

δ 'ξ , I Α 1 5

ει ετε οτι: - < < - . 6 4 Απόδειξη : Έχουμε: (2) =>9<y <i6 και ( i)=>i<x3 <8=> => -8 < -Χ3 < -\ => 9 + ( -8) < / +( -χ3 ) < 1 6 + ( -!) => => Ι < y2 - χ 3 < 1 5 (α) . Επίσης: ( 1 ) , ( 2) => 3 < xy < 8 =>

=> ! < __!_ < ! (β) , [θυμηθείτε ότι για να πολλαπλασιάσουμε 8 xy 3

δ, '

'

α<β} ' 'λ ' δ ' δ υο ομοιοστροφες ανισοτητες κατα με η αρκει υο ι-γ <δ

αγώνιοι όροι να είναι θετικοί π.χ. β>Ο, γ>Ο και όχι όλοι οι ό­ροι θετικοί όπως πλεονασματικά αναφέρεται στο Σχολικό βι­βλίο] . Οπότε:

(α),(β) => 1 ·i <{y -χ3) · � < 15 ·�=>

=> ! < / - χ3 < 5 => Α ε (! , 5) = D1 ( 1 ) . 8 xy 8

Εξάλλου: Α = j'_ - χ2

και ( Ι ) => ..!._ < ..!._ < 1 (3) , οπότε χ Υ 2 χ (2) , (3) => 3 · .!. < y · .!. < 4 · Ι => � < r < 4 (γ)

2 χ 2 χ Επίσης: ( 1 ) => 1 < χ2 < 4 και ( 2) => ..!._ < ..!._ < ! , οπότε

4 y 3

I 2

I I I χ 2 4 4 χ 2 Ι , 1 · - < χ · - < 4 · - => - < - < - => -- < -- < -- (δ) Άρα: 4 y 3 4 y 3 3 y 4

(γ) . (δ) =>% + ( -�) <� + ( _ x;J < 4 + ( -υ=>

::::? ..!_ < Ι _� < _!2 => Α ε (..!.. .!2) = Ο ( ΙΙ ) . 6 χ y 4 6 ' 4 2

Τελικά: ( Ι ) , ( ΙΙ ) => Α ε D1 n D2 = D2 = 0·, 1:) . Ερώτημα: Υπάρχει ή όχι περίπτωση να είναι q � ;ι,D_ι; • Σκοπός της άσκησης αυτής είναι να συνειδητοποιήσουν οι μα­θητές της Α' Λυκείου ότι, επειδή δεν μπορούμε να αφαιρέσουμε ή να διαιρέσουμε ομοιόστροφες ανισότητες κατά μέλη, τις πρά-

ξεις α-β και α:β τις γράφουμε αντιστοίχως α+( -β) και α · ..!._ . β

Β Ι Ο. Τρεις φίλοι, οι Α, Β και Γ, έχουν από ένα περιβόλι με μηλιές. Ο πρώτος παρήγαγε 724 μήλα, ο δεύτερος 857 και ο τρίτος 1 503 μήλα. Ο Α είπε ότι αν μοίραζε εξίσου τα μήλα του, σε όλες τις οικογένειες του χωριού, τότε θα του περίσ­σευαν κάποια μήλα. Ο Β, που άκουσε, πόσα μήλα θα περίσ­σευαν, απάντησε ότι αν έπαιρνε αυτά τα περίσσεια μήλα, θα μπορούσε να

Επιμέλεια: Αντώνης Κυριακόπουλος Γιώργος Τασσόπουλος

μοιράσει τα δικά του μήλα, στις οικογένειες του χωριού, χωρίς να περισσέψει κανένα μήλο. «Περίεργο», απάντησε ο Γ. «το ίδιο ακριβώς θα μπορούσε να γίνει και στη δική μου περίπτω­ση .. . Πόσες οικογένειες έχει το χωριό;

Κώστας Καββαδ ίας - 2" Γ ΕΛ Κi:ρκυρ ας

Λ\)ση

Ονομάζουμε χ το πλήθος των οικογενειών του χωριού. Σύμ­φωνα με τις πληροφορίες που διαθέτουμε πρέπει να ισχύουν οι ακόλουθες τρεις ισότητες: 724=χλ+υ ( !) , 857 + υ = χμ (2) , 1 503 + υ = χν (3) Από τις ( I ), (2) προκύπτει ότι: 857 + 724 = χ (μ + λ) 1 58 1 = χ (λ + μ) => 3 · 1 7 · 3 1 = χ (λ + μ) (4) . Από τις ( I ) και (3) προκύπτει ότι 1503+ 724= χ(λ+ν) 2227 = χ (λ + ν) => 1 7 · 1 3 · 1 = χ (λ + ν) (5) . Από τις (4),(5) προκύπτει ότι x= l 7, αφού 1 7 είναι ο μοναδικός αριθμός που διαιρεί ταυτόχρονα τους 3 · 1 7 · 3 1 και 1 7 · 1 3 · I .

Γ9. Συνάρτηση f ικανοποιεί τη σχέση: ( I ) f (f (x )) = x - 2, για κάθε χ ε ffit . α) Να δείξετε ότι η f είναι " 1 - 1 ". β) Να δείξετε ότι για κάθε χ ε ffit η κατακόρυφη απόσταση ανάμεσα στην Cr και cΓ, είναι ίση με 2 μονάδες.

Μ ίλτος Γpαμμι':νος - Κέρκυρα

Λ\Jση :

α) Έστω χ 1 , Χ2 ε ffit τέτοιοι ώστε: f(x1 ) = f(x2 ) (2) (!)

Τότε: (2) =>ψ(x1 )} =f(f(χz)}=>x1 -2=χz -2=>x1 =χz => Η f είναι « 1 - 1 ». Άρα f ανnστρέφεται. Αρχικά πρέπει να δείξουμε ότι η Γ1 έχει σύνολο ορισμού το ffit , δηλαδή ότι f(ΊHt) = ffit . Γι'

αυτό αρκεί να δείξουμε ότι για κάθε yεffit, υπάρχει χ εffit ώστε

f( χ) =y. Πράγματι, για χ =f( 2 + y) από ( 1 ) r( f{2+y)} =2+y-2=y.

β) Στην ( I ) θέτουμε f ( χ) = y , δηλαδή χ = Γ1 (y) και έχου­

με: ( 1)=>f(y) =Γ1 (y) -2, για κάθε y ε ffit => Γ1 (χ ) - f (χ ) = 2 για κάθε χ ε ffit. Άρα σε ορθοκανονικό σύστημα (0, Ι,} ) η

cΓ, αποτελεί παράλληλη μεταφορά της cr κατά διάνυσμα 2} . Π pοη:ινόμενα θέματα :

Α 1 2 Ένας αγόρασε κάποιο αντικείμενο λιγότερο από 40€ και το πούλησε 21€. Έτσι ζημιώθηκε επί τοις εκατό, όσο αγόρασε το αντικείμενο αυτό. Πόσα χρήματα έχασε; Β Ι Ι .Να λυθεί η εξίσωση:

(�2 - J3 )x + {2συνl 5° )' = �J9 - x2 ( Ι )

Γ Ι Ο. Ν α βρεθούν οι συνεχείς συναρτήσεις f : (Ο, +α:>) � ffit ώστε να ισχύει:

xf (y) + yf ( x ) = 2xf (x ) ( Ι ) για κάθε χ, y ε (Ο,+οο) .

Καλοκαιρινά σχολεία : (Γυμνάσιο - Λύκειο) Πιερία (8 - 1 4 Ιουλίου 201 2), Ημαθία (5- 1 1 Αυγούστου 201 2)

Καλοκαιρινή Κατασκήνωση : Σοφικό Κορινθίας : Δημοτικό (Δ, Ε, Στ ' ) (28 Ιουλίοu-4 Αυγούστου 20 1 2 και 5 - 1 2 Αυγούστου 201 2 )

Πληροφορ ί ε ς Γραφ ε ία Ε.Μ.Ε. τηλ. : 2 103616532 -2103617784, www.hms . r - info@ h ms .gr

ΕΥΚΛΕΙΔΗΣ Β ' 83 τ. 3/67

Page 70: Ευκλειδης Β 83

Ο Ε u κλ�f δnς . n ροτ�fν�ι .......

«Η καρδιά των μαθηματικών είναι τα προβλήματα και οι λύσεις και ο κύριος λόγος ύπαρξης

του μαθηματικού είναι να λύνει προβλήματα». P. R. HALMOS

Επιμέλεια: Γ. Κ. Τ Ρ Ι ΑΝΗ)Σ - Ν. Θ ΑΝΤΩΝ Ο Π ΟΥ ΛΟΣ - Θ. Α . ΤΖΙΩΤΖΙΟΣ

ΑΣΚΗΣΗ 1 84 ( ΤΕΥΧΟΥΣ 76 ) χ2 = Ι Λ Ρ( χ, y, z) = Ο . Δηλ, χ = ± I Λ y = z = Ο . Τελι-Να αποδειχθεί ότι η εξίσωση ( Ι ) (x + k)98 + χ1 1 - ν14 = 0 ,

όπου k , ν είναι δοσμένοι θετικοί ακέραιοι, δεν έχει α-

κέραιη και θετική ρίζα. (Απόστολος Καζακόπουλος - Θεσσαλονίκη) Λ ΥΣΗ (Βαγγέλης Μουρούκος - Αγρίνιο )

Υποθέτουμε ότι ο θετικός ακέραιος χ0 είναι μία λύση

της εξίσωσης ( Ι ), οπότε ισχύει: (Χα +k)98 +Χσ1 1 -ν14 =0(2).

Από τη σχέση (2) προκύπτει άμεσα ότι (χ0 + k)98 < ν14

κά, το σύνολο λύσεων Λ = { (0, Ο, Ο) , (1 , Ο, Ο) , ( - I, Ο, Ο)} . Λ ΥΣΗ 2η (Γιώργος Μ ήτσιος - Ράμια Άρτας) Η εξίσωση ( 1 ) γράφεται ισοδυνάμως:

(χ2 + y2 )2 + (y2 + z2 )2 + z2 = χ2 Προφανώς, για τις α-

κέραιες λύσεις της εξίσωσης, ισχύουν: jO :O:: (x 2 + / )2 :0:: x 2 {O :O:: x 2 + / :0:: I x l (Ι ) O :O:: (yz + zz )z :O:: xz <=> O :O:: yz + zz :o:: Ι x l (2)

O :O:: z2 :0:: x 2 ο :ο:: Ι z Ι :ο:: Ι χ l (3)

και ισοδύναμα τη σχέση (χ0 + k)7 < ν (3) . Από το (Ι ) => Χ2 :0:: l x l <=> 14 - l x l :0:: Ο <=> χ = O v l x l :0:: Ι

διώνυμο του Νεύτωνα (κρατώντας μόνο τους δύο τελευ- <=> χ = Ov lxl = Ι (αφού χ ε Ζ ) . ταίους όρους του, έχουμε: [(χ0 + k)7 + 1 ] 1 4 =

tσ( ];}χσ + k)7 i > (χο + k)98 + 1 4((χο + k)9 ι >

(χο + k)9s + χ�ι = ν ι4 => (χο + k)Ί + I > ν (4)

Από τις (3),(4) ότι (χ0 + k)7 < ν < (χ0 + k)7 + I που υπο-

δηλώνει το άτοπο, αφού ο θετικός ακέραιος ν βρίσκεται μεταξύ δύο διαδοχικών ακεραίων. Άρα, η εξίσωση ( 1 ) δεν έχει λύση στο σύνολο των θετικών ακεραίων. Λύσεις έστειλαν επίσης οι συνάδελφοι: Κωνσταντίνος

Τζαγκαράκης - Χανιά, Ροδόλφος Μπόρης - Δάφνη . ΑΣΚΗΣΗ 1 85 ( ΤΕΥΧΟΥΣ 76 ) Να βρεθούν οι τιμές των ακεραίων αριθμών χ , y , z

που επαληθεύουν : ( 1 ) : (χ2 +y)2 +(Υ +z2 )2 -χ2 +z2 = 0

(Δη μήτριος Καρβέλας - Πεύκη ) ΛΥΣΗ lη (Γιάννης Ηλιόπουλος - Καλαμάτα) Η εξίσωση ( 1 ) γράφεται ισοδυνάμως:

χ 4 + 2χ 2 y2 + y4 + (yz + 22 )2 + 22 = χ 2

<::::> x4 + P(x, y, z) = x2 (2) όπου, έχει τεθεί:

P(x, y, z) = 2x2y2 + y4 + (/ + z2 )2 + z2 2 Ο

Όμως, χ ε Ζ ::::> χ2 = 0v χ2 2 1 , οπότε x4 z x2 •

Έτσι, για χ = Ο , (2) <=> P(x, y, 2) = 0 <=> y = 2 = 0 Άν Jt >l

τότε χ 4 > χ2 και η (2) είναι αδύνατη . Άρα, πρέπει

Αν χ = Ο , (2)<=>y +i =O<=>y=2=0 . Αν l x l = 1 ,

(l)<=>l+y = 1<=>y=0 και η αρχική γράφεται:

1 + z4 + 22 = 1 <=> 22 = Ο <=> 2 = Ο . Τελικά, οι μοναδικές

λύσεις της εξίσωσης είναι οι τριάδες (x, y, 2) = (O, O, O) v (l , O, O) v (- 1 , 0, 0) . ΛΥΣΗ 3η (Κουκάκη.; �ημήτριος - Κιλκίς , Π αύλου

Νικόλαος - Χαλκίδα, Ζεί.εβάρη Άννα - Θεσσαλονίκη)

Έστω (χ , y, 2) μία ακέραιη λύση της ( 1 ) . Η εξίσωση ( Ι )

Υποθέτουμε ότι είναι 2 * Ο .τότε ισχύει χ2 -22 < χ2 και

πο. Άρα, είναι 2 = Ο και η ( 1 ) γράφεται:

(χ2 + y2 )2 + y4 = χ2 (4) . Υποθέτουμε ότι είναι

Άτοπο, λόγω της ( 4 ). Άρα, είναι y = Ο και η ( 1 ) δίνει: { χ = Ο x4 - x 2 = 0 <=> xz (xz - 1) = 0 <=> χ = 1

χ = - 1

ΕΥΚΛΕΙΔΗΣ Β ' 8 3 τ . 3/68

Page 71: Ευκλειδης Β 83

--------------- 0 Ευκλείδης Προτείνει . . . --------------­Τελικά, το σύνολο λύσεων της ( 1 ) είναι Λ = { (O, O, O) , (l , 0, 0) , (- 1 , 0, 0)} . Λύσεις έστειλαν επίσης οι συνάδελφοι: Κωνσταντίνος Τζαγκαράκης - Χανιά, Γιαννακοπούλου Ιωάννα - Άν­δρος, Βαγγέλης Μουρούκος - Αγρίνιο, Ορέστης Κα­τσάνος - Πρέβεζα, Γιώργος Αποστολόπουλος - Μεσο­λόγγι, Γιάννης Σταματογιάννης - Δροσιά, Διονύσης Γιάνναρος - Πύργος, Ροδόλφος Μπόρης - Δάφνη.

ΑΣΚΗΣΗ 1 86 (τΕΥΧΟΥΣ 76 )

Σε τρίγωνο ΑΒΓ είναι Α = � , ΒΓ = α και μα η διάμε­

σος, που αντιστοιχεί στην πλευρά ΒΓ. Να

δ θ , , , α 2-J3 ει χ ει οτι ισχυει: - :2: -- . μα 3

(Γιώργο'iι Αποστολόπουλος - Μεσολόγγι ) Λ ΥΣΗ 1 (Στέλιος Πετρολέκας - Δραπετσώνα ) Από το θεώρημα των συνημιτόνων στο τρίγωνο ΑΒΓ, έχουμε:

Α

Β

Γ

2 7 7 7 2 1 α = β- + γ- - 2βγσυνΑ = β- + γ - 2βγ-2

� α2 = β2 + γ2 - βγ � 2α2 = 2β2 + 2γ2 - 2βγ � 2α2 = β2 + γ2 + (β - γ)2 � 2α2 :2: β2 + γ2 (1) (η ισότητα ισχύει όταν και μόνον όταν β=γ )

Από το πρώτο θεώρημα της διαμέσου στο τρίγωνο r Ί 1 7 α2 ΑΒΓ εχουμε: β- + γ- = 2μ� + 2 (2)

Από την ( 1 ) , λόγω της (2), προκύπτει ότι : α2 α2 4 2α2 :2: 2μ� + - � 3α2 :2: 4μ� � 2 :2: -2 μα 3

α 2 4 α 2 α 2-J3 .

� (-) :2: - � - :2: - � - :2: --μα 3 μα -J3 μα 3

Λ ΥΣΗ 2" (Ορέστης Κατσάνος - Πρέβεζα,) Η αποδεικτέα σχέση γράφεται ισοδύναμα

J3 > ' 'λ ' θ 'ζ ' α2 - μα , το πρωτο με ος της οποιας υμι ει το υψος

ισοπλεύρου τριγώνου πλευράς α. Κατασκευάζουμε ισόπλευρο τρίγωνο ΔΒΓ πλευράς α και τον περιγεγραμμένο κύκλο του , που ταυτίζεται με τον περιγεγραμμένο κύκλο του τριγώνου ΑΒΓ , αφού Λ Π Λ ΒΔΓ = - = ΒΑΓ .

3

Ε , ΔΜ αJ3 τ ' ο ' ι ναι = -- . ο περικεντρο του τριγωνου ΑΒΓ 2

ανήκει στη ΔΜ και είναι (ΟΔ) = (ΟΑ) = R . Στο τρίγωνο ΟΑΜ είναι: (ΑΜ) :$ (ΟΑ) + (ΟΜ)

<=> (ΑΜ) :$ (ΟΔ) + (ΟΜ) = (ΔΜ) = α-J3 � 2

α-J3 α 2-J3 μα :$ -- <::> - :2: --2 μα 3

Δ

Λύσεις έστειλαν επίσης οι συνάδελφοι: Κωνσταντίνος Τζαγκαράκης - Χανιά, Γιαννακοπούλου Ιωάννα - Άνδρος, Βαγγέλης Μουρούκος - Αγρίνιο, Γιάννης Σταματογιάννης - Δροσιά, Διονύσης Γιάννα­ρος - Πύργος, Ροδόλφος Μπόρης - Δάφνη, Αντώνης Ιωαννίδης - Λάρισα, Δημήτριος Κουκάκης - Κάτω Απόστολοι Κιλκίς , Βασίλειος Ν ικολάκης - Θεσσαλο­νίκη , Φίλιππος Σερέφογλου - Μελίσσια, Διονύσης Γιάνναρος - Πύργος, Γεώργιος Μήτσιος - Ράμια Άρ­τας, Ιωάννης Ανδρής - Αθήνα.

ΑΣΚΗΣΗ 1 87 ( ΤΕΥΧΟΥΣ 76 )

Δίνεται τρίγωνο ΑΒΓ με Α = 1 44° και ΑΓ = Φ , όπου ΑΒ

Φ ο λόγος της χρυσής τομής. Στην πλευρά ΑΓ θεωρούμε σημείο Η τέτοιο, ώστε να

ισχύει ΑΒ = 2Φ ενώ στην προέκταση της ΒΑ θεωρού-ΑΗ

με τμήμα ΑΘ=ΑΗ . Ονομάζουμε Ζ την τομή της ΘΗ με την ΒΓ.

1 ) Να υπολογισθεί ο λόγος των εμβαδών: (ΑΒΓ) (ΑΖΘ)

2) Να δειχθεί ότι η ΑΗ διχοτομεί την γωνία ΖΑΘ. (Γεώργιος Μήτσιος - Ράμια Άρτας ) Λ ΥΣΗ (Βαγγέλης Μουρούκος - Αγρίνιο)

Ι ) Εφαρμόζουμε το Θεώρημα του Μενελάου στο τρί­γωνο ΑΒΓ με διατέμνουσα ΖΗΘ και έχουμε:

ΖΓ . ΘΒ . ΗΑ = 1 (1) ΖΒ ΘΑ ΗΓ

ΕΥΚΛΕΙΔΗΣ Β' 83 τ. 3/69

Page 72: Ευκλειδης Β 83

---------------0 Ευκλείδης Προτείνει . . . --------------­Γ

Θ Είναι, όμως: ΑΓ = ΑΓ . ΑΒ = φ . 2φ = 2φ2 = 2( 1 + .J5

)2 = ΑΗ ΑΒ ΑΗ 2

= 3 + .J5 =.> ΗΓ = ΑΓ - ΑΗ = ΑΓ _ 1 = 2 + .J5 ΑΗ ΑΗ ΑΗ

ΗΑ 1 =.> ΗΓ

= 2 + .J5

(2)

Επίσης, είναι ΘΒ = ΘΑ + ΑΒ = 1 + ΑΒ = 1 + ΑΒ = 1 + 2Φ ΘΑ ΘΑ ΘΑ ΑΗ

=.> ΘΒ = 1 + 2 1 + .J5 = 2 + J5 (3) ΘΑ 2

(1)� ΖΓ · (2 + .J5) ·� = 1 =.> ΖΓ = ΖΒ (3) ΖΒ 2 + ν5

Β

Δηλ, το Ζ είναι το μέσον της πλευράς ΒΓ, οπότε είναι (ΑΒΓ) = 2(ΑΒΖ) . Τα τρίγωνα ΑΒΖ και ΑΖΘ έχουν δύο γωνίες τους παραπληρωματικές, οπότε

(ΑΒΖ) = ΑΒ · ΑΖ = ΑΒ = ΑΒ = 2φ (ΑΖΘ) ΑΖ · ΑΘ ΑΘ ΑΗ

Τελικά (ΑΒΓ) = 2(ΑΒΖ) = 4Φ . ' (ΑΖΘ) (ΑΖΘ)

Από το θεώρημα του συνημιτόνου στο ΑΒΓζ

λ ' ' (1 44° ) .J5 + 1 φ

' ογω οτι συν = - -- = -- , εχουμε : 4 2 ΒΓ2 = ΑΒ2 + ΑΓ2 - 2ΑΒ · ΑΓ · συνΑ =

2 2 2 φ

ΑΒ + ΑΒ · Φ - 2ΑΒ · ΑΒ · Φ · (--) = 2

ΑΒ2 + 2ΑΒ2 · Φ2 = ΑΒ2 (1 + 2Φ2 ) ( 4) Από το 1 ° θεώρημα Διαμέσων στο τρίγωνο ΑΒΓ (με χρήση της σχέσης (4) ) , παίρνουμε:

ΑΒ2 + ΑΓ2 = 2ΑΖ2 + ..!._ ΒΓ2 <:::} ΑΒ2 + ΑΒ2 · Φ2 = 2

2ΑΖ2 + ..!._ ΑΒ2 (1 + 2Φ2 ) <:::} ..!._ ΑΒ2 = 2ΑΖ2 <:::} 2 2

ΑΒ2 = 4ΑΖ2 =.> ΑΖ = ..!._ = ΓΖ ΑΒ 2 ΓΒ

Οπότε, σύμφωνα με το αντίστροφο θεώρημα της εξωτε­ρικής διχοτόμου στο τρίγωνο ΑΒΖ, η ΑΓ είναι διχοτό­μος της εξωτερικής της γωνίας ΒΑΖ. Λ Δηλ. η ΑΗ διχοτομεί τη γωνία ΖΑΘ . Λύσεις έστειλαν επίσης οι συνάδελφοι: Κ ωνrπαηί\ ·ο� I Χανιά, I ΊαννακοποίJλΟΙ Ι I ω ιι ' \ ' U - Ι ι cΊ.\ ' \ 'α ιιο: Πύργος, !'ο-

Δάφνη, J< ουκιΊ. ι,ηc Δη ,ι ι ΊΊψω.: Κιλ-1 - - Πρέβεζα. - ι I ΠfΙΟ-

Δρο-

204 Αν a Ε R , τότε να δειχθεί ότι η εξίσωση χ6 + χ4 + χ 2 + ax - 1 = Ο , έχει ακριβώς δύο πραγματικές ρίζες , ετερόσημες. (Λ ντι:y · 1 ., 1 cΨινν ίιiης Λάρισα)

2 0 5 . Αν k Ε R , τότε να λυθεί η εξίσωση χ4 + (1 - 2k)x3 + (k2 + 2)χ2 - (k2 + 4k - 5)χ + 3(k - 1)2 = Ο ( llΊι!ψγος - - Μεσολόγγι )

2 0 6 . Δίνεται τρίγωνο ΑΒΓ με ΑΒ < ΑΓ , ο εγγε- γραμ­μένος κύκλος C(I, ρ) που εφάπτεται των ΑΒ, ΑΓ στα Ζ, Ε αντίστοιχα, ο παρεγγεγραμμένος κύκλος C α (Ι α , ρ α ) που εφάπτεται των ΑΒ, ΑΓ στα Ζ ' , Ε ' αντίστοιχα. Η ΓΙ τέμνει τη ΖΕ στο Σ και η ΓΙ" τη ΖΈ' στο Σ' . Να δει-χθεί ότι ΣΣ' I I ΑΓ . Πρέβεζα)

. Α ν α, β, γ ε R • , να λυθεί ( με τη βοήθεια των ομο-γενών συστημάτων) το σύστημα : { χ + y + z = α + β + γ

� + r + � = 3 α β γ

xz + / + z2 = αz + β2 + γ2 ( ' r . J - Αθήνα )

Δίνεται τρίγωνο ΑΒΓ ορθογώνιο στο Α με ΑΓ > ΑΒ και περίμετρο ίση με το υπόλοιπο της διαίρε­σης 22022 : 1 3 . Στο ημιεπίπεδο που ορίζει η ΒΓ και δεν ανήκει το Α, θεωρούμε σημεία Δ, Ε ώστε: ΒΔ Ι ΙΑΓ , ΓΕ Ι ΙΑΒ , ΒΔ = ΓΕ < ΑΒ . Αν Ζ η τομή των Γ Δ και ΒΕ και Η σημείο της ημιευθείας ΑΖ τέτοιο, ώ­στε 6(ΑΗ) = .J2(ΑΒΓ) , τότε 1 ) Να δειχθεί ότι το Η είναι εσωτερικό σημείο του ΑΒΓ. 2) Αν η κάθετη στη ΒΗ στο σημείο Η τέμνει τη ΒΓ στο Θ και Ι είναι η ορθή προβολή του Θ πάνω στην ΑΓ με (ΑΙ)=2, τότε να υπολογισθούν τα μήκη των πλευρών του τριγώνου.

(i ι ι!ψγος \ I Ι\ηηος - Ράμια Άρτας)

2 !19 . Να βρεθούν οι πραγματκές λύσεις της εξίσωσης: 6χ + 1 = 8' - 27 '- l ( 1 ) .

(Γι{ιννης Ηλιlιπουλος - Καλαμάτα )

ΕΥΚΛΕΙΔΗΣ Β ' 83 τ. 3170

Page 73: Ευκλειδης Β 83

Τ'ο Βήμα του Ευκλ..είδη Επιμέλεια: Γιάννης Στρατής - Βαγγέλης Ευσταθίου

Μέθοδος εύρεσης του πλήθους Π( α), αε Ν * των πρώτων φυσικών αριθμών που είναι < α

Λευτέρης Γ. Τσιλιακός Θα βασιστούμε στο γνωστό θεώρημα:

«Κάθε φυσικός αριθμός α> 1 είναι πρώτος, αν δεν έχει πρώτο θετικό διαιρέτη p, με p$ fa ».

Θεωρούμε το σύνολο Σ= { 1 ,2 ,3 , . . . , α} αρχικό απόκομμα του Ν* και έστω p ο μεγαλύτερος

πρώτος φυσικός αριθμός για τον οποίον ισχύει: p$ � . Έστω τώρα, το σύνολο Β= {2,3 ,5 , . . . , p } c Σ των διαδοχικών πρώτων, με πληθικό αριθμό λ. Θεωρούμε την 1-1 αντιστοιχία:

όπου πχ:

Α( Ι I ) , Α( Ι 2 ) , ! ! 2 3

! Ρ

• Α( Ι 2 ) είναι ο πληθικός αριθμός του υποσυνόλου του Σ, τα στοιχεία του οποίου έχουν την ιδιότη-

τα Ι 1 , δηλαδή έχουν την ιδιότητα να διαιρούνται δια του δεύτερου πρώτου, που είναι ο 3 .

• Α( Ι i. ) είναι ο πληθικός αριθμός του υποσυνόλου του Σ, τα στοιχεία του οποίου έχουν την ιδιότη­

τα Ι ;. , δηλαδή έχουν την ιδιότητα να διαιρούνται δια του λ-οστού πρώτου, που είναι ο p. Με χρήση του συνδυαστικού τύπου του Sylvester έχουμε : λ λ

λ

ο = α - ΣΑ(Ι) + ΣΑ(Ιj , Ιμ ) - Σ Α(Ιj , Ιμ , Ι ν ) + . . . + (- l)λΑ(Ι , , Ι 2 ' . . . , Ι λ ) j ,μ , j<μ

μι , ν J<μ<v

Υπολογίζουμε το πλήθος a των στοιχείων του Σ που δεν διαιρούνται με κανέναν από τους 2,3,5, . . . ,p και άρα είναι πρώτοι > p που ανήκουν στο Σ.

Στο Σ όμως ανήκει και ο 1, που είναι ένας εκ των a και πρέπει να απορριφθεί ως μη πρώτος. Άρα οι πρώτοι του Σ είναι οι 8 -1 , που μόλις επισημάναμε, καθώς και οι λ πρώτοι του συνόλου Β ( B c Σ ) . Οπότε : Π(α)=λ + ο - 1

Διευκρίνιση : Ο Α(Ιj , Ιμ , Ιν ) είναι ο πληθικός αριθμός των στοιχείων του Σ που διαιρούνται

δια του j · μ · ν , με j ,μ,ν ε Β και j<μ<ν.

Πχ: για j=3 , μ=5 , ν=7, ο αριθμός A(I2 , l3 , l4 ) είναι ο πληθικός αριθμός των στοιχείων του Σ που διαιρούνται δια του γινομένου 3 · 5 · 7 και προφανώς ισχύει:

A(l2 , 13 , 14 ) = [-a-] = ακέραιο μέρος του

_a_ . 3 · 5 · 7 3 · 5 · 7

Α(Ι 1 , 12 , . . . , 1λ ) είναι ο πληθικός αριθμός των στοιχείων του Σ που διαιρούνται δια του γινομένου

των στοιχείων του Β και ισχύει: Α (Ι , , Ι υ

. . . , Ι λ )

= [ a ] .

2 · 3 · . . . · p

Είναι φανερό ότι η μέθοδος αυτή είναι λογιστική μετάφραση του Κόσκινου του Ερατοσθένη. Για να λειτουρ­γήσει σε μεγάλα αποκόμματα του

Ν*

χρειάζεται ισχυρούς υπολογιστές, κατάλληλο πρόγραμμα της εργασίας που

ΕΥΚΛΕΙΔΗΣ Β ' 83 τ.3/71

Page 74: Ευκλειδης Β 83

Το Βήμα του Ευκλείδη

παρουσιάσαμε και χρόνο. Επισημαίνουμε απλά, ότι ο ακριβής υπολογισμός του Π( α), είναι εφικτός και ήδη έχουμε αναπτύξει σχετικό πρόγραμμα σε γλώσσα προγραμματισμού perl, με το οποίο υπολογίστηκε με οικιακό ηλεκτρονι­κό υπολογιστή, το πλήθος των πρώτων αριθμών :5 7000, που είναι 783.

Παραδείγματα: 1 . Υπολογισμός του Π(30)

• J30 = 5 ,477 . . . Άρα Β= {2,3 ,5 } , δηλαδή λ=3 . • Παρονομαστές:

ο Ανά ένας: 2, 3 , 5 ο Ανά δύο: 6, 1 0, ι 5 ο Ανά τρεις : 30

• 8 = 30 _ {[3

2

ο J + [ 33ο J + [ 35ο ]} + {[ 3

6

ο J + [�� J + [�� ]} - [�� J =

= 30 - ( 1 5 + 1 0 + 6) + (5 + 3 + 2) - ι = 30 - 3 ι + 1 0 - ι = 8 Επομένως: Π(30) = λ + a - 1 = 3 + 8 - 1 = 1 Ο. Πράγματι, οι πρώτοι :::; 30 είναι οι 2 , 3 , 5 , 7, 1 1 , 1 3 , 1 7, 1 9, 23 , 29, και είναι 1 0 .

2 . Υπολογισμός του Π(100) • Μο = 1 0, άρα Β= {2,3 ,5 ,7 } , δηλαδή λ=4. • Παρονομαστές:

ο Ανά ένας: 2, 3 , 5 , 7 ο Ανά δύο: 6, 1 0, ι 4, ι 5 , 2 ι , 35 ο Ανά τρεις: 30 , 42, 70, ι 05 ο Ανά τέσσερις : 2 ι Ο

• 8 = 1 00 _ {[ ι �ο J + [ ι �ο J + [ ι �ο J + [ ι �ο ]} + + {[

ι �ο J + [ \ο0ο J + [ \ο4ο J + [ \ο5ο J + [ι�� J + [ ι3ο5ο ]}

-{[ι3ο�] + [ι��] + [ι��] + [:��]} + [���] =

= 1 00 - (50 + 33 + 20 + ι 4) + ( ι 6 + 1 0 + 7 + 6 + 4 + 2) - (3 + 2 + ι + Ο) + Ο = = 1 00 - ι ι 7 + 45 - 6 = 22. Επομένως: Π(100) = λ + 8 - ι = 4 + 22 - ι =25 .

3 . Υπολογισμός του Π(127) • Jl27 = ι ι ,269, άρα Β= {2, 3 , 5 , 7, ι ι } , δηλαδή λ=5 . • Παρονομαστές:

ο Ανά ένας: 2, 3 , 5 , 7, ι ι ο Ανά δύο : 6, 1 0, ι 4, 22, ι 5 , 2 ι , 33 , 35 , 55 , 77 ο Ανά τρεις: 30 , 42, 66, 70, 1 05 , ι 1 0, ι 54, ι 65 , 23 ι , 385 ο Ανά τέσσερις : 2 1 0, 330, 462, 770, ι ι 55 ο Ανά πέντε: 23 1 0.

• 8 = ι 27 _ {[ ι�7 ] + [ ι�7 ] + [ ι�7 ] + [ ι�7 ] + [ ι:ι7]} +

+{[ι�7] + [\

2;] + [\

2:] + [ι;;] + [\

257] + [ι�;] + [ι:;] + [ι:;] + [ι;;] + [ι;;]}

-{[1:;] + [ι:;] + [ι:;] + [ι;;] + [:��] + [���] + Ο + Ο + Ο + 0} +

+ (όλα τα άλλα Ο, γιατί οι παρονομαστές είναι > ι 27) = = ι 27 - (63 + 42 + 25 + ι 8 + ι ι ) + (2 ι + ι 2 + 9 + 5 + 8 + 6 + 3 + 3 + 2 + ι ) - (4 + 3 + ι + ι + ι + ι ) = = ι 2 7 - ι 59 + 70 - ι ι = 27. Επομένως: Π(127) = λ + 8 -ι = 5 + 27 - ι = 3 ι .

Β ιβλιογραφία: «Εισαγωγή εις την Αριθμοθεωρίαν» [σελ. ι 83] , Παναγ. Μάγειρα, 'Εκδοση [ 1981] .

ΕΥΚΛΕΙΔΗΣ Β ' 81 τ. l/72

Page 75: Ευκλειδης Β 83

Το Βήμα του Ευκλείδη

Εφαρμογές της Γεωμετρ ι κής Π ιθανότητας Γιώργος Πετράκος, Πάντειο Πανεπιστήμιο

Ο κλασικός ορισμός της πιθανότητας εμφάνισης ενός ενδεχόμενου Ε που ανήκει σε ένα δειγματικό χώρο Ω, όπου τα Ε, Ω είναι σύνολα διακριτών, απλών και ισοπίθανων ενδεχομένων,

είναι: Ρ(Ε) = :i�) όπου n(E) είναι το πλήθος των στοιχείων του συνόλου Ε και η(Ω) είναι το

πλήθος των στοιχείων του δειγματικού χώρου Ω. Α ν ο δειγματικός χώρος Ω ενός τυχαίου πειρά­ματος είναι ένα συνεχές σύνολο το οποίο μπορεί να παρασταθεί ως ένα γεωμετρικό σχήμα σε μία, δυο ή περισσότερες διαστάσεις και Α είναι ένα ενδεχόμενο (υποσύνολο) του Ω που μπορεί να παρασταθεί ως ένα γεωμετρικό σχήμα ίδιας διά­στασης με το Ω, η πιθανότητα εμφάνισης του Α εί-

ναι: Ρ(Α) = μ(Α)

όπου μ είναι το κοινό μέτρο των μ( Ω)

παραπάνω συνόλων, το οποίο απλά είναι το μήκος στην μία διάσταση, το εμβαδόν στις δύο, ο όγκος

λ Σ ' '

δ '

( '

θ '

) Ρ(Α) --μήκος(Α) Σε δυο στις τρεις κ. π. υγκεκριμενα σε μια ιασταση πραγματικη ευ εια :

μήκος( Ω)

διαστάσεις (επίπεδο) : Ρ(Α) = εμβαδ�ν(Α)

. Σύμφωνα με τον παραπάνω ορισμό, η πιθανότητα εμβαδον(Ω)

ενός ενδεχόμενου Α ως προς ένα δειγματικό χώρο Ω είναι η αναλογία των μέτρων τους. Αυτό προϋποθέτει ομοιομορφία στην κατανομή πιθανότητας μέσα στο Ω, που σημαίνει ότι δύο οποια­δήποτε υποσύνολα-ενδεχόμενα ιδίου μέτρου (μήκους, εμβαδού, κ.λπ.) έχουν ίδια πιθανότητα να εμφανιστούν. Επειδή τα σύνολα Α, Ω παριστάνονται από γεωμετρικά σχήματα και ο υπολογι­σμός της ζητούμενης πιθανότητας εμπεριέχει στοιχεία είτε Ευκλείδειας είτε Αναλυτικής Γεωμε­τρίας, η πιθανότητα ονομάζεται Γεωμετρική . Ας δούμε μερικές εφαρμογές της Γεωμετρικής Πι­θανότητας

Λ ΥΛ1ΕΝΑ ΠΡΟΒΛ ΗΜΑ ΤΑ 1 . Αν σπάσουμε ένα μακαρόνι τύπου σπαγγέτι μήκους 26 εκ., τυχαία σε δυο κομμάτια, να

υπολογιστεί η πιθανότητα το ένα κομμάτι να είναι μεγαλύτερο από 20 εκατοστά (του μέ­τρου).

Λύση Έστω ότι το σημείο που σπάμε το μακαρόνι είναι ένα οποιοδήποτε σημείο του διαστήματος

(0,26). Για να είναι το ένα κομμάτι μεγαλύτερο από 20 εκατοστά θα πρέπει το μακαρόνι να σπάσει είτε στα 6 πρώτα εκατοστά (0,6) είτε στα 6 τελευταία (20,26).

ο 6 20 26

Άρα η ζητούμενη πιθανότητα με δεδομένα τα μήκη των αντίστοιχων διαστημάτων είναι: 6 + 6 6 '

Ρ(Ε) = -- = - = Ο, 46 1 5 περιπου 46%. 26 1 3

2. Σε ένα παιχνίδι με βελάκια ο παίκτης έχει να επιλέξει ανάμεσα σε δυο στόχους Α και Β. Με ποια επιλογή έχει μεγαλύτερη πιθανότητα να πετύχει τον γραμμοσκιασμένο στόχο;

ΕΥΚΛΕΙΔΗΣ Β ' 81 τ.l/73

Page 76: Ευκλειδης Β 83

Για να υπολογίσουμε την πιθανότητα επιτυχίας στο στό­χο Α, Ρ(Α) θα πρέπει πρώτα να βρούμε το εμβαδόν του τετραγώ­νου με πλευρά α, Ε(Ω) και το εμ­βαδόν του εγγεγραμμένου κύκλου με ακτίνα α/2 , Ε(Α) . Ε(Ω) = α2

2 α2 και Ε(Α) = πR = π-

4 Άρα η ζητούμενη πιθανότητα

Ε (Α) π είναι: Ρ(Α) = -- = -Ε (Ω) 4

Το Βήμα του Ευκλείδη

- - - - - - - - - α - - - - - - - - - α

Για το δεύτερο στόχο Β, το εμ- Α Β βαδόν του τετράγωνου πλαισίου είναι το ίδιο (α2) ενώ το εμβαδόν των τεσσάρων ίσων κύκλων με ακτίνα α/4 είναι: Ε(Β) =

2 2 Ε (Β) 4 π� = π� . Άρα η ζητούμενη πιθανότητα είναι: Ρ(Β) = -(-

)= π

1 6 4 Ε Ω 4

'Ετσι καταλήγουμε ότι και στις δυο περιπτώσεις η πιθανότητα είναι ίδια ( π = 0,785) . 4

3. Ο Bryan κάθε πρωί ξεκινάει με το υδροπλάνο του από την Anchorage της Αλάσκα για να εφοδιάσει με τρόφιμα κάποιες δυσπρόσιτες περιοχές της επαρχίας. Οι προορισμοί βρί­σκονται μέσα σε μία ακτίνα 200 μιλίων και ο Bryan κάθε πρωί γεμίζει το ρεζερβουάρ με καύσιμα που διαρκούν για πτήση 500 μιλίων. Ένα πρωί ο Bryan έφυγε εσπευσμένα για ένα προορισμό έχοντας πληροφορίες ότι κατά μήκος της διαδρομής του γινόταν παράνο­

μο κυνήγι άσπρου λύκου. Ταραγμένος ο Bryan ξέχασε να γεμίσει το ρεζερβουάρ και ξε­κίνησε με την ελπίδα να συναντήσει τους λαθροκυνηγούς και να ειδοποιήσει τις αρχές. Ποια είναι η πιθανότητα να μην μπορέσει να ολοκληρώσει την πτήση ;

D 300

I I

Έστω χ 1 η απόσταση που διάνυσε ο Bryan για να πάει στον προορισμό του και να γυρίσει την προηγούμενη μέρα από το συμβάν και xz η απόσταση που πρέπει να διανύσει για να επιστρέψει στην βάση του την ημέρα που ειδοποιήθηκε για το παράνομο κυ­νήγι. Εφόσον ο Bryan έχει καύσιμα για πτήση 500 μι­λίων θα έχει πρόβλημα αν χι + χ2 > 500. Αν τοποθετή­σουμε τα χ ι και Xz σε ένα ορθογώνιο σύστημα αξόνων παρατηρούμε ότι το πρόβλημα για τον Bryan δημιουρ­γείται όταν το ζευγάρι (χ ι ,χ2) παίρνει τιμές στο τριγω­νικό τμήμα D.To εμβαδόν του τριγώνου D είναι: �

E(D) = 300 · 300 = 90000 = 45000 2 2

400 500

Ενώ το εμβαδόν που αντιστοιχεί στον δειγματικό χώρο Ω είναι αυτό του τετραγώνου με πλευρά 400, δηλαδή : Ε(Ω) = 400 · 400 = 1 60000 . Άρα η ζητούμενη πιθανότητα εί-

ναι: P(D) = Ε (

( D )

) = 45 = Ο, 28 1 3 . Άρα η πιθανότητα ο Bryan να μείνει από καύσιμα κατά την

Ε Ω 1 6 διάρκεια της πτήσης την δεύτερη μέρα είναι περίπου 28%. 4. Ο Στέφανος και η Λένα δουλεύουν σε διαφορετικά τμήματα μιας μεγάλης ασφαλιστικής

εταιρείας, χωρίς να έχουν επίσημα γνωριστεί. Συνηθίζουν όμως και οι δυο να κατεβαί-

ΕΥΚΛΕΙΔΗΣ Β' 81 τ. l/74

Page 77: Ευκλειδης Β 83

Το Βήμα του Ευκλείδη

νουν στην καφετέρια στο ισόγειο για ένα δεκάλεπτο διάλειμμα για καφέ καθημερινά, α­νάμεσα στις 1 1 :00 και στις 12 :00. Ποια η πιθανότητα να συναντηθούν στην καφετέρια

μια οποιαδήποτε ημέρα της εβδομάδας; Πόσο θα πρέπει να παραμείνει στην καφετέρια ο Στέφανος ώστε να αυξήσει την πιθανότητα του να συναντήσει την Λένα σε 50% ;

Λύση : Έστω ότι ο Στέφανος φτάνει στην καφετέρια χ λεπτά μετά τις 1 1 :00, χ ε [0,60] , ενώ η Λένα φτάνει y λεπτά μετά τις 1 1 : 00 y ε [0,60] . Ο δειγματικός χώρος πε­ριγράφεται από έναν σύστημα αξόνων xy και προσδιορί­ζεται από το τετράγωνο ΑΒΓΔ.

Αν ο Στέφανος φτάσει πρώτος χ λεπτά μετά τις 1 1 :00, θα συναντήσει την Λένα αν αυτή φτάσει σε y λε­πτά μετά τις 1 1 : 00 έτσι ώστε y - χ < 1 0 (περιοχή ΑΕΖΓ). Αντίστοιχα αν η Λένα φτάσει πρώτη θα συναντηθούν αν ο Στέφανος φτάσει σε χ λεπτά με τις 1 1 :00 έτσι ώστε χ -

Υ

Δ

Ε

z r

Η

y < 1 0 (περιοχή ΑΘΗΓ). Άρα η ζητούμενη πιθανότητα ιL.------L---------, υπολογίζεται ως ο λόγος των επιφανειών ΑΕΖΓΗΘ και Α ΑΒΓΔ. Δηλαδή : Θ

50 · 50 50 · 50 Β χ

( ) Ε (ΑΕΖΓΗΘ) _ Ε(ΑΒΓΔ) - Ε(ΕΔΖ) -Ε(ΘΗΒ) 3600 --2- - -2- 1 1 00

Ρ Ε = - = = -- = Ο 3056 Ε (ΑΒΓΔ) Ε(ΑΒΓΔ) 3600 3600 '

Αντί να υπολογίσουμε το εμβαδόν ΑΕΖΓΗΘ απευθείας (με εμβαδά τραπεζίων) το κάνουμε αφαιρώντας από το εμβαδόν του ΑΒΓ Δ τα εμβαδά των δύο συμπληρωματικών τριγώνων ΕΔΖ και ΘΗΒ. Στους παραπάνω υπολογισμούς λάβαμε υπόψιν ότι τα ευθύγραμμα τμήματα ΑΕ, ΑΘ, ΖΓ και ΓΗ έχουν μήκος 1 0 ενώ τα ΑΔ, ΔΓ, ΓΒ και ΑΒ έχουν μήκος 60. Συνεπώς οι κάθετες πλευρές των δύο τριγώνων ΕΔ, ΔΖ, ΗΒ, ΘΒ έχουν μήκος 50. Τελικά η πιθανότητα ο Στέφανος και η Λένα να συναντηθούν στην καφετέρια μια οποιαδήποτε ημέρα είναι περίπου 30%.

Ας δούμε το ίδιο πρόβλημα λαμβάνοντας υπόψη ότι ο Στέφανος θα παραμείνει τόσο ώστε η πιθανότητα να συναντήσει την Λένα να είναι 50% (Ρ(Ε) = 1 /2) . Αν η Λένα φτάσει πρώτη, τότε θα συναντηθούν αν ο Στέφανος φτάσει σε χ λεπτά μετά τις 1 1 :00 έτσι ώστε x-y< 10 (όπως στο προηγούμενο ερώτημα) ασχέτως πόσο θα μείνει ο Στέφανος εφόσον η Λένα θα φύγει στο δεκά­λεπτο. Αν ο Στέφανος φτάσει πρώτος και παραμείνει z λεπτά, τότε η πιθανότητα να συναντη-

50 . 50 ( 60 - z ) . ( 60 - z ) -- + --'----------'-------'----------'--θούν είναι: Ρ (Ε ) = 1 - 2 2 . Για να είναι Ρ(Ε) = 1 /2 θα πρέπει

3600 50 . 50 ( 60 - z ) . ( 60 - z ) -- + --'----------'-------'----------'--

1 _:,2 ____ _:2=------ = - � 3600 2

� 2500 + 3600 - 60z - 60z+z2 = 3600 � � z2 - 1 20z + 2500 = Ο

Το τριώνυμο έχει δυο ρίζες ρ 1 = 93 , 1 7 και ρz = 26,83 . Επειδή z<60 συμπεραίνουμε ότι ο απαιτούμενος χρόνος είναι (z = 26,83) περίπου 27 λεπτά.

Βιβλιογραφία 1 . Johnson, Α. ( 1 995) . Geometric Probability . COMAP, lnc 2 . Patrick, D. Art of Problem Solving: Introduction to Counting and Probability.

www .artofproblemsolving.com

ΕΥΚΛΕΙΔΗΣ Β' 81 τ.l/75

Page 78: Ευκλειδης Β 83

«Στο βαθμό που οι νόμοι των μαθηματικών αναφέ­ρονται στην πραγματικότητα δεν είναι απόλυτοι. Στο

βαθμό που είναι απόλυτοι δεν αναφέρονται στην πραγματικότητα. »

Albert Eίnsteίn

Παναγιώτης Π. Χριστόπουλος

Επίσης ο Eίnsteίn έλεγε: «Είναι ευκολότερο να διασπασθεί ένα άτομο από το να αλλάξει μια πεποίθηση» δηλαδή , όλοι οι άνθρωποι έχουν την τάση να αντιστέκονται στις αλλαγές και στις καινοτομίες καταναλώνοντας για αυτό μεγάλο μέρος από την ενέργειά τους. Αυτό είναι γνωστό από παλιά και ισχύει για όλους πολύ περισσότερο για τους επιστήμονες, που δεν θα μπορέσουν ποτέ να αποδεχθούν νέες θεωρίες, επειδή αυτές για το επιστημονικό κατεστημένο σημαίνουν απώλεια ισχύος, χρήματος και γοήτρου .

Γνωρίζετε ότι: •:• Ο Blaise Pascal [ 1623-1662] ήταν ένα παιδί θαύμα; Γεννήθηκε στο

Κλερμόν της Γαλλίας. Σε ηλικία 3 ετών έχασε τη μητέρα του. Ο πατέ­ρας του ήταν φοροεισπράκτορας, ερασιτέχνης μαθηματικός και του είχε απαγορεύσει τη μελέτη μαθηματικών βιβλίων πριν γίνει 1 5 ετών ! Όμως ότι είναι απαγορευμένο εξάπτει την περιέργεια, έτσι ο Pascal δώδεκα ετών έδειξε μεγάλη πρόοδο στη Γεωμετρία και ο πατέρας του αναγκάστηκε να του επιτρέψει τη μελέτη στο έργο, Στοιχεία του Ευ­κλείδη . Ως έφηβος κατασκεύασε την πρώτη aριθμομηχανή που ονο­μάστηκε «Πασκαλίνα». Όμως η επιστολή που έστειλε στις 24 Αυγού­στου 1654 στον Pierre de Fermat για το «πρόβλημα των πόντων» έμελε να αλλάξει τη ζωή των ανθρώπων αφού με αυτή μας έδειξε το δρόμο για τη θεωρία των πιθανοτήτων. Πολλές παγκοσμιοποιμένες ε­ταιρείες σήμερα, ασφαλιστές, δημοσκόποι, Google, Καζίνο, στηρίζουν • • • • • - . - . - - . - - - - • • τη λειτουργία τους στις πιθανότητες. Βέβαια ο πρώτος που υπολόγισε πιθανότητες συστηματικά ή­ταν ο Gerolamo Cardano [ 1501-1576] στο βιβλίο του «Liber de Ludo Aleae». Ο Cardano, στο βι­βλίο του ασχολήθηκε με την ειδική περίπτωση ισοπίθανων γεγονότων, όπου κατάλαβε ότι η πιθανό­τητα να συμβεί ένα γεγονός είναι ο λόγος του αριθμού των ευνοϊκών αποτελεσμάτων προς τον ολικό αριθμό αποτελεσμάτων. Η συστηματική όμως μελέτη του αντικειμένου των πιθανοτήτων οφείλεται στην αλληλογραφία των Pascal και F ermat. Η αλληλογραφία μεταξύ τους άρχισε από τον Pascal, ο οποίος ήθελε να συμβουλευτεί τον Fermat σχετικά με προβλήματα που του δόθηκαν από τον Cheνa­lier de Mere, έναν ευγενή της αυλής του Λουδοβίκου του 4ου , γνωστό συγγραφέα και παίκτη τυχε­ρών παιχνιδιών. Οι αρχαίοι Έλληνες; . . . Ήταν δυνατόν να μην ασχοληθούν; . . .

ΕΥΚΛΕΙΔΗΣ Β' 83 τ. 3/76

Page 79: Ευκλειδης Β 83

------------- Τα Μαθηματικά μας διασκεδάζουν -------------Ο Αριστοτέλης (384-322 π.Χ.] διατύπωσε τη διάκριση μεταξύ των λέξεων γνώση και γνώμη. Θεώρησε δηλαδή ότι η γνώση αφορά σε κάτι που είναι σωστό ή λάθος, ενώ η γνώμη σε κάτι που μπορεί να είναι σωστό ή λάθος. Έδωσε επίσης τις έννοιες του τυχαίου, του aπροσδόκητου και της

σχετικής συχνότητας. Θεωρούσε όμως ότι το τυχαίο δεν είναι επιστημονική έννοια, οφείλεται στη δική μας αδυναμία να ερμηνεύσουμε τα φαινόμενα. Το τυχαίο χρησιμοποιήθηκε και στην

Αθηναϊκή πολιτεία. Στη νομοθεσία του Δράκοντα [624 ή 62 1 π.Χ.] η επιλογή των αρχόντων (βουλευτές, στρατηγοί) γινόταν με κλήρο και όχι με εκλογή . Όμως όσοι κληρώνονταν για μια θητεία δεν μετείχαν στην επόμενη κλήρωση .

•:• Τα πρόβατα που θάβονται από χιονοστιβάδες, μπορούν να επιζήσουν μέχρι και 2 εβδομάδες; •:• Οι Εσκιμώοι χρησιμοποιούν ψυγεία για να προστατεύσουν το φαγητό τους από το ψύχος κρατώντας

το σε σταθερή θερμοκρασία μεγαλύτερη από τη θερμοκρασία του περιβάλλοντος; •:• Χρειάζονται 1 6 θέσεις να είναι εξαρχής στον πίνακα του Sudoku συμπληρωμένες προκειμένου να

έχει το παιχνίδι μία και μοναδική λύση; Το ερώτημα άφηνε άγρυπνους τους μαθηματικούς εδώ και χρόνια. Τώρα όμως, ένας Ιρλανδός ερευνητής ο Γκάρυ Μακ Γκουάιρ υποστηρίζει ότι έλυσε το γρίφο βάζοντας σε έναν υπερυπολογιστή να τρέξει ένα ειδικό αλγόριθμο και σε λίγες μέρες η απάντηση που βρήκε είναι 16 θέσεις.

•:• Επειδή κανείς μέχρι σήμερα δεν είχε καταφέρει να επινοήσει ένα παιχνίδι με 1 6 γνωστούς αριθμούς, το οποίο να έχει μία και μοναδική λύση , οι μαθηματικοί είχαν διατυπώσει την εικασία ότι αυτός είναι ο ελάχιστος δυνατός αριθμός των γνωστών θέσεων στον πίνακα του Sudoku.

':• Ένας τρόπος να επιβεβαιώσει κανείς αυτή την εικασία είναι να λύσει όλα τα πιθανά παιχνίδια με 1 6 συμπληρωμένες θέσεις. Αυτό όμως, θα χρειαζόταν έναν αιώνα. Θυμίζουμε ότι οι πιθανές λύσεις είναι 6, 7 δισεκατομμύρια τρισεκατομμύρια όπου από αυτές οι 5,4 δισεκατομμύρια είναι ανεξάρτητες. Η μέθοδος αυτή που ακλούθησαν για τον περιορισμό των μαθηματικών υπολογισμών σε «διαχειρίσιμη>> έκταση μπορεί να εφαρμοστεί και σε άλλα , εντελώς διαφορετικά επιστημονικά πεδία. Τέτοια μπορεί να είναι η ανάλυση της ακολουθίας των γονιδίων του DNA των ανθρώπων των ζώων και των φυτών.

Ένας αστεροειδής και από ένας κρατήρας στη Σελήνη και στον Άρη έχουν το όνομά του Μιλουτίν Μιλάνκοβιτς; Ο Μιλάνκοβιτς ( 1879-1958] ήταν Σέρβος αστρονόμος, μαθηματικός, γεωφυσικός και πολιτικός μηχανικός περισσότερο γνωστός για τους λεγόμενους «Κύκλους του Μιλάνκοβιτς)) που ερμηνεύουν αστρονομικώς τις εποχές των παγετώνων και γενικά τις μακροχρόνιες μεταβολές στο κλίμα της Γης και των άλλων πλανητών. Απο την έδρα των Εφαρμοσμένων Μαθηματικών του Βελιγραδίου ενδιαφέρθηκε για το ηλιογενές κλίμα και τις θερμοκρασίες που επικρατούν πάνω στους πλανήτες. Έργα του «Μαθηματική θεωρία των θερμικών φαινομένων που προκαλεί η ηλιακή ακτινοβολία)), «η καμπύλη του ηλιακού φωτός που

δέχεται η γήινη επιφάνεια)), «Μαθηματική Κλιματολογία και αστρονομική θεωρία των μεταβολών του κλίματος)) και «Κανών του ηλιακού φωτός που προσπίπτει στη Γη και η εφαρμογή του στο πρόβλημα των Εποχών των Παγετώνων)), Ιστορία της Αστρονομίας, «Μέσα στο Χώρο και στους Αιώνες)), «Μέσα από το Βασίλειο της Επιστήμης)), «Αναμνήσεις, εμπειρίες και όραμα)) . Το 1 965 οι έρευνες ιζημάτων στα βάθη των ωκεανών επαλήθευσαν τη θεωρία των «Κύκλων του Μιλάνκοβιτς)), αφού η περιοδικότητα που ανακαλύφθηκε

ΕΥΚΛΕΙΔΗΣ Β ' 83 τ. 3177

Page 80: Ευκλειδης Β 83

------------- Τα Μαθηματικά μας διασκεδάζουν ------------­στα στρώματά τους ( 1 00.000 έτη) προσέγγιζε πολύ τον μακρύτερο κύκλο Μιλάνκοβιτς. Ήταν μέλος πολλών επιστημονικών εταιρειών του κόσμου [Στο τεύχος 80 του Ευκλείδη Β ' είχαμε αναφερθεί και για το έργο του στα ημερολόγια] . !

I Ί ι ίφο ι κ ω σπιι�οκι :φαλιi:ς , J f : ι 'η μικο νησι

Σε ένα ερημικό νησί έχει ξεμείνει χωρίς μεταφορικό μέσο μια 6μελής οικογένεια πατέρας, μητέρα και τα ανήλικα παιδιά τους δυο αγόρια και δυο κορίτσια. Για καλή τους τύχη πέρασε από το νησί ένας αστυ­νόμος που μεταφέρει με μια βάρκα έναν κρατούμε­νο. Πώς θα περάσουν όλοι στην απέναντι ακτή αν:

1 . Η βάρκα χωράει μόνο δυο άτομα 2. Τη βάρκα μπορούν να οδηγήσουν μόνο ο

αστυνόμος και οι δυο γονείς 3 . Ο κρατούμενος δεν πρέπει να μείνει με

κανένα μέλος της οικογένειας χωρίς την παρουσία του αστυνόμου

4. Κανένα κορίτσι δεν μένει με τον πατέρα χωρίς την παρουσία της μητέρας

5 . Κανένα αγόρι δεν μένει με την μητέρα χω­ρίς την παρουσία του πατέρα

Υπάρχει άραγε τρόπος να περάσουν όλοι απέναντι; - � 1 Ι ·' ,'

Δύο παίκτες έχουν στοιχηματίσει το ίδιο ποσό ο καθένας. Το παιχνίδι είναι: Κερδίζει όποιος μετά από 5 ρίψεις ενός ζαριού(η καλύτερα νομίσματος Κ,Γ) κερδίσει τις περισσότερες παρτίδες. Το πρόβλημα όμως είναι: Πώς θα μοιραστούν τα χρήματα αν το παιχνίδι διακοπεί μετά από τρείς ρίψεις, όταν ο ένας παίκτης έχει κερδίσει δύο παρτίδες και ο αντίπαλός του μία;

Μαντέψτε την ηλικία του φίλου σας και το νούμε­ρο των παπουτσιών του. Πείτε του : πολλαπλασίασε το νούμερο των πα­πουτσιών σου επί 5 και πρόσθεσε 50, ύστερα πολλαπλασίασε το αποτέλεσμα επί 20 και στη συ­νέχεια πρόσθεσε 1012. Τώρα αφαίρεσε από το αποτέλεσμα τη χρονιά που γεννήθηκες. Αφού σας πει τον αριθμό που βρήκε εσείς τώρα ξέρετε την ηλικία του και το νούμερο των παπουτσιών του . Πώς;

Γα {ι.λογα Ο κυρ Γιώργης όταν έκανε την διαθήκη του είχε 20 άλογα και η εντολή του ήταν να τα μοιραστούν οι 3 γιοι του ως εξής: ο πρωτότοκος να πάρει τα μισά, !ο δεύτερος το 1 /4 και ο μικρότερος το 1 /5 . Όμως μέχρι που ν α ανοίξουν την διαθήκη ο ι γιοί του το ένα άλογο ψόφησε, έτσι έμειναν τα 1 9 άλο­γα. Πώς θα τα μοιράσουν τώρα;

Ευχαριστούμε για τις προτάσεις του, τον φυσικό Δημήτρη Στρατικόπουλο από την Κυπαρισσία και όλους όσους στηρίζουν τη στήλη αυτή .

Απαντ ' σει Το ερη μικό νησί Ονομάζουμε τα άτομα Π=πατέρας, Α 1 ,Α2=αγόρια, Μ=μητέρα, Κ1 ,Κ2=κορίτσια, Ας=αστυνόμος , Κρ=κρατούμενος. Οι διαδρομές είναι 1 7 ως εξής:

1η από Ν�Ξ Ας+Κρ, 2η Ξ�Ν Ας, 3η Ν�Ξ Ας+Α 1 , 4η Ξ�Ν Ας+Κρ, 5η Ν�Ξ Π+Α2, 6η Ξ�Ν Π, 7η Ν�Ξ Π+Μ, 8η Ξ�Ν Μ, 9η Ν�Ξ Ας+Κρ,

10η Ξ�Ν Π, l l η Ν�Ξ Π+Μ, 12η Ξ�Ν Μ, 13η Ν�Ξ Μ+Κ 1 , 14η Ξ�Ν Ας+Κρ, 15η Ν�Ξ Ας+Κ2, 1 6η Ξ�Ν Ας, 1 7η Ν�Ξ Ας+Κρ όλοι είναι στην ξηρά.

Το πρόβλημα των πόντων Παρόμοιο ήταν το πρόβλημα με το οποίο καταπιά­στηκαν στην αλληλογραφία τους ο Pascal και F ermat και έδειξαν έτσι στην ανθρωπότητα το δρόμο για τον επιστημονικό κλάδο των πιθανοτή­των. [σχετικό βιβλίο «Το τελευταίο παιχνίδι» εκδ : ΤΡΑΥ ΛΟΣ] . Με το πρόβλημα αυτό ασχολήθηκαν για πολλά χρόνια οι μαθηματικοί, η λύση είναι % και Υ-ι. 'l άνπψι: Από τον 4ψήφιο αριθμό που σας είπε τα δύο πρώ­τα ψηφία είναι το νούμερο των παπουτσιών του και τα άλλα δύο η ηλικία του . π.χ. 5·43(νούμερο παπουτσιών)= 2 1 5, 2 1 5+50= 265, 265·20= 5300, 5300+1 Ο 1 2= 63 12, 63 12-1 995 [χρ. γέννησης]= 43 17. Ή αλλιώς: (5·43+50)·20+1012-1995= 5·(43+10)·20+1012-1995=(43+10)· 100+1012-1995= 4300+ 1 000+ 1 ο 12-1995= 4300+ 20 12-1995= 4300+ 1 7=43 1 7 [νο : παπούτσι 43, ηλικία 1 7] . Τα άλογα Μια χαριτωμένη λύση λέει: Θα αγοράσουν ένα άλογο και στη συνέχεια αφού εκτελεστεί η διαθή­κη θα πάρουν 1 0 ,5 ,4 άλογα, θα πουλήσουν πάλι το άλογο που αγόρασαν να πάρουν τα λεφτά τους. Όμως το πρόβλημα είναι στη διαθήκη, διότι το Υ2+ 1 /4+ 1 /5=1 9/20 δηλαδή το 95% των αλόγων πρέπει να δοθεί στους γιούς, το ένα άλογο περίσ­σευε από τη διαθήκη .

Σz6λιο της Σ Ε : Στη σελ. 77 του τ. 82 της στήλης ΜΔ στην ερώτηση . Τι αριθμός είναι το ίί, θεωρούμε απαραίτητο να τονίσουμε ότι με το σύμβολο ίί δεν εν­νοούμε ένα μόνον αριθμό αλλά ένα σύνολο αριθμών. Γι' αυτό σε πολλά βιβλία Μιγαδικής ανάλυσης αντί ίί

γράφουν {ίί} . Πράξεις λοιπόν, με το ίί δεν είναι μονο­σήμαντα ορισμένες.

ΕΥΚΛΕΙΔΗΣ Β' 83 τ. 3/78

Page 81: Ευκλειδης Β 83

ΔΡΑΣΕΙΣ

§eenBυus 2012 Από 6-1 1 Μαρτίου 2012 γίνεται στη Βουλγαρία η φοι­τητική Ολυμπιάδα Seemous 20 1 2, με τη συμμετοχή πολλών χωρών. Θυμίζουμε ότι η Ελληνική ομάδα το 20 1 1 είχε κατακτήσει 2 χρυσά μετάλλια, 2 αργυρά και 2 χάλκινα. Σε αυτή την προσπάθεια συμμετέχουν οι φοι­τητές από το Μαθηματικό (7] : Α. Εσκενάζης, Δ. Μπο­γιόκας, Γεώργιος Παπάς, Κ. Ζέμας, Κ. Τσουβαλάς, Αθ. Τσαρέας, Αν. Καφετζόπουλος, από το Πολυτεχνείο [ 1 ) : Σ . Κωνσταντινίδης και από την Πληροφορική [1 ] : Μ. Αγγελής. Την Ελληνική ομάδα συνοδεύει ο καθηγητής Ιάκωβος Ανδρουλιδάιcης από το ΕΚΠΑ. Περισσότερα για τα αποτελέσματα στο [ www. seemous.eu ] .

ICME 20 1 2 Με ιδιαίτερα αφιερώματα και πολλές εκθέσεις στο χώρο της τέχνης και των Μαθηματικών και σε ειδικές αίθου­σες που έχουν ενδιαφέροντα θέματα, όπως: τα Παζλ, «τα παιχνίδια και τα Μαθηματικά» τα μαγικά σόου, . . . θα γίνει στη Νότια Κορέα [Σεούλ] το φετινό ICME 8-15 Ιουλίου 2012. Επίσης θα υπάρξει ξεχωριστή αίθουσα για τα Μαθηματικά της Ασίας και της Αυστραλίας. Το πρό­γραμμα περιλαμβάνει όλες τις ενότητες των μαθηματι­κών στο χώρο της εκπαίδευσης και της έρευνας. Το πα­γκόσμιο αυτό συνέδριο γίνεται κάθε τέσσερα χρόνια. Ξεκίνησε το 1 969 από τη Λυών της Γαλλίας και συνεχί­στηκε το 1 972 [Αγγλία] , 1 976 [Γερμανία], 1 980 [ΗΠΑ] , 1 984 [Αυστραλία], 1 988 [Ουγγαρία, που έγινε στη Βου­δαπέστη και συμμετείχε και η ΕΜΕ με 35 συνέδρους], 1 992 [Καναδά] , 1 996 [Ισπανία], 2000 [Ιαπωνία] , 2004 [Δανία], 2008 [Μεξικό] . Σε αρκετά από αυτά έχουν, κα­τά καιρούς, συμμετάσχει ατομικά και αρκετοί Έλληνες ερευνητές. Περισσότερα στο [www . icme 1 2.org] και στον Ευκλείδη Γ τεύχος 26.

Η }lit:�f.Λδα ΝΙαθημα·η�αο ν στα Τρίκαλα

Την Κυριακή 19-2-2012 το Παράρτημα Ν. Τρικάλων της ΕΜΕ διοργάνωσε ημερίδα Μαθηματικών διαλέξεων και στη συνέχεια κοπή πίτας. Αρχικά ο Πρόεδρος του Παραρτήματος κ. Στέλιος Αμπράζης υπογράμμισε: «Σήμερα, διοργανώνουμε ημερίδα διαλέξεων και μαζί μας είναι τρεις εξαίρετοι και διακεκριμένοι Καθηγητές του Πανεπιστημίου Αθηνών. Θέλω να τους ευχαριστή­σω που αποδέχτηκαν την πρόσκλησή μας και ήρθαν στη πόλη μας. Ένας από τους σκοπούς των δράσεών μας εί­ναι η αναβάθμιση της Παιδείας του τόπου μας και ιδιαί­τερα της Μαθηματικής Παιδείας. Η σημερινή μας ημε­ρίδα εντάσσεται σ' αυτό το πλαίσιο». Ακολούθως ο κ. Διονύσιος Λάππας χαιρέτησε την εκδήλωση, εκ μέρους του Δ.Σ. της ΕΜΕ και του Προέδρου της ΕΜΕ και εκ­φράστηκε αρκετά θετικά για το τοπικό Παράρτημα καθώς και για τους Τρικαλινούς μαθηματικούς, αλλά και τους Τρικαλινούς μαθητές* που διακρίνονται. Ο πρώτος ομιλητής ήταν ο κ. Στυλιανός Νεγρεπόντης, Ομότιμος Καθηγητής Μαθηματικού Τμήματος Πανεπι­στημίου Αθηνών και ανέπτυξε το θέμα: «Τα παράδοξα

ΑΠΡΟΟΠΤΑ

του Ζήνωνος και η ασυμμετρία». Ακολούθως ο κ. Δι­ονύσιος Λάππας, Αναπληρωτής Καθηγητής και Διευθυ­ντής Τομέα Άλγεβρας - Γεωμετρίας του Μαθηματικού Τμήματος Πανεπιστημίου Αθηνών, μέλος του Δ.Σ. της Ε.Μ.Ε. μίλησε με θέμα: «Γεωμετρία και συμμετρία». Τον κύκλο έκλεισε η κα. Βασιλική Φαρμάιcη, Καθηγή­τρια Μαθηματικής Ανάλυσης του Μαθηματικού Τμή­ματος Πανεπιστημίου Αθηνών με τίτλο της διάλεξής της: «Η ση­μασία του συνδυα­στικού τρόπου σκέ­ψης)) . Όλες τις ομιλίες παρακολούθησαν στην κατάμεστη αί­θουσα ο Δήμαρχος του Δήμου Τρικκαίων κ. Χρ. Λάππας, η Α- Δ. Λάππας ντιδήμαρχος παιδείας κα. Β. Κάκλα, ο Αντιδήμαρχος κ. Κων. Μανώλης, ο Καθηγητής του Πανεπιστημίου Θεσσαλίας κ. Γ. Δή­μος, ο Διευθυντής Δ/θμιας Εκπαίδευσης Τρικάλων κ. Θεόδ. Τσέλιος, ο Σχ. Σύμβουλος Μαθηματικών κ. Δ. Ντρίζος, ο τέως Σχ. Σύμβουλος Μαθηματικών κ. Ν. Παπανικολάου και περίπου 1 50 μαθηματικοί της περιο­χής, που συζήτησαν στη συνέχεια με τους εισηγητές.

Στη συνέχεια το Παράρτημα Ν. Τρικάλων σε εστιατό­ριο κεντρικού ξενοδοχείου της πόλης πραγματοποίησε την κοπή της πρωτοχρονιάτικης πίτας με παρευρισκό­μενους και τους τρεις προσκεκλημένους Καθηγητές. Ο κ. Δ. Λάππας εκπροσωπώντας το Δ.Σ. της ΕΜΕ έκοψε την πίτα και ευχήθηκε διαρκή επιτυχία στις εκδηλώσεις του δραστήριου Παραρτήματος του Ν. Τρικάλων.

(* Οι Θεσσαλοί της Γερμανίας προσκάλεσαν στο 8° Φε­στιβάλ τους, που θα πραγματοποιηθεί στο Ντίσελντορφ της Γερμανίας, από κάθε Θεσσαλικό Παράρτημα της Ελ­ληνικής Μαθηματικής Εταιρείας των αντίστοιχων τεσσά­ρων νομών, τρεις διακεκριμένους μαθητές με ένα συνοδό. Η φετινή τους εκδήλωση είναι αφιερωμένη στην Εmστήμη των Μαθηματικών και mo συγκεκριμένα στις μαθηματι­κές διακρίσεις και εmτυχίες που είχαν Θεσσαλοί μαθητές σε Μαθηματικούς Διαγωνισμούς, όπου και συμμετείχαν. Μια αξιέπαινη πρωτοβουλία που δεν πρέπει να περάσει α­παρατήρητη στους δύσκολους καιρούς που περνάει η χώρα μας, . . . Πολλά μπράβο στους διοργανωτές.

ΕΥΚΛΕΙΔΗΣ Β ' 83 τ.3179

Page 82: Ευκλειδης Β 83

-------- Δράσεις - Γεγονότα - Διαγωνισμοί - Βραβεία - Συνέδρια - Απρόοπτα

Νάξος 201 1 Οι Κυκλάδες, τα νησιά, με τη γεωγραφική ιδιομορφία, επιμένουν, και ξεχωρίζουν . . . Έτσι στις 10 Δεκεμβρίου

Ο Γ.Γ. της ΕΜΕ Μ. Κρητικός

201 1 πραγματοποιήθηκε στη Νάξο, εκδήλωση από την Ελληνική Μα­θηματική Εταιρεία (Ε­ΜΕ) για τις βραβεύσεις των μαθητών των δημο­τικών σχολείων που συμμετείχαν στο διαγω­νισμό «Παιχνίδι και Μαθηματικά». Η ΕΜΕ

για πρώτη φορά διοργάνωσε ειδική εκδήλωση στη Νάξο. Στην εκδήλωση πήραν μέρος μεταξύ άλλων οι μαθητές, οι γονείς και οι τοπικοί φορείς, ενώ από την ΕΜΕ ο Γε­νικός Γραμματέας Μανόλης Κρητικός, ο Ανδρέας Τριανταφύλλου, υπεύθυνος του περιοδικού «Ο μικρός Ευκλείδης» και ο Αδάμ Αγγελής, Πάρεδρος του Παιδα­γωγικού Ινστιτούτου. Η αλήθεια είναι ότι τα νησιά είναι δύσκολα στην επικοινωνία, όμως μάθαμε ότι δραστή­ρια μέλη της μαθηματικής κοινότητας προσπαθούν να δημιουργήσουν Παράρτημα στις Κυκλάδες. Ευχάριστη είδηση και ελπίζουμε να τα καταφέρουν . . .

eu�ath Η U,}"'''Ό- 1 2 Στις 21 - 25 Μαρτίου 2012 γίνεται στην Σόφια της Βουλ­γαρίας το 3° Διεθνές Μαθηματικό Μαθητικό Συνέδριο στα Μαθηματικά. Περιλαμβάνει πρωτότυπες εργασίες μαθητών στο χώρο των Μαθηματικών και σε διάφορους άλλους το­μείς που σχετίζονται με την μαθηματική επιστήμη όπως η Οικονομία, η Μουσική, η Φιλοσοφία, η Πληροφορική, το Διάστημα, η Τεχνολογία, η Στατιστική, η Ιστορία κ.λ.π.

Βράβευση της Ε Μ Ε Έγινε από την Interamerican τον Οκτώβριο του 20 1 1 ει­

δική τελετή βρά­βευσης των μαθη­τών που διακρίθη­καν στην Μαθημα­τική Ολυμπιάδα (IMO) της Ολλαν­δίας το 20 1 1 . Από το Δ.Σ. της ΕΜΕ συμμετείχαν ο Α ' Αντιπρόεδρος της ΕΜΕ Γεώργιος

Δημάκος, ο Γ. Γ. της ΕΜΕ Μανόλης Κρητικός όπως και η Ελληνική ομάδα των μαθητών.

Η Ελληνική ομάδα των μαθητών που διακρίθηκε

Εσείς γρdφετε . . . . . . εμείς aπavτtiμε

Ο όγκος της αλληλογραφίας, για τον Ευκλείδη Β ', είναι πολύ μεγάλος. Θυμίζουμε ότι τα μέλη της επιτροπής, προσπαθούν να μελετούν με προσοχή τα γράμματά

σας και να αξιοποιούν όλες τις εργασίες σας, αρκεί να έχουν

σχέση με την ύλη και το ύφος του περιοδικού. Κάποιες φορές, μερικά άρθρα έρχονται χειρόγραφα και τούτο δυσκολεύει το έργο μας, που μη ξεχνάμε, είναι από όλους μας, σε εθελοντική βάση προσφοράς. Βέ­βαια χαρά μας είναι η επικοινωνία μαζί σας, με οιονδή­ποτε τρόπο και για οποιοδήποτε θέμα. Η στήριξή σας στο περιοδικό και η φροντίδα για τη διάδοσή του, είναι η ελπίδα ότι το περιοδικό μας θα συνεχίσει να προ­σφέρει στους μαθητές και την Ελληνική Μαθηματική παιδεία, . . . Ευχόμαστε το καλύτερο για όλους . . . Έτσι γι' αυτό το τεύχος έχουμε:

Γιάννης Χελιώτης, [μαθητής από το Ναύπλιο] διαβάζοντας για «Το κόσκινο του Ερατοσθένη» σκέ­φτηκε ένα δικό του τρόπο, να βρει το πλήθος των πρώ­των αριθμών από το 1 έως το 1 00, με αριθμητική πρό­οδο. Μπράβο του, μόνο να συμπληρώσουμε ότι: i) Μέ­χρι το 1 00 έχουμε 25, όχι 26 πρώτους. ii) Όταν το διά­στημα γίνει πολύ μεγάλο είναι σχεδόν αδύνατο να βρεθούν οι πρώτοι αριθμοί με αυτόν τον τρόπο . . . «Βρίσκω τα πολλαπλάσια του 2, δηλαδή βρίσκω όλα τα πολλαπλάσια των άρτιων αριθμών από το 1 έως το 1 00. Παίρνω ως α1= 4, (γιατί το 2 είναι πρώτος αριθμός), αν = 1 00, ω = 2 και βρίσκω το ν δηλαδή : αν = α1 + (ν- 1 )ω, 1 00 = 4 + ( ν - 1 )2, ν=49. Βρίσκω τα πολλαπλάσια του 3 . Παίρνω ως α 1=9 (γιατί είναι το πρώτο περιττό πολ­λαπλάσιο του 3), αν = 99 και ω = 6 (γιατί θέλω να υπο­λογίσω τα πολλαπλάσια των περιττών αριθμών). Δηλα­δή : αν = α1 + (ω- 1 )ω, 99=9+ (ν- 1 )6, ν= 1 6 .Βρίσκω τα πολλαπλάσια του 5 . Παίρνω ως α1 = 1 5 (γιατί είναι το πρώτο περιττό πολλαπλάσιο του 5), αν =95 και ω = 1 0. Δηλαδή : αv=α1 +(ν- 1 )ω, 95 = 1 5 + (ν- 1 ) 1 0, ν=9. Επειδή μέσα σ' αυτό το πλήθος βρίσκονται και πολλαπλάσια του 3 , που τα υπολόγισα προηγουμένως , με την βοή­θεια δεύτερης αριθμητικής προόδου θα τα υπολογίσω και θα τα αφαιρέσω από τα προηγούμενα. Παίρνω α1= 1 5 (γιατί είναι κοινό πολλαπλάσιο του 5 και του 3) αv=75 ( για τον ίδιο λόγο ) και ω = 30.

Δηλαδή : αv=α1 + (ν- 1 )ω , 75 = 15 + (ν- 1 )30, ν=3 . Επομένως το πλήθος των αριθμών που πολλαπλασιάζο­νται μόνο με το 5 είναι 9-3 = 6. Επειδή δεν μπορώ να χρησιμοποιήσω αριθμητική πρόοδο για να βρω τα πολ­λαπλάσια μόνο του 7, παρατηρώ ότι είναι οι αριθμοί 49, 77, 9 1 , δηλαδή τρεις αριθμοί. Επειδή τα πολλαπλάσια του 9 συμπίπτουν με τα πολλαπλάσια του 3 , άρα δεν τα υπολογίζω στο πλήθος. Τελικά το πλήθος των πρώτων αριθμών από το 1 έως το 1 00 είναι:

1 00 - ( 49 + 1 6 + 6 + 3 ) = 1 00 - 74 = 26 Με αυτό τον τρόπο μπορώ να υπολογίσω το πλήθος των πρώτων αριθμών, όχι μόνο από το 1 έως το 1 00, αλλά σε οποιοδήποτε διάστημα θετικών ακεραίων εmθυμώ» . . .

ΕΥΚΛΕΙΔΗΣ Β ' 83 τ.3/80

Page 83: Ευκλειδης Β 83

------- Δράσεις - Γεγονότα - Διαγωνισμοί - Βραβεία - Συνέδρια - Απρόοπτα

Τα άλυια προβλήματα της Έχουν έρθει δύο εργασίες ακόμη για τον τετραγωνισμό του κύκλου [Κ. Θεοδώρου - Αθήνα] , [Αλέξανδρος Παπαλεξάτος - Βίγλι Κεφαλονιάς, . . . με καλή προ­σέγγιση και μεθοδολογία] , και την τριχοτόμηση της γωνίας [Μ. Δημητριάδης, Κ. Δεληβέρης - Αθήνα] . Όπως είχαμε αναφέρει και παρουσιάσει στο προηγού­μενο τ. 82 [σελ. 78 ] , για το εφικτό ιστορικά, αυτών των κατασκευών (με κανόνα και διαβήτη). Δεν είχαμε μιλήσει για τον διπλασισμό του κύβου (Δήλιο Πρόβλη­μα] . Στο Δήλιο Πρόβλημα πήραμε από τον Αργύριο Β. Σπυρίδωνος [Αθήνα] μια προσεγγιστική και καλά προ­σεγμένη λύση, με ενδεικτική βιβλιογραφία. Σημειώ­νουμε ότι στο τ. 82 υπήρξε αναφορά στο Δήλιο πρό­βλημα (Διπλασιασμός του κύβου) που για λόγους χώ­ρου, δεν εμφανίστηκε ευκρινώς.

Αρκετή και πλούσια αλληλογραφία, με επίκαιρα γεγονότα, στο επόμενο τεύχος. Σας ευχαριστούμε για την ανταπόκριση

Ευκλείδεια Γεωμετρία - Επιπεδομετρία του Μι­χάλη Καντικάκη - [Ηράκλειο Κρήτης 2009] χει­ρόγραφη επιμελημένη έκδοση με διευρυμένο επί­πεδο ενασχόλησης για όλους τους μελετητές. Θέσεις για τη θεωρία των πάντων (Διαλεκτική θεωρία. Η λογική των ισοδυναμιών) του Διονύση Σιοκορέλη - Μηχανολόγου Μηχανικού ΕΜΠ -

[Σκάλα Λακωνίας 201 1 ] . Γνωριμία μ ε την Προβολική Γεωμετρία του Γιάννη Περδικάρη -[ΕΜΕ Παράρτημα Ρόδου -

2009] χειρόγραφη επιμελημένη έκδοση [βασική ορολογία, θεώρημα του Πάππου ομογενείς συντε­ταγμένες τα θεωρήματα του Steiner, η αρχή του δυ­ϊσμού, . . . ]. Οι πολλές ασκήσεις τα λειτουργικά σχήματα και η μεθοδολογία γραφής καθιστούν την όλη προσπάθεια, σημαντική για τους ενδιαφε­ρόμενους στο χώρο της Γεωμετρίας.

Βιβλιοθη κών και ΜαΟηματικια. Η ΕΜΕ, στα πλαίσια ενίσχυσης των σχολικών βιβλιοθηκών αλλά και των δημοτικών βιβλιοθηκών της χώρας, για αυτή τη χρο­νιά που πέρασε , έστειλε μια σειρά των εκδόσεων της, ΔΩΡΕΑΝ, [περιοδικά και βιβλία] στη Λέρο και στους Λειψούς ( ακριτι­κά νηmά), στα σχολεία της ευρύτερης περιοχής της Αράχωβας (Δίστομο, Αράχωβα, Αντίκυρα), όπως επίσης και μια σεφά [περιοδικά και βιβλία] στα σχολεία του Μοσχάτου - Ταύρου όπου και βράβευσε τους πρώτους μαθητές της περιοχής. Επίσης ανανέωσε τις εκδόσεις της, στον ειδικό χώρο των διαδραστικών εκθέσεων στο Κέντρο Πολιτισμού, Ελληνικός Κόσμος, όπου προβάλλεται ο θεμελΙώδης ρόλος των Μαθηματικών στην εξέλιξη της επιστημονικής σκέψης. [Αναστάσιος Μπακάλης] .

Καλό ταξίδι δάσκαλε . . . . . . Αλησμόνητε καθηγητή μας Στέλιο Ανδρεαδάκη . . . . . . Ο καθηγητής Στέλιος Ανδρεαδάκης είχε μια συνεχή δραστηριότητα και προσφορά στα δρώμενα της Ελληνικής Μα­θηματικής Εταιρείας την τελευταία εικοσαετία. Πιο συγκεκριμένα από 1995 - 2001 ήταν μέλος του Δ.Σ. της ΕΜΕ με ουσιαστική συμμετοχή και δράση στο Bulle­tin ως editor ίη chief διαμορφώνοντας νέο αντιπροσωπευτικό σχήμα στο Editorial Board με την συμμετοχή εκ­προσώπων από όλα τα Μαθηματικά Τμήματα και τα Γενικά τμήματα των Πολυτεχνείων της χώρας μας. Επίσης την περίοδο από 2005 - 201 1 ήταν Α ' και Β ' Αντιπρόεδρος του Δ.Σ. της ΕΜΕ και Executive Committee στο Bulletin. Ενδεικτικά αναφέρω ότι είχε πάρα πολύ ουσιαστικό ρόλο στη Διεθνή Μαθηματική Ολυμπιάδα 2004 (1-ΜΟ 2004) ως μέλος της Οργανωτικής Επιτροπής άλλα και ως Chief Coordinator δηλαδή ως πρόεδρος της Επι­τροπής Βαθμολογητών. Επιπλέον είχε ουσιαστικό ρόλο στη συγγραφή των βιβλίων Άλγεβρας όλων των Τάξεων του Λυκείου από το 1 988 μέχρι σήμερα, και τέλος ήταν πρόεδρος της ΚΕΓΕ μετά το 1 990. Ήταν συνεπής - ακριβής - αξιοπρεπής - εργατικός, ότι αναλάμβανε το υλοποιούσε. Ανοιχτός Άνθρωπος με με­γάλη καρδιά, ήρεμος, χαμηλών τόνων, συναινετικός. Για μας τους νεότερους ήταν μέντορας για τον τρόπο συ­

μπεριφοράς και λειτουργίας. Αγαπητός με ευρύτατη αποδοχή σε όλη την Εκπαιδευτική κοινότητα, όπου μεταξύ των άλλων έχουν περάσει από τα χέρια του, πολλές γενιές Μαθηματικών. Ολόκληρο το Δ.Σ. της ΕΜΕ εκφράζει τα θερμά συλλυπητήρια του στην Αγαπημένη του γυναίκα Ρούλα, στις δύο κόρες του Κάτια και Στέλλα και την εγγονή του τις οποίες υπεραγαπούσε ήταν με δύο λόγια άριστος οικογενειάρχης με όλη τη σημασία της λέξης.

Φίλε και συνάδελφε Στέλιο καλό ταξίδι στην αιωνία Ζωή . . . [Γεώργιος Δημάκος] Ήταν ο aποχαιρετιστήριος λόγος προς τον αγαπημένο μας δάσκαλο, στο μεγάλο ταξίδι του. Παρευρέθησα\' στψ

εξόδιο ακολουθία, ο Πρόεδρος της ΕΜΕ Γ. Καλογερόπουλος, το Δ.Σ. της ΕΜΕ και πλήθος κόσμου. Λίγα λόγια για το δάσκαλο . . . Ο ομότιμος καθηγητής Στυλιανός Ανδρεαδάκης γεννήθηκε στην Αθήνα 79 ϊ.Ρό\1α ::ι:-:·.

Πήρε το πτυχίο του στα Μαθηματικά από το Τμήμα Μαθηματικών του Πανεmστημίου .\Η-. · . το 1958. Έλαβε το διδακτορικό του δίπλωμα από το Πανεmστήμιο του Λονδί\ 'ου ι σι•··"λ<Χ.:'�-Ξ"- ::

από το Queen Mary College, έτm ονομαζόταν τότε) κάτω από την καθοδήγηση :ο'.· •,x:.:-,· - :­

Philip Higgins. Πήρε τη θέση του καθηγητή κατ αρχάς στο Πανεmστήμιο τω" l ι·x.r.· .-:-. · · :: · :­

θέση του καθηγητή της Άλγεβρας στο Πανεmστήμιο Αθηνών, όπου παρέμεηc: ι.·.·� :-. �.- -: :� : ­δότηση του το 200 1 . Η συμβολή του στην θεωρία των Απείρων ΟμάδωΥ κα: :Ξ-.,::-.:Ξ:..·· : -_ .- _ : ­

λέτη της ομάδας αυτομορφισμών των ελευθέρων ομάδων ήτα" σηum-:-.:-...- ι = · -� �- · � : -.­

Ελλάδα με μεγάλη εmτυχία δύο διεθνή συνέδρια επί της Θεωρία.; Ομάι:-... -'· i-: : :- '" ·Ξ:..� -" . της διοίκησης στο Τμήμα Μαθηματικών (ως Διευθυντής του Τομέα λ_:._. . Ξ:Ξ·:·� :- : . _ :-:- _· ..: · • •

ως Αναπληρωτής Πρόεδρος). Επίσης ήταν ενεργό μέλος της ΕΜΕ για πολλά χρόνια. Ο Στυί.ια-. c•.: .-\ . : .. -.: . .. = . �. - .: : :

τη ζωή στις 1 0 Φεβρουαρίου 2012, 2.20 τα ξημερώματα . "&:r···υ.n..: Ρ ::..�-.:

ΕΥΚΛΕΙΔΗΣ Β ' 83 τ.3/81

Page 84: Ευκλειδης Β 83